Old Q&A – Other Questions

A 27-year-old HIV-positive man comes to the clinic for a periodic health maintenance examination. He contracted the disease 5 years ago from a former partner. He has been followed in the community health clinic since that time. He has no other medical history and takes only diazepam orally for anxiety. His last visit was 11 months ago. His temperature is 37.0 C (98.6 F), blood pressure is 140/85 mm Hg, pulse is 78/min, and respirations are 12/min. He has clear lung fields bilaterally, his skin is free of rashes or excoriations, and his abdomen is soft and nontender. Blood work drawn a few weeks ago reveals a CD4 count of 98 cells/mm3 and a hematocrit of 34% with an MCV of 95 fl. His last tuberculin skin test was 3 months ago and was read as 4mm and flat. In addition to initiating vitamin B12 and folate therapy for his patient, the most appropriate intervention at this time is

Top of Form

A. antibiotic prophylaxis for PCP pneumonia
B. antibiotic prophylaxis for tuberculosis
C. a skin test for tuberculosis
D. treatment for active tuberculosis infection
E. none is indicated based upon his CD4 count at this time
Bottom of Form

Explanation:
The correct answer is A. Opportunistic infections occur in people with HIV and define many of the components of the clinical syndrome known as AIDS. They are caused by a wide variety of pathogens and all have the common etiology that the host is susceptible due to the immune destruction brought on by the HIV virus. Prognosis depends on the type of infection and often, even with appropriate therapy, morbidity and mortality is high. There are means to prevent or reduce the likelihood of developing these infections. For this patient, his CD4 count of less than 200 cells/mm3 indicates that he should begin antibiotic prophylaxis, usually with TMP/SMX, for PCP pneumonia.

Starting antibiotic prophylaxis for tuberculosis (choice B) is not routine practice except in persons with a PPD-positive skin test, which this patient does not have.

Since the patient was tested for tuberculosis within the last year, and there is no evidence that he is anergic, there is no indication to test him again at this time (choice C).

The patient does not have active tuberculosis infection (choice D). Even if his PPD test were positive, active infection requires documentation of the organism in sputum by PCR or acid-fast staining.

Any patient with a CD4 count of less than 200 cells/mm3 should be considered for prophylaxis therapy for at least PCP pneumonia and toxoplasmosis (choice E).

A 33-year-old man comes to the office complaining of 3 months of severe headaches preceded by sweating and palpitations. He denies any chest pain or shortness of breath with these episodes. He has not taken any new medications and denies excessive caffeine intake and illicit drug use. His temperature is 37.0 C (98.6 F), blood pressure is 120/70 mm Hg, pulse is 78/min, and respirations are 14/min. Physical and neurological examinations are unremarkable. Thyroid function tests, a complete blood count, and a 24-hour urine cortisol are all within normal limits. Urine catecholamines are elevated. You order a CT scan of the abdomen, which shows an adrenal mass. The most appropriate next step is to
Top of Form

A. begin therapy with phenoxybenzamine

B. begin therapy with propanolol

C. consult a surgeon for emergent laparoscopic adrenalectomy

D. order serial CT scans to evaluate progression

E. perform a fine needle aspiration of the mass

Bottom of Form

Explanation:

The correct answer is A. This patient most likely has a pheochromocytoma. The classic triad of headache, palpitations, and diaphoresis should suggest this diagnosis. Patients are frequently hypertensive but sometimes the hypertension is paroxysmal and a normal blood pressure is found on exam. Prior to surgery this patient must be started on an alpha-blocker (phenoxybenzamine) to avoid intraoperative hypertensive crisis. Laparoscopic adrenalectomy is the treatment of choice for a pheochromocytoma.

Treatment with a beta blocker (choice B) is initiated only after a patient is started on an alpha blocker. Unopposed alpha stimulation can worsen hypertension in patients with a pheochromocytoma.

Laparoscopic adrenalectomy (choice C) is the treatment of choice for pheochromocytoma but medical stabilization is essential prior to surgery. Pheochromocytoma is not an indication for emergent surgery.

Serial CT scans (choice D) are important in the management of non-functioning adrenal masses (incidentaloma). When an adrenal mass is discovered incidentally and it is < 6cm repeat imaging is indicated to ensure that the mass is not quickly growing. A rapidly growing adrenal mass is suggestive of malignancy.

Fine needle aspiration (choice E) is an important tool in the workup of an adrenal mass. However, prior to aspirating any adrenal mass, pheochromocytoma must be ruled out to avoid precipitation of hypertensive crisis. Fine needle aspiration is not indicated in the management of pheochromocytoma since it can exacerbate a hypertensive crisis.

A 25-year-old man is admitted to the hospital after sustaining head injuries in a motor vehicle accident. On his 2nd day in the hospital, he shows you a sore on his penis that he developed a few days ago. He proudly admits to numerous sexual encounters in the past 5 years, and tells you that he has been tested for HIV every 6 months, and that the last negative test only was about 3 months ago. He is otherwise generally healthy, and does not take any medications on a regular basis. He denies any penile discharge in the past or present, and no history of other sexually transmitted diseases. On physical examination, there is painful lymphadenopathy of the left groin region. On the distal penis, there are 2 tender, ragged ulcers that appear punched out with surrounding hyperemia. The base of the ulcers are covered with a purulent, dirty exudate, which bleeds easily during the examination. This patient most likely has

Top of Form

A. chancroid and should be treated with azithromycin 1g orally in a single dose, while cultures and tests for other sexual transmitted diseases are performed
B. gonorrhea dermatitis and he should receive therapy for both gonorrhea and chlamydia, while cultures and tests for other sexual transmitted diseases are performed
C. granuloma inguinale and should be treated with trimethoprim-sulfamethoxazole, while cultures and tests for other sexual transmitted diseases are performed
D. lymphogranuloma venereum and should be treated with doxycycline, while cultures and tests for other sexual transmitted diseases are performed
E. a primary syphilitic chancre and serum RPR should be checked to confirm the diagnosis, while cultures and tests for other sexual transmitted diseases are performed
Bottom of Form

Explanation:
The correct answer is A. Chancroid is correct because it usually presents as an inflammatory papule that ruptures early with the formation of a ragged ulcer that lacks the induration of a chancre. The ulcers have undermined irregular edges surrounded by mild hyperemia, and the base is usually covered with purulent, dirty exudate. This is an infectious, contagious, ulcerative, sexually transmitted disease caused by the Gram-negative bacillus Haemophilus ducreyi. It is characterized by 1 or more deep or superficial tender ulcers on the genitalia and painful unilateral inguinal adenitis. However, the diagnosis of chancroid does not rule out syphilis and the subsequent development of syphilis should be anticipated since the incubation time for a chancre is longer than that of chancroid.

Gonococcal dermatitis (choice B) is incorrect because it is a rare infection that occurs mostly as erosions on the median raphe without urethritis. Grouped pustules on an erythematous base is the usual presentation.

Granuloma inguinale (choice C) is incorrect because it is a mildly contagious, chronic, granulomatous, locally destructive disease characterized by progressive indolent, serpiginous ulcerations of the groins, pubis, genitalia, and anus. The disease begins as single or multiple subcutaneous nodules, which erode through the skin to produce clean, sharply defined lesions, which are usually painless. The lesions typically demonstrate hypertrophic, vegetative granulation tissue which is soft, has a beefy-red appearance, and bleeds readily. The regional lymph nodes are usually not enlarged. This is caused by Calymmatobacterium granulomatis.

Lymphogranuloma venereum (choice D) is incorrect, because it is a sexually transmitted disease characterized by suppurative inguinal adenitis with matted lymph nodes, inguinal bubo with secondary ulceration, and constitutional symptoms. The primary lesion consists of herpetiform vesicle or erosion develops on the glans penis followed by bilateral lymphadenopathy. It is caused by Chlamydia trachomatis, serotypes L1, L2 and L3.

Primary syphilitic chancre (choice E) is incorrect, because it typically presents as a crusted superficial erosion that becomes a round or oval, indurated, slightly elevated papule, with an eroded, but not ulcerated surface that exudes a serous fluid. The lesion is usually painless. The regional lymph nodes on one or both sides are usually enlarged, firm, nontender and do not suppurate. With this said, when a patient presents with a penile ulcer, it is wise to obtain a serum RPR since patients can often times have more than one sexually transmitted disease.

A 66-year-old man with type II diabetes mellitus and atrial fibrillation comes to the emergency department with right body weakness and slurred speech that he noticed upon awakening in the morning. There were no complaints of word finding difficulties and no dysesthesia. He smokes a pack of cigarettes a day and “rarely exercises.” His wife hands you a prescription medicine bottle of warfarin and tells you that he has been taking this “for some time now.” His blood pressure is 210/95 mm Hg and his pulse is irregularly irregular. He has left-sided neglect with slurred speech and weakness of the right body; face and upper extremity worse than lower extremity. Routine chemistries and cell counts are normal. His INR is 1.7. A CT scan of the head shows a large left-sided subdural hematoma. The most appropriate next step is to
Top of Form

A. administer fresh frozen plasma and vitamin K

B. give her intravenous labetalol immediately

C. order a brain MRI

D. start him on heparin

E. tell the family that he will die and do nothing further

Bottom of Form

Explanation:

The correct answer is A. You should reverse the warfarin. The risk of him having a stroke from atrial fibrillation is far outweighed by the immediate goal of stopping the intracranial bleeding.

This patient requires a high blood pressure to maintain cerebral perfusion. If you drop his blood pressure with labetalol (choice B) this will cause his brain to lose oxygen. Also, subdural bleeds are venous, so the high arterial blood pressure would not be expected to increase the bleeding.

A brain MRI (choice C) will not provide additional useful information at this time.

Heparin (choice D) is contraindicated during an intracranial bleed. The goal is to reverse anticoagulation.

It is too early to predict outcome. You should not tell the family that he will die and do nothing further (choice E).

A 27-year-old Caucasian man comes to the office because of an 8-month history of fulminant episodes of chest palpitations, nausea, shortness of breath, and hyperventilation. Each episode lasts approximately 5 minutes. The patient states that he fears that he will die during these episodes and has had 4 emergency department assessments in the last month with no organic etiology found. The patient’s physical examination, thyroid function tests, complete blood count, and metabolic workup are within normal limits. The patient refuses psychiatric referral, but states that he will take any medication prescribed by you to “make me feel less out of control”. An appropriate pharmacologic regimen for this patient may include

Top of Form

A. carbamazepine
B. haloperidol
C. lithium
D. methylphenidate
E. sertraline
Bottom of Form

Explanation:
The correct answer is E. The patient meets the criteria for a diagnosis of panic disorder. The first line in treatment of panic disorder is selective serotonin reuptake inhibitors (SSRIs), a group of medications including sertraline, paroxetine, fluoxetine, and citalopram. Benzodiazepines may also be used during the titration of SSRI maintenance, especially if panic attacks are disabling. Maintenance benzodiazepine treatment for panic disorder, however, is discouraged due to risks of abuse, dependence, and potentially serious withdrawal complications of long-term benzodiazepine treatment.

Carbamazepine (choice A) is an anticonvulsant that is also used for mood stabilization in the treatment of bipolar disorder. It has no indication in the treatment of panic disorder.

Haloperidol (choice B) is an antipsychotic medication that is also commonly used to treat agitation or combativeness regardless of etiology. It has no indication in the treatment of panic disorder.

Lithium (choice C) is used primarily for mood stabilization in bipolar disorder and can also be used as an adjunctive treatment in unipolar depression. It has no indication in the treatment of panic disorder.

Methylphenidate (choice D) is a psychostimulant that is effective in the treatment of attention deficit hyperactivity disorder and can also be used in the treatment of cachectic medically ill depressed patients (such as those with terminal cancers and HIV). It has no use in panic disorder and its stimulating qualities may actually exacerbate many anxiety disorders.

You are called to the cardiac intensive care unit to evaluate a 73-year-old patient who underwent an emergent cardiac catheterization for an acute myocardial infarction yesterday. You are told that the patient has had a urine output of only 60 cc of urine over the last 12 hours. Prior to this, his urine output had been within normal limits. His temperature is 37 C (98.6 F), blood pressure is 130/70 mm Hg, pulse is 60/min, and respirations are 14/min. He has no complaints at this time and his physical examination is unremarkable. Reviewing the medical chart, you learn that his medical history is significant for benign prostatic hypertrophy and hypertension. You catheterize his bladder and get 20 cc of dark urine, which you send for urine analysis and culture. You deliver a 500 cc normal saline fluid bolus and start intravenous fluid at 150 cc/hour. After 4 hours his urine output does not improve. Laboratory studies show:

Sodium 144 mEq/dL
Potassium 5.3 mEq/dL
Chloride 98 mEq/dL
Bicarbonate 21 mEq/dL
BUN 28 mg/dL
Creatinine 2.2 mg/dL (*Admission Creatinine- 1.3 mg/dL)

Urinalysis
Color Muddy brown
Specific gravity 1.020
Osmolality 55 mOsmol/kg
Leukocyte esterase Negative
Nitrite Negative
Protein Trace
Blood Negative
Microscopic Many muddy brown granular casts
Urine eosinophils None
The most likely cause of this patient’s new condition is

Top of Form

A. acute interstitial nephritis
B. acute tubular necrosis
C. postrenal azotemia
D. prerenal azotemia
E. rapidly progressive glomerulonephritis
Bottom of Form

Explanation:
The correct answer is B. This patient has acute tubular necrosis (ATN) and acute renal failure. Remember that when confronted with acute renal failure, the cause is divided into prerenal or “dry”, postrenal or “obstructed”, and intrarenal. The history and the urinalysis provide us with the correct diagnosis. Patients with ATN can have muddy, granular casts in their urine. They will also have an elevated BUN and creatinine, but in a ratio of less than 20:1 as is seen in patients with prerenal azotemia. The history of recent cardiac catheterization should also provide a clue since the patient received a large ionic dye load during the procedure to image his coronary vessels. Dye is a common precipitant of ATN and needs to be considered in anyone who has an elevated creatinine after a procedure that involves contrast dyes.

Acute interstitial nephritis (AIN) (choice A) is another cause of acute renal failure. It is associated with a transient maculopapular rash, fevers, eosinophiluria, hematuria, and white cell casts. Drugs account for most cases of AIN.

Postrenal azotemia (choice C) would also be reasonable in our patient since he has a history for BPH (which is the most common cause of obstruction). If a Foley catheter is placed and a large amount of urine is found in the bladder, obstruction is a likely cause of poor urine output. Since this patient had a virtually empty bladder, it would be unlikely that he has a postrenal cause of his renal failure.

Prerenal azotemia (choice D) should be your first thought when evaluating a patient with dressed urine output because it is the most common cause of acute renal failure. This patient was given a fluid challenge and still did not increase his urine output. Therefore, prerenal azotemia should be lower on the differential. Other clues that this patient is not prerenal is that his BUN to creatine ratio is less than 20:1 and his urinalysis revealed granular casts.

Rapidly progressive glomerulonephritis (choice E) is a rare cause of acute renal failure. It is associated with hypertension and edema. Urinalysis reveals dysmorphic red blood cells, red cell casts, and mild proteinuria.

A 64-year-old woman is brought to the emergency department after her husband called 911. The couple was eating dinner together when the patient suddenly experienced left facial droop, left arm and leg weakness, and slurred speech. This occurred at approximately 6:30 p.m. The husband, who suspected that his wife had had a stroke, gave her an adult sized aspirin (325mg) before calling 911. She has a past medical history significant for diabetes mellitus and has never had surgery. She takes metformin for her diabetes and a daily multivitamin, but takes no other medications. Her family history is significant for diabetes: her mother and both of her sisters have it. She is a schoolteacher approaching retirement. She denies any smoking history. She rarely drinks alcohol and denies any drug use in the past. On arrival to the hospital, her temperature is 37.0 C (98.6 F), blood pressure is 160/85mm Hg, and pulse is 82/min. She has normal lung and heart sounds. She is awake, alert, and fully oriented. Her language is natural and fluent. She has a severe left facial droop and severe dysarthria. She has grade 0/5 strength in her left arm and in her left leg. She has intact sensation for fine touch and pinprick. Her glucose level is 95mg/dL, a platelet count of 355,000/mm3, a partial thromboplastin time of 21.0 sec, and an INR of 0.95. A CT of the head, done without contrast, is interpreted by a radiologist as being normal. You diagnose the patient with ischemic stroke and note that it is now 8:30 p.m. Your neurological consultant agrees that there are no contraindications for thrombolysis. The most appropriate treatment at this time is to

Top of Form

A. administer heparin, intravenously after a weight-appropriate loading bolus
B. administer tissue plasminogen activator, intravenously
C. give her aspirin 325mg, in addition to the dose she had been given at 6:30 p.m. by her husband
D. give her clopidogrel 75mg
E. give her no additional treatment as she has already received a dose of aspirin
Bottom of Form

Explanation:
The correct answer is B. The patient has had an acute ischemic stroke. The Federal Drug Administration (FDA) has approved IV-tPA for acute ischemic stroke provided there are no contraindications and that the drug is given within 3 hours of symptom onset. This approval, in 1996, was based on the results of the National Institutes of Neurologic Disorders and Stroke (NINDS) study which showed that patients who received IV-tPA after fulfilling certain criteria have significantly better outcomes than those who did not. Examples of contraindications include hemorrhage on CT scan; early ischemic changes on CT scan; uncontrolled hypertension; minor or resolving presenting symptoms; recent GI hemorrhage, and/or abnormal PT, PTT, platelet count, or glucose. In this case, the patient has severe stroke symptoms, has been worked up within 2 hours of symptoms onset, and has no contraindications for the use of IV-tPA.

Heparin (choice A) is incorrect. There is no known role for heparin in the treatment of acute stroke. Its use remains controversial today, and is not approved by the FDA for acute stroke.

Additional aspirin (choice C) is incorrect. There is no proven role for multiple doses of aspirin in the setting of acute stroke. The use of aspirin is based on the results of the International Stroke Trial of 1997, which showed that 300mg daily of this drug showed a modest benefit in reducing new neurologic symptoms 6 months after the first stroke.

Clopidogrel (choice D) is incorrect. There is no known role for clopidogrel in the treatment of acute stroke. Its use in prevention of stroke is based on the European Stroke Prevention Study (ESPS-II) of 1996, which showed a synergistic effect of aspirin and dipyridamole (in the same class as clopidogrel) when used together, in the prevention of stroke, but was never investigated in the setting of acute stroke.

No treatment (choice E) is incorrect. As mentioned above, aspirin is moderately effective in the prevention of new strokes, but it is not adequate in the setting of an acute stroke, especially in the case of a patient who qualifies for IV-tPA.

An 8-year-old girl is brought to the office for a well-child visit. She has no complaints at this time. The mother is very concerned about her daughter’s health because she has been reading about the threats of bioterrorism on the Internet. She is particularly worried about smallpox and anthrax. At this time you should

Top of Form

A. administer a vaccine for anthrax
B. explain that while there is no routinely recommended prophylaxis for smallpox or anthrax, there are treatment options available if she acquires either of these diseases
C. prescribe ciprofloxacin as a prophylaxis for anthrax
D. reassure the mother that her daughter has received all of the recommended childhood immunizations and should be immune to the smallpox virus
E. tell the mother that you understand her concerns but that there is no prophylaxis for either smallpox or anthrax
Bottom of Form

Explanation:
The correct answer is B. Anthrax and smallpox are potential biological weapons that can be used for bioterrorism. Smallpox has been eradicated with vaccines and so the vaccine is no longer part of the routinely recommended childhood immunizations. Cidofovir is a treatment option for an established infection. Anthrax is a potentially deadly bacteria that can cause cutaneous, gastrointestinal, and respiratory diseases. A vaccine is available, but it is only routinely given to persons at a significant continuing risk (military personnel). Ciprofloxacin is used for prophylaxis and treatment for anthrax in adults and penicillin is used for children.

It is inappropriate to administer a vaccine for anthrax (choice A) because it is only routinely given to persons at a significant continuing risk. This patient does not seem to be at a “significant continuing risk” at this time.

Since there is no reason to believe that this patient is going to be exposed to anthrax, you should not prescribe ciprofloxacin as a prophylaxis for anthrax (choice C). Also, ciprofloxacin is not approved by the FDA for use in persons under 18 years of age. It may be indicated, however, for an established serious or life-threatening infection.

Since the smallpox vaccine is not routinely given as part of the recommended childhood immunizations in the United States, it is incorrect to reassure the mother that her daughter has received all of the recommended childhood immunizations and should be immune to the smallpox virus (choice D).

It is incorrect to tell the mother that you understand her concerns but that there is no prophylaxis for either smallpox or anthrax (choice E) because as stated above, there is prophylaxis for anthrax and there is an effective vaccine for smallpox, but neither of these are indicated for this patient. According to the Centers of Disease Control in March 2001, pre-exposure vaccination against smallpox is not recommended for any group other than laboratory or medical personnel working with non-highly attenuated orthopoxviruses.

You are notified that one of your patients, a 3 -year-old previously healthy girl, is admitted to the hospital because of 6-day history of fever, irritability, and erythema of the hands and feet. Her physical examination on admission showed a temperature of 39.7 C (103.4 F), bilateral conjunctival injection, an enlarged left-sided cervical lymph node (2.0 cm), fissured lips, a red tongue with red papillae, pharyngeal hyperemia, erythematous and edematous palms and soles, and a confluent, blanching erythematous rash on the trunk. The mother told the emergency department physician that she had been giving her daughter aspirin for the past week to reduce her fever. In the emergency department, intravenous fluids were started, the aspirin therapy was continued, and laboratory studies were ordered. These laboratory studies just returned and show an erythrocyte sedimentation rate of 28mm/h and a platelet count of 490,000/mm3. The patient is extremely uncomfortable and now shows desquamation of the fingers and toes. The mother is very concerned about her daughter’s condition. At this time the most correct statement about her condition is:
Top of Form

A. Corticosteroids are necessary to decrease the risk of aneurysms within 10 days of the onset of fever

B. Influenza vaccination is necessary if this patient requires long-term salicylate therapy

C. MMR vaccination should be given within a month if this patient receives intravenous gamma globulin

D. Nasopharyngeal cultures will help to establish the diagnosis

E. There is a 40% risk of death associated with her disease

Bottom of Form

Explanation:

The correct answer is B. This patient most likely has Kawasaki disease, which is treated with aspirin and intravenous gamma globulin. The disease is characterized by a high fever for longer than 5 days, bilateral conjunctival injection, fissured lips, a “strawberry tongue”, mucosal change in the oral pharynx, erythematous and edematous palms and soles with desquamation, a polymorphous rash, cervical lymphadenopathy, an elevated erythrocyte sedimentation rate, and thrombocytosis. The most important complication is coronary artery aneurysms, which may be prevented by early treatment with aspirin and intravenous gamma globulin. An echocardiogram is necessary to evaluate cardiac involvement. The influenza vaccine is necessary if this patient requires long-term salicylate therapy because of the possible increased risk of Reye syndrome. Reye syndrome may develop in patients with influenza who receive salicylates.

Corticosteroids are necessary to decrease the risk of aneurysms within 10 days of the onset of fever (choice A) is incorrect. Initially, intravenous gamma globulin and aspirin are routinely used to decrease the risk of aneurysms in patients with Kawasaki disease. Some believe that steroids should be used later if therapy with intravenous gamma globulin and aspirin is unsuccessful. But steroids are not necessary to decrease the risk of aneurysms.

MMR vaccination should be given within a month if this patient receives intravenous gamma globulin (choice C) is incorrect. MMR vaccination should be given 11 months after treatment with intravenous gamma globulin. It should not be given sooner because intravenous gamma globulin interferes with the serologic response to the measles vaccine. If there is a high risk that this child will be exposed to measles, which is unlikely in the United States, immunization should be given, and if at 11 months serologic testing indicates that the earlier immunization was unsuccessful, she should be re-immunized.

It is incorrect to say that nasopharyngeal cultures will help to establish the diagnosis (choice D) because no single study can diagnose Kawasaki disease. The exact cause of this disease is unknown.

There is a 40% risk of death associated with her disease (choice E) is incorrect. In the United States, the mortality rate due to Kawasaki disease is 0.05%. Death is usually due to a myocardial infarction or ruptured coronary aneurysm.

A 71-year-old man with osteoarthritis comes to the office complaining of a painful “band-like” rash across his left chest. He denies ever having a similar rash before. He plays golf 3 times per week and takes only nonsteroidal antiinflammatory agents for pain from his arthritis. His temperature is 37.0 C (98.6 F). On his left chest, in the T5 dermatomal distribution, is a macular-papular, erythematous rash that is painful to the touch. There is mild weeping of some of the papules. The most appropriate therapy is at this time is

Top of Form

A. antibiotics
B. antifungal agents
C. corticosteroids
D. ganciclovir
E. gabapentin
Bottom of Form

Explanation:
The correct answer is D. The patient has herpes zoster, also known as shingles. The disease is a result of reactivation of latent varicella zoster virus in the dorsal root ganglia. The disease follows a dermatomal distribution and is very painful. The goals of therapy are to hasten the resolution of the symptoms and to prevent the development of postherpetic neuralgia, an often crippling neuropathic pain disorder resulting from the shingles infection. The drugs best able to accomplish both of these goals are the oral antiretroviral drugs, ganciclovir, acyclovir, and famciclovir.

Neither oral antibiotics (choice A) nor oral antifungal agents (choice B) have any role in the treatment of this disease, since it is a reactivation of a viral infection. Unless there is evidence of a superinfection with a bacterial or fungal etiology, these classes of drugs should be avoided.

The use of corticosteroids (choice C) for this disease will exacerbate the symptoms. Although dermatologists liberally utilize steroid therapy, its use in this case acts as an immunosuppressive agent and will exacerbate the primary manifestations of rash and pain.

Gabapentin (choice E) is used for the treatment of postherpetic neuralgia and other neuropathic pain syndromes. Gabapentin is structurally related to the neurotransmitter GABA (gamma-aminobutyric acid) but it does not interact with GABA receptors, as it is not converted metabolically into GABA or a GABA agonist, and it is not an inhibitor of GABA uptake or degradation. It has no utility in treatment of acute shingles.

You are seeing a 41-year-old man with alcoholic cirrhosis in your office for a follow-up visit after a recent upper endoscopy showed significant lower esophageal varices. His current medications include a multivitamin, folate, and thiamine. While he strongly denies any continued alcohol use, you are suspicious that he is still drinking. His blood pressure is 100/63 mmHg, pulse is 98/min, and respirations are 21/min. Physical examination shows a slightly protuberant abdomen. Given his varices, you are concerned about an upper gastrointestinal bleed, especially in the setting of continued alcohol use. Given this concern, the most appropriate pharmacotherapy to add to his treatment regimen is

Top of Form

A. aspirin
B. atorvastatin
C. isosorbide mononitrate
D. nadolol
E. warfarin
Bottom of Form

Explanation:
The correct answer is D. Non-selective beta antagonists such as nadolol have been shown to decrease the risk of an initial variceal bleed (through a reduction in splanchnic blood flow) in someone with esophageal varices (primary prevention).

Aspirin (choice A), an antiplatelet agent, has no role in the primary prevention of variceal bleeding.

Atorvastatin (choice B), an HMG Co-A Reductase antagonist used in the management of hyperlipidemia, has no role in the primary prevention of variceal bleeding.

Nitrates such as isosorbide mononitrate (choice C) should not be started as monotherapy for the primary prevention of variceal bleeding in cirrhotics since it has been associated with increased mortality when used alone. However, it can be used in combination with beta antagonists.

Warfarin (choice E), an oral anticoagulant against vitamin K dependent clotting factors, has no role in the primary prevention of variceal bleeding.

A 16-year-old boy comes to the emergency department complaining of severe headache, dizziness, and difficulty concentrating. The patient has been working as a painter for the summer. He had been working in a well-ventilated bedroom, but he mistakenly spilled some paint remover on himself. About 2 hours later, he began developing a severe headache. He brought the paint remover with him and you look on the label and see that methylene chloride is listed as one of the solvents. His blood pressure is 110/80 mm Hg, pulse is 65/min, and respirations are 14/min. A neurological examination is normal. His lips are very red and there is evidence of cyanosis of his fingers. Pulse oximetry, however, shows an oxygen saturation of 100% on room air. An arterial blood gas shows a carboxyhemoglobin level of 42% by co-oximetry. The most appropriate next step in management is to

Top of Form

A. administer methylene blue, intravenously
B. begin continuous positive airway pressure ventilation
C. give him hyperbaric oxygen therapy immediately
D. give him 100% supplemental oxygen by non-rebreather mask
E. intubate him and provide pressure support ventilation
Bottom of Form

Explanation:
The correct answer is C. This is a case of severe carbon monoxide poisoning from methylene chloride exposure. Methylene chloride is a solvent found in many paint removers and is readily absorbed through exposed areas of the skin. Once in the bloodstream, methylene chloride is metabolized in the liver and one by-product of its metabolism is carbon monoxide. Carbon monoxide poisoning typically presents with vague neurological complaints and history of exposure. In severe cases, carbon monoxide poisoning can lead to seizure, coma, or death. Cherry-red lips are a classical physical finding associated with this condition. A routine pulse oximeter cannot distinguish carboxyhemoglobin from normal hemoglobin. Carboxyhemoglobin levels must be measured by co-oximetry, which can detect the spectral differences between carboxyhemoglobin and hemoglobin. Treatment consists of administering 100% supplemental oxygen. Patients with carboxyhemoglobin levels greater than 40% require hyperbaric oxygen therapy to improve tissue oxygenation and reduce carboxyhemoglobin levels.

Methylene blue (choice A) is the treatment of choice for severe methemoglobinemia. It has no use in carbon monoxide poisoning.

Continuous positive airway pressure (choice B) can improve oxygenation by recruiting alveoli and stent open airways. It is useful in pulmonary causes of hypoxia, but does not treat the underlying carboxyhemoglobinemia.

For routine cases of carbon monoxide poisoning, 100% supplemental oxygen (choice D) is the treatment of choice. However, this patient has very high carboxyhemoglobin levels and requires hyperbaric oxygen treatment.

The patient is able to maintain his airway and breathe on his own. If there are more severe complications of carbon monoxide poisoning like seizure or coma, intubation (choice E) may be required.

A previously healthy 21-year-old woman comes to the local college clinic because of a headache and low-grade fevers. She is sent home with acetaminophen and advised to return if she does not improve. Approximately three hours later her roommate calls 911 reporting that her friend is unconscious and not arousable. On arrival the paramedics find a lethargic, febrile female lying on the floor and unresponsive. The patient is stabilized and she is rushed emergently to the local hospital where an abdominal CT scan shows bilateral adrenal hemorrhages. The patient is transported to the intensive care unit where a pulmonary artery catheter is inserted via a right internal jugular vein. Her temperature is 39.3 C (102.8 F), blood pressure is 85/40 mm Hg, and pulse is 140/min. An electrocardiogram demonstrates sinus tachycardia. Her extremities are warm to the touch and appear pink. A cardiac output (CO), systemic vascular resistance (SVR), pulmonary capillary wedge pressure (PCWP) and central venous pressure (CVP) are obtained. The data acquired from her PA-line is most likely to be
Top of Form

A. CO 5.0 L/min, SVR 800 dynes-sec/cm5, PCWP 12 mm Hg, CVP 4 mm Hg

B. CO 9.5 L/min, SVR 210 dynes-sec/cm5, PCWP 8 mm Hg, CVP 6 mm Hg

C. CO 2.3 L/min, SVR 1500 dynes-sec/cm5, PCWP 30 mm Hg, CVP 10 mm Hg

D. CO 3.0 L/min, SVR 1500 dynes-sec/cm5, PCWP 8 mm Hg, CVP 1 mm Hg

E. CO 9.5 L/min, SVR 1210 dynes-sec/cm5, PCWP 12 mm Hg, CVP 6 mm Hg

Bottom of Form

Explanation:

The correct answer is B. The patient likely has septic shock that is characterized by high cardiac output, low systemic resistance, and a relatively normal contractile function (PCWP) and filling pressures (CVP).

CO 5.0 L/min, SVR 800 dynes-sec/cm5, PCWP 12 mm Hg, CVP 4 mm Hg (choice A) is a profile of a normal, healthy person.

CO 2.3 L/min, SVR 1500 dynes-sec/cm5, PCWP 30 mm Hg, CVP 10 mm Hg (choice C) is a profile of cardiogenic shock. This patient has a depressed cardiac output, a markedly elevated PCWP reflecting failure, and an appropriately high peripheral vasoconstrictive response to maintain blood pressure.

CO 3.0 L/min, SVR 1500 dynes-sec/cm5, PCWP 8 mm Hg, CVP 1 mm Hg (choice D) reflects hypovolemic shock with a low output, an appropriately high peripheral resistance but very low filling pressures.

CO 9.5 L/min, SVR 1210 dynes-sec/cm5, PCWP 12 mm Hg, CVP 6 mm Hg (choice E) is representative of a healthy person undergoing vigorous exercise, with augmented cardiac output, normal systemic resistance and filling pressures.

A 29-year-old man comes to the clinic because he and his wife have “not been able to have a baby”. The patient states that he has been happily married for 4 years and he and his wife have been trying to have a child for the last 13 months. He has never fathered a child and his wife has never been pregnant. His wife has been evaluated by her physician and no abnormalities were identified. Your patient denies any history of cryptorchidism, sexually transmitted diseases, urinary tract infections, genital trauma, or erectile dysfunction. He has not received any chemotherapy nor does he have any known genetic disorders. Physical examination reveals a circumcised phallus without meatal discharge. Testicles are descended bilaterally, and are normal in size and contour. There is a grade 3 varicocele on the left side. No varicocele is identified on the right. On rectal examination the prostate is normal to palpation. Serum testosterone, LH, and FSH are normal. You send the patient for semen analysis. The results are as follows
At this time you should

Top of Form

A. advise them that no intervention is indicated

B. obtain a transrectal ultrasound

C. perform a testicular biopsy

D. prescribe a testosterone patch

E. refer them for in vitro fertilization

F. refer him for a varicocelectomy (ligation of varicocele)

Bottom of Form

Explanation:

The correct answer is F. A varicocele is defined as a dilated vein or set of veins in the pampiniform plexus in the spermatic cord, and is the most common identifiable cause of male factor infertility. It is present in 15% of the total male population, but is found in approximately 40% of men with male factor infertility. A varicocelectomy (the ligation of varicoceles) improves semen quality in approximately two-thirds of men and doubles the chance of conception. Varicoceles form secondary to incompetent or absent valves in the spermatic veins. This valvular deficiency, combined with the long vertical course of the internal spermatic vein on the left side, leads to the formation of most varicoceles on the left side. A unilateral right sided varicocele suggests venous thrombosis (from a tumor) in the inferior vena cava. The effect of a varicocele on fertility has to do with the prevention of efficient blood flow out of the scrotum. There is pooling of blood in the pampiniform plexus, leading to an increase in scrotal temperature and an adverse effect on spermatogenesis. Varicoceles tend to cause a “stress pattern” on semen analysis. This is characterized by a low sperm concentration, low sperm motility, low sperm count, and low sperm morphology. The technique for varicocelectomy is varied and may be performed via an inguinal, retroperitoneal, subinguinal, laparoscopic, or embolization approach. The subinguinal approach with aid of a microscope, (microscopic varicocelectomy), is the approach with the fewest complications.

No intervention (choice A) is incorrect. Any couple who has been unable to conceive for over 1 year warrant investigation. In general, 90% of normal couples conceive within 1 year of trying. Some investigators advocate a simple, basic, cost-effective evaluation of both male and female at the time of presentation, no matter how long they have been attempting to conceive.

A transrectal ultrasound (choice B) is a valuable diagnostic tool when there is an obstructive process causing infertility. The majority of semen volume comes from the prostate, seminal vesicles, and Cowper’s gland. The majority of ejaculated sperm comes from the distal epididymis. Therefore, the normal volume of ejaculate and the fact that the patient is not azoospermic (no sperm present) makes obstruction of the vas, seminal vesicles, and ejaculatory ducts, unlikely. The patient also has a normal semen pH which is caused by the fructose within the semen. Fructose is secreted by the seminal vesicles, therefore this patient’s normal semen pH makes seminal vesicle obstruction unlikely. In this scenario, the yield of transrectal ultrasound to look for seminal vesicles or ejaculatory duct dilatation, is low.

Testicular biopsy (choice C) is indicated in azoospermic patients with normal FSH levels or in patients with abnormal hormone parameters. This patient has neither.

The patient has normal hormone parameters, and there is no evidence that increasing his testosterone (choice D) will improve his semen parameters.

Referring this couple for in vitro fertilization (choice E) is not the correct management. The male in the relationship has a potentially reversible cause of his infertility (varicocele) and having them go through the emotional and financial hardship of assisted reproduction is not recommended prior to attempting correction of other causes of infertility.

A 10- year-old girl is brought to the clinic because of a 2-day history of a sore throat and fever. The mother reports that the fever has been as high as 39 C (102.2 F) and that the child is complaining of pain on swallowing. She has had no rhinorrhea, cough, or ear pain, and no one else is ill at home. Physical examination reveals a well-appearing girl with an exudative pharyngitis and multiple 1-2-cm tender submandibular lymph nodes. There is no other adenopathy, rash, or hepatosplenomegaly present. A rapid antigen detection test for group A streptococcus done on a swab of the posterior pharynx is positive. The most appropriate action at this time is to
Top of Form

A. perform a complete blood count and monospot test

B. repeat the rapid antigen detection test

C. send a throat swab for culture

D. treat with oral amoxicillin-clavulanate

E. treat with oral penicillin

Bottom of Form

Explanation:

The correct answer is E. The positive rapid test for group A streptococcus (S. pyogenes) confirms the diagnosis of streptococcal pharyngitis, and antibiotic therapy is essential for the prevention of suppurative complications (e.g., adenitis, peritonsillar abscess) as well as rheumatic fever. Penicillin remains the treatment of choice for streptococcal pharyngitis in non-allergic patients, as treatment failures are extremely rare.

Infectious mononucleosis is another potential cause of fever, pharyngitis, and rash in children. Associated findings would be generalized lymphadenopathy and splenomegaly on physical examination. The lack of these makes the diagnosis unlikely in this case. A complete blood count and monospot (choice A) can help confirm this diagnosis when it is suspected.

The rapid antigen detection tests have excellent specificity (>95%) and thus a positive test does not need to be confirmed by repetition (choice B).

Most practitioners choose to send a swab for bacterial culture (choice C) in the setting of a negative rapid test in order to increase sensitivity because of the potentially devastating sequelae of a missed case of streptococcal pharyngitis, but this is unnecessary with a positive test.

Amoxicillin-clavulanate (choice D) is a combination of a beta-lactam antibiotic and a beta-lactamase inhibitor. It is useful in the treatment of infections due to organisms that have developed resistance via the production of a beta-lactamase enzyme. Group A streptococcus does not produce a beta-lactamase. Thus, this combination is unnecessary in this case.

A 53-year-old man comes to the office because of a 2-day history of knee pain. He says that he was relaxing with his family in the living room, and when he got up to get a beer, he realized that he was unable to bear weight on his left knee. When he pulled up his left sweatpant leg, he saw that his knee was very swollen. He is normally very active and until yesterday, had participated in a 3-times a week racquetball game with his business partner and a daily basketball game with his sons on the court in their backyard. He states that he recently returned from a 2-week business trip to Southeast Asia. He tells you that his wife went to a health spa in California while he was travelling. He drinks “a couple of beers” a few times a week, smokes about a pack of cigarettes per day, eats a “fairly healthy diet” of low-fat, high-fiber foods, and does not take any medications. He cannot recall a history of trauma, does not have any other symptoms, and has never had any similar episodes of joint pain in the past. Physical examination shows a tender, erythematous, and swollen left knee with pain on flexion, and a limited range of motion. The most appropriate question that will help to establish a diagnosis is:
Top of Form

A. “Did you have unprotected sexual intercourse on your trip to Southeast Asia?”

B. “Did you receive the Hepatitis B series of vaccinations before your trip to Southeast Asia?”

C. “Do you ever wake up in the morning with severe stiffness in your hands that lasts for several hours?”

D. “Have you experienced headaches, scalp tenderness, jaw pain, or visual changes?”

E. “Have you had any crampy abdominal pain and explosive, watery diarrhea while on your trip or since you returned?”

Bottom of Form

Explanation:

The correct answer is A. This patient most likely has acute septic arthritis, due to a gonococcal infection, most likely acquired during unprotected sexual intercourse on his business trip. The clues in the history include the onset of symptoms after he returned from his trip, and the lack of any other symptoms such as fatigue, malaise, and anorexia. He has never had any similar symptoms, and he does not have a history of trauma. He had led an active, healthy life until this trip, and so it seems that it may be related to an experience that occurred in Asia. An arthrocentesis with joint fluid analysis will most likely show Gram-negative diplococci. In addition, cultures should be taken from all potential sites such as the urethra, anus, and mouth and should be performed on a Thayer-Martin medium. The treatment is cephalosporins.

While Hepatitis B (choice B) can lead to immune complex related arthritis, without fatigue, malaise, anorexia, and a rash, this diagnosis is unlikely. Also the incubation period for Hepatitis B is usually between 1-6 months.

Morning stiffness in the hands (choice C) is associated with rheumatoid arthritis, which is systemic disease that usually does not present with acute monoarticular arthritis. It is characterized by symmetric inflammation, bony erosions, and joint deformities as a consequence of synovial inflammation. Laboratory studies often show rheumatoid factor, an elevated erythrocyte sedimentation rate, and anemia of chronic disease.

Headaches, scalp tenderness, jaw pain, and visual changes (choice D) are the symptoms of temporal arteritis, which are often associated with polymyalgia rheumatica. It typically occurs in elderly patients and is characterized by proximal muscle pain, morning stiffness, fatigue, weight loss, and a low-grade fever. These patients have an elevated erythrocyte sedimentation rate and normal muscle enzymes. A temporal artery biopsy is necessary if there are complaints of visual changes and headaches. The treatment is prednisone. It is unlikely that the patient in this case has polymyalgia rheumatica.

Crampy abdominal pain and explosive, watery diarrhea (choice E) are the symptoms of traveler’s diarrhea caused by E. coli. Acute monoarticular arthritis is not commonly associated with traveler’s diarrhea.

A 64-year-old woman is brought to the emergency department after her husband called 911. The couple was eating dinner together when the patient suddenly experienced left facial droop, left arm and leg weakness, and slurred speech. This occurred at approximately 6:30 p.m. The husband, who suspected that his wife had had a stroke, gave her an adult sized aspirin (325mg) before calling 911. She has a past medical history significant for diabetes mellitus and has never had surgery. She takes metformin for her diabetes and a daily multivitamin, but takes no other medications. Her family history is significant for diabetes: her mother and both of her sisters have it. She is a schoolteacher approaching retirement. She denies any smoking history. She rarely drinks alcohol and denies any drug use in the past. On arrival to the hospital, her temperature is 37.0 C (98.6 F), blood pressure is 160/85mm Hg, and pulse is 82/min. She has normal lung and heart sounds. She is awake, alert, and fully oriented. Her language is natural and fluent. She has a severe left facial droop and severe dysarthria. She has grade 0/5 strength in her left arm and in her left leg. She has intact sensation for fine touch and pinprick. Her glucose level is 95mg/dL, a platelet count of 355,000/mm3, a partial thromboplastin time of 21.0 sec, and an INR of 0.95. A CT of the head, done without contrast, is interpreted by a radiologist as being normal. You diagnose the patient with ischemic stroke and note that it is now 8:30 p.m. Your neurological consultant agrees that there are no contraindications for thrombolysis. The most appropriate treatment at this time is to

Top of Form

A. administer heparin, intravenously after a weight-appropriate loading bolus
B. administer tissue plasminogen activator, intravenously
C. give her aspirin 325mg, in addition to the dose she had been given at 6:30 p.m. by her husband
D. give her clopidogrel 75mg
E. give her no additional treatment as she has already received a dose of aspirin
Bottom of Form

Explanation:
The correct answer is B. The patient has had an acute ischemic stroke. The Federal Drug Administration (FDA) has approved IV-tPA for acute ischemic stroke provided there are no contraindications and that the drug is given within 3 hours of symptom onset. This approval, in 1996, was based on the results of the National Institutes of Neurologic Disorders and Stroke (NINDS) study which showed that patients who received IV-tPA after fulfilling certain criteria have significantly better outcomes than those who did not. Examples of contraindications include hemorrhage on CT scan; early ischemic changes on CT scan; uncontrolled hypertension; minor or resolving presenting symptoms; recent GI hemorrhage, and/or abnormal PT, PTT, platelet count, or glucose. In this case, the patient has severe stroke symptoms, has been worked up within 2 hours of symptoms onset, and has no contraindications for the use of IV-tPA.

Heparin (choice A) is incorrect. There is no known role for heparin in the treatment of acute stroke. Its use remains controversial today, and is not approved by the FDA for acute stroke.

Additional aspirin (choice C) is incorrect. There is no proven role for multiple doses of aspirin in the setting of acute stroke. The use of aspirin is based on the results of the International Stroke Trial of 1997, which showed that 300mg daily of this drug showed a modest benefit in reducing new neurologic symptoms 6 months after the first stroke.

Clopidogrel (choice D) is incorrect. There is no known role for clopidogrel in the treatment of acute stroke. Its use in prevention of stroke is based on the European Stroke Prevention Study (ESPS-II) of 1996, which showed a synergistic effect of aspirin and dipyridamole (in the same class as clopidogrel) when used together, in the prevention of stroke, but was never investigated in the setting of acute stroke.

No treatment (choice E) is incorrect. As mentioned above, aspirin is moderately effective in the prevention of new strokes, but it is not adequate in the setting of an acute stroke, especially in the case of a patient who qualifies for IV-tPA.

An 8-year-old girl is brought to the office for a well-child visit. She has no complaints at this time. The mother is very concerned about her daughter’s health because she has been reading about the threats of bioterrorism on the Internet. She is particularly worried about smallpox and anthrax. At this time you should

Top of Form

A. administer a vaccine for anthrax
B. explain that while there is no routinely recommended prophylaxis for smallpox or anthrax, there are treatment options available if she acquires either of these diseases
C. prescribe ciprofloxacin as a prophylaxis for anthrax
D. reassure the mother that her daughter has received all of the recommended childhood immunizations and should be immune to the smallpox virus
E. tell the mother that you understand her concerns but that there is no prophylaxis for either smallpox or anthrax
Bottom of Form

Explanation:
The correct answer is B. Anthrax and smallpox are potential biological weapons that can be used for bioterrorism. Smallpox has been eradicated with vaccines and so the vaccine is no longer part of the routinely recommended childhood immunizations. Cidofovir is a treatment option for an established infection. Anthrax is a potentially deadly bacteria that can cause cutaneous, gastrointestinal, and respiratory diseases. A vaccine is available, but it is only routinely given to persons at a significant continuing risk (military personnel). Ciprofloxacin is used for prophylaxis and treatment for anthrax in adults and penicillin is used for children.

It is inappropriate to administer a vaccine for anthrax (choice A) because it is only routinely given to persons at a significant continuing risk. This patient does not seem to be at a “significant continuing risk” at this time.

Since there is no reason to believe that this patient is going to be exposed to anthrax, you should not prescribe ciprofloxacin as a prophylaxis for anthrax (choice C). Also, ciprofloxacin is not approved by the FDA for use in persons under 18 years of age. It may be indicated, however, for an established serious or life-threatening infection.

Since the smallpox vaccine is not routinely given as part of the recommended childhood immunizations in the United States, it is incorrect to reassure the mother that her daughter has received all of the recommended childhood immunizations and should be immune to the smallpox virus (choice D).

It is incorrect to tell the mother that you understand her concerns but that there is no prophylaxis for either smallpox or anthrax (choice E) because as stated above, there is prophylaxis for anthrax and there is an effective vaccine for smallpox, but neither of these are indicated for this patient. According to the Centers of Disease Control in March 2001, pre-exposure vaccination against smallpox is not recommended for any group other than laboratory or medical personnel working with non-highly attenuated orthopoxviruses.

You are notified that one of your patients, a 3 -year-old previously healthy girl, is admitted to the hospital because of 6-day history of fever, irritability, and erythema of the hands and feet. Her physical examination on admission showed a temperature of 39.7 C (103.4 F), bilateral conjunctival injection, an enlarged left-sided cervical lymph node (2.0 cm), fissured lips, a red tongue with red papillae, pharyngeal hyperemia, erythematous and edematous palms and soles, and a confluent, blanching erythematous rash on the trunk. The mother told the emergency department physician that she had been giving her daughter aspirin for the past week to reduce her fever. In the emergency department, intravenous fluids were started, the aspirin therapy was continued, and laboratory studies were ordered. These laboratory studies just returned and show an erythrocyte sedimentation rate of 28mm/h and a platelet count of 490,000/mm3. The patient is extremely uncomfortable and now shows desquamation of the fingers and toes. The mother is very concerned about her daughter’s condition. At this time the most correct statement about her condition is:

Top of Form

A. Corticosteroids are necessary to decrease the risk of aneurysms within 10 days of the onset of fever
B. Influenza vaccination is necessary if this patient requires long-term salicylate therapy
C. MMR vaccination should be given within a month if this patient receives intravenous gamma globulin
D. Nasopharyngeal cultures will help to establish the diagnosis
E. There is a 40% risk of death associated with her disease
Bottom of Form

Explanation:
The correct answer is B. This patient most likely has Kawasaki disease, which is treated with aspirin and intravenous gamma globulin. The disease is characterized by a high fever for longer than 5 days, bilateral conjunctival injection, fissured lips, a “strawberry tongue”, mucosal change in the oral pharynx, erythematous and edematous palms and soles with desquamation, a polymorphous rash, cervical lymphadenopathy, an elevated erythrocyte sedimentation rate, and thrombocytosis. The most important complication is coronary artery aneurysms, which may be prevented by early treatment with aspirin and intravenous gamma globulin. An echocardiogram is necessary to evaluate cardiac involvement. The influenza vaccine is necessary if this patient requires long-term salicylate therapy because of the possible increased risk of Reye syndrome. Reye syndrome may develop in patients with influenza who receive salicylates.

Corticosteroids are necessary to decrease the risk of aneurysms within 10 days of the onset of fever (choice A) is incorrect. Initially, intravenous gamma globulin and aspirin are routinely used to decrease the risk of aneurysms in patients with Kawasaki disease. Some believe that steroids should be used later if therapy with intravenous gamma globulin and aspirin is unsuccessful. But steroids are not necessary to decrease the risk of aneurysms.

MMR vaccination should be given within a month if this patient receives intravenous gamma globulin (choice C) is incorrect. MMR vaccination should be given 11 months after treatment with intravenous gamma globulin. It should not be given sooner because intravenous gamma globulin interferes with the serologic response to the measles vaccine. If there is a high risk that this child will be exposed to measles, which is unlikely in the United States, immunization should be given, and if at 11 months serologic testing indicates that the earlier immunization was unsuccessful, she should be re-immunized.

It is incorrect to say that nasopharyngeal cultures will help to establish the diagnosis (choice D) because no single study can diagnose Kawasaki disease. The exact cause of this disease is unknown.

There is a 40% risk of death associated with her disease (choice E) is incorrect. In the United States, the mortality rate due to Kawasaki disease is 0.05%. Death is usually due to a myocardial infarction or ruptured coronary aneurysm.

A 53-year-old man is admitted to the hospital because of rapid onset of shortness of breath. He reports that a little less than 2 weeks ago he noticed that he was short of breath and since that time it has progressed to the point where at rest, he is barely able to breath, and he is unable to walk without “nearly passing out.” He denies chest pain, pressure, any altered mental status, cough, or fever. His past medical history is remarkable only for hypertension treated with atenolol. The patient denies any recent travel, occupational exposures, or sick contacts. On arrival to the emergency department, the patient is mildly cyanotic and breathing at 24-28/min. He is conversant and appropriate, but visibly short of breath. There are no obvious signs of accessory muscle engagement. His room air oxygen saturation is 82%. The most appropriate management of this patient at this time is to
Top of Form

A. administer heliox

B. administer high flow oxygen via non-rebreathing mask

C. administer 3 liters/min oxygen via nasal prongs

D. administer 3 liters/min oxygen via simple face mask

E. perform endotracheal intubation

Bottom of Form

Explanation:

The correct answer is B. This patient has severe hypoxia of unknown etiology. The nature of his illness and rapid course suggests a disease such as interstitial pulmonary fibrosis. Most rapid cases such as this are idiopathic (Hammond-Rich syndrome). Regardless of the cause, immediate management is the same, provide adequate oxygen to determine if the hypoxia can be corrected. High flow oxygen delivered via NRB mask offers about 82-86% inspired oxygen concentration. If the shunt fraction is less than 50%, inspired oxygen of this amount will be able to correct the hypoxia. The patient can then continue on oxygen until he can no longer protect his airway, his work of breathing becomes too great, or he begins to desaturate.

Heliox (choice A) is a mixture of helium and oxygen that is used in patients with severe bronchoconstriction. The combination gas is more laminar with its’ flow and allows better delivery of oxygen to the distal airways. It has no role in the correction of hypoxia since it is a low oxygen concentration mixture.

Oxygen via nasal prongs (choice C) is inadequate for this patient. 3 L/min offered in this manner is essentially 26-28% inspired oxygen concentration. With this marginal escalation over ambient tensions, the patient will improve minimally, or more likely, not at all.

In order to use a simple face mask for oxygen delivery (choice D) the flows need to be greater than 6 L/min in order to effectively evacuate the expired carbon dioxide from the mask and prevent rebreathing.

There is no indication to place an endotracheal tube at this time (choice E). Although the patient is exerting tremendous effort to breathe, he is not in distress, has no accessory muscle use, and is not discoordinate. The first attempt at management should be to determine if oxygen, delivered via external devices, can augment his oxygenation. If this is successful, his respiratory rate will decline and his work of breathing will decrease substantially.

An 18-year-old man comes to the clinic complaining of heaviness in his left testicle. He noticed this for the first time 3 weeks ago after “pulling his groin” in a high school football game. The groin pull has improved but the discomfort in the testicle has not. He also states that he has noticed the left testicle is larger than the right testicle. His pain is non-radiating, dull in character, and not associated with any dysuria or discharge. He admits to an episode of unprotected intercourse with a new partner approximately 1 month ago. There is no weight loss, fever, cough, or headaches. Physical examination is significant for a left testicle that is non-tender, hard, increased in size as compared to the right, irregular in contour, and without transillumination. There is no inguinal adenopathy. The right testicle is normal in size and shape. No discharge is expressed per urethra. Urinalysis and urine culture are negative. Beta-human chorionic gonadotropin (bHCG) level and alpha-fetoprotein (AFP) levels are normal. The next most appropriate management for this condition is
Top of Form

A. to administer chemotherapy

B. external beam radiation therapy to scrotum

C. external beam radiation therapy to retroperitoneum

D. left radical orchiectomy via a scrotal incision

E. left radical orchiectomy via an inguinal incision

F. open testicular biopsy

G. prescribe antibiotics

H. recommend scrotal support, antiinflammatory drugs, and reevaluation in 2 weeks

I. schedule incision and drainage of scrotal abscess

Bottom of Form

Explanation:

The correct answer is E. This patient has testicular cancer until proven otherwise. The most common symptom of testicular cancer is painless enlargement of the testis. Patients frequently complain of a sensation of testicular heaviness. 10% of patients will present with acute testicular pain as a result of intratesticular hemorrhage or infarction and 10% of patients will present with symptoms related to metastatic disease (back pain from retroperitoneal metastases involving nerve roots, cough or dyspnea from pulmonary metastases, etc). And 10% of patients are asymptomatic at presentation and the mass may be picked up incidentally following trauma or by the patient’s sexual partner. An incorrect diagnosis is made at the initial examination in up to 25% of patients with a testicular tumor and may result in delay in treatment or surgery via a scrotal approach. It is important not to violate the scrotal skin in the presence of testicular cancer. The lymphatic drainage of the testis is the retroperitoneum, while the scrotal skin drains via the inguinal nodes. Violating the scrotal skin has the potential to extend the spread of metastases outside the normal anatomic path. This patient’s symptoms and physical findings are classic for testicular carcinoma. While bHCG and AFP may be elevated in a variety of testicular cancers, they do not have to be. The normal levels in this patient should not discourage one from making the proper diagnosis. Epididymitis or epididymoorchitis are the most common misdiagnoses in patients with testis cancer. These patients typically have an enlarged tender epididymis that is separable from testicle. In advanced stages the inflammation may spread to the testis and enlarge the testicle as well. There is usually an acute onset of symptoms associated with fever, urethral discharge, and irritative voiding symptoms. Hydrocele is also a common misdiagnosis. Transillumination of the scrotum may reveal a translucent, fluid-filled hydrocele versus a solid tumor. Since approximately 5-10% of testicular tumors may be associated with hydroceles, if there is any question, then a scrotal ultrasound is mandatory.

Further therapy of testicular cancer depends on the type of tumor and clinical stage. Chemotherapy (choice A) and retroperitoneal external beam radiation therapy (choice C) both play a role in the adjuvant treatment of testicular cancer. However, neither is appropriate management prior to radical orchiectomy.

Radiation therapy to the scrotum plays no role in the treatment of testicular cancer (choice B).

Initial treatment for all testicular cancer or presumed cancer is inguinal exploration and high ligation of the spermatic cord with removal of the testicle and spermatic cord (radical orchiectomy). As previously stated, scrotal approaches (choice D) and open testicular biopsy (choice F) should be avoided so as not to disrupt the lymphatic drainage system.

Antibiotics (choice G), scrotal support, and antiinflammatories (choice H) are all appropriate therapy for patients with epididymitis or epididymoorchitis. As previously explained, this patient does not have these diagnoses and providing this form of treatment will only delay appropriate care.

Drainage of an abscess (choice I) is appropriate for any patient who has this diagnosis. Patients with scrotal abscess have swelling, tenderness, erythema, fluctuance, and possibly fever. This patient does not have any of these symptoms, making this diagnosis extremely unlikely.

You are seeing a 23-year-old woman in your office for a follow up visit. She had presented for an initial visit a month ago complaining of swelling in her fingers. Today, she continues to describe edema and erythema of her metacarpophalangeal joints. She has also had some edema of her left elbow. She explains that her morning stiffness is lasting more than 1 hour. She is currently on no medications and has no allergies to medications. Her mother and maternal aunt have a history of severe rheumatoid arthritis. Her temperature is 37.2 C (99.0 F), blood pressure is 123/65 mm Hg, pulse is 76/min, and respirations are 18/min. She has edema and erythema of the metacarpophalangeal joints of both hands. The remainder of her joint examination is unremarkable. Her breath sounds are clear, and her cardiac rhythm is regular. The results of routine laboratory studies sent during her previous visit are consistent with a diagnosis of rheumatoid arthritis. The most appropriate pharmacologic intervention at this time to alter the course of her disease is
Top of Form

A. aspirin

B. 5-fluorouracil

C. methotrexate

D. penicillamine

E. sertraline

Bottom of Form

Explanation:

The correct answer is C. It is reasonable to start the patient on a trial regimen of methotrexate since studies have shown that disease modifying antirheumatic drugs (DMARD) such as methotrexate have improved outcomes with rheumatoid arthritis, and may alter the course of her disease. An NSAID or COX-2 specific inhibitor should also be used.

Aspirin (choice A) is a first-time therapy that can help with the signs and symptoms of her disease but will have minimal effect on the progression of disease.

5-fluorouracil (choice B), an antineoplastic agent, has no proven benefit with rheumatoid arthritis.

Penicillamine (choice D), while being a disease modifying antirheumatic drug (DMARD), is associated with too many toxic side effects to be chosen as a first-line DMARD.

Sertraline (choice E), a selective seratonin reuptake inhibitor (SSRI) used for the treatment of depression, has no role in the management of rheumatoid arthritis.

You are notified that one of your patients, a 76-year-old woman with hypertension, osteoporosis, and hypothyroidism, is being discharged from the hospital where she has been treated for an acute subdural hematoma sustained after a fall 2 weeks ago. A quick review of her hospital chart reveals that she had an uneventful stay with her blood pressures ranging from 125/90 mm Hg to 140/90 mm Hg, and pulse from 60/min to 70/min. You review her current medications and note that she is taking hydrochlorothiazide, alendronate sodium tablets, and thyroxine. You schedule a visit with the patient and her son to establish a discharge plan. She tells you that she lives alone and is planning on going back to her own apartment, not to “nursing home hell.” Her son says that he will make sure that she has some type of part-time home care. The most appropriate intervention to prevent another fall is to
Top of Form

A. discontinue her medications

B. keep phones at floor level

C. recommend a daytime home health care worker

D. schedule a home safety evaluation

E. tell her to walk around in stockings or socks at home

Bottom of Form

Explanation:

The correct answer is D. Falls are one of the most important problems in the elderly population. Approximately one third of individuals over the age of 65 fall annually. Falls are the most important cause of injury in this age group. There are many factors that usually lead to a fall, both intrinsic and extrinsic, and include medications, a medical illness, foot problems, and a “hazardous” home environment. After an acute medical illness is ruled out or treated, targeted interventions must occur. Targeted interventions include a full medication review with the elimination of unnecessary drugs, physical therapy, and a home safety evaluation. Warped floorboards, inadequate lighting, and throw rugs may all contribute to an unsafe home environment.

Discontinuing her medications (choice A) is inappropriate as she has a specific need for each of these agents. It is possible to switch some of her medications, like possibly try a different antihypertensive agent. However, the thyroxine for hypothyroidism and the alendronate for osteoporosis, should not be discontinued.

Keeping phones at floor level (choice B) is recommended so that the patient can have easy access if a fall occurs. This way they can get help as soon as possible. However, this does not prevent a fall.

A daytime home health care worker (choice C) may be useful when they are there, but she will be at greatest risk for a fall at night. Also, the worker cannot keep an eye on her every second. Therefore, a home safety assessment is a better answer.

Telling her to always walk around in stockings or socks at home (choice E) is inappropriate because they are slippery and can increase the risk of a fall. Your goal is to prevent another fall, not to cause one.

A 56-year-old man comes to the emergency department with severe right flank pain for the past 5 days. He says that the pain started after he returned from a long hiking trip in the Grand Canyon, and despite taking some ibuprofen, it has not improved. Infact, he came to the hospital today because the pain has increased and he now has new onset of nausea and chills. He tells you that he had similar pain several years ago that was diagnosed as uric acid stones. He was treated conservatively and eventually passed all the stones spontaneously. His temperature is 38.1 C (100.6 F), blood pressure is 130/80 mm Hg, and pulse is 115/min. On examination, the patient is unable to lie still because of the pain and has significant right costovertebral tenderness radiating to his right testicle. Leukocyte count is 16,000/mm3 and his creatinine is 2.1 mg/dL. The most appropriate study after starting the patient on intravenous hydration and antibiotics is
Top of Form

A. a CT scan of the abdomen and pelvis

B. a DMSA renal scan

C. an intravenous pyelogram

D. a radiograph of the kidneys, ureters, and bladder

E. a renal ultrasound

Bottom of Form

Explanation:

The correct answer is A. The patient’s symptoms are consistent with an infected and obstructing ureteral stone, especially in light of his past stone history and probable dehydration from his hiking trip. Colicky flank pain radiating to the groin typically marks the presence of a midureteral obstruction. An obstruction in the presence of an infection is a medical emergency because of the very high possibility of septic shock, therefore a quick diagnosis is needed. The best imaging study for nephrolithiasis is a non-contrast CT scan which can visualize all types of stones and then accurately locate them. It can also tell us whether the patient has hydronephrosis, hence, disclosing whether the patient has an obstruction. Once the diagnosis is made, a urologist will most likely need to put in a ureteral stent to correct the obstruction.

A DMSA renal scan (choice B) is a radionucleotide study that evaluates renal function and does not give much information about a renal obstruction. It also takes 4-24 hours to obtain optimal images.

An IVP (choice C) is incorrect because of the patient’s renal insufficiency at the time of presentation. The elevated creatinine is a contraindication to IV contrast. In any other cases of noninfected nephrolithiasis with a normal creatinine, an IVP would be appropriate.

A KUB (choice D) would not be able to demonstrate the presence of a renal obstruction. It can be helpful in verifying the presence of kidney stones. However, uric acid stones are radiolucent and would not be seen on a plain film.

A renal ultrasound (choice E) is incorrect because an ultrasound cannot visualize the presence of ureteral stones. It can tell us whether the patient has hydronephrosis, but nothing more.

A 71-year-old retired police officer comes to the clinic for his annual physical examination. He has no complaints, but you notice that he is not his usual self. He appears distraught and distant, speaks in a low voice, and avoids eye contact. He seems to have lost some weight, and admits to not caring about his meals. The physical examination and routine laboratory tests are unremarkable. On further questioning the patient shares that his wife had died several months ago. He lives alone, has no children, and misses her a lot. You ask him about his daily activities. It seems that he stays to himself and rarely sees his fellow officers or friends any more. The most important next step is evaluation for
Top of Form

A. Alzheimer’s dementia

B. feelings of guilt

C. religious preference

D. suicidal ideation

E. weight loss

Bottom of Form

Explanation:

The correct answer is D. It is important to ask this patient openly about suicidal ideation, plans, or an existing wish not to live anymore. Apart from anhedonia and other signs of depression or complicated grief, he seems to have all the risk factors for suicide present. These risk factors include being male, older, retired, widowed, having potential access to weapons since he was a police officer, and having no close social support.

Alzheimer’s dementia (choice A) in an elderly patient who seems to have some cognitive difficulties, should be considered. However, in this case, pseudodementia secondary to depression may be present. The imminent risk of a possible suicide attempt is, in this case, more important to assess. If that is ruled out, other tests and steps can be taken to rule out other medical conditions.

Guilt feelings (choice B) may be a part of the depressive syndrome or grief, if a patient feels that he should have been the one that deserved to die. Guilt feelings can sometimes also lead to suicidal ideation. However, it is important to ask about suicidal ideation independently before continuing further assessment.

Religious preference (choice C) can be explored in light of the possible suicidal ideation. If a patient is practicing his religion, even though suicidal ideation may be present, the fact that most religious beliefs stand against self-harm, may be helpful in preventing suicide. However, exploring this comes only after the presence of suicidal ideation is established.

Weight loss (choice E) in light of other normal findings is not the most urgent concern. It should be considered as a part of the depressive syndrome and monitored.

A 34-year-old woman comes to the office for a follow-up examination after passing a kidney stone in the hospital last week. You were away on vacation and so your partner was involved in her in-patient treatment. The patient tells you that your partner did not tell her anything about her condition and always seemed “as if he was late for his golf tee-off time.” Passing the stone was “more painful than the vaginal delivery of all 3 children combined” and so she wants to make sure that she never has one again. She has no chronic medical conditions, never had surgery, and takes no medications. Her father and brother both suffer from nephrolithiasis. You go over to the computer to check if the laboratory report on the composition of her kidney stone is complete. You see that the stone was composed of calcium and that she had a 24-hour urine collection done in the hospital that showed 295 mg of calcium and 15 mg of oxalate. Her serum calcium level is 8.5 mg/dL. The most appropriate course of action is to
Top of Form

A. prescribe allopurinol, orally

B. prescribe cholestyramine, orally, and a low-fat diet

C. prescribe hydrochlorothiazide, orally

D. recommend a high fluid intake, but no medications or supplements

E. recommend megadoses of vitamin C

Bottom of Form

Explanation:

The correct answer is C. This patient most likely has idiopathic hypercalcuria, which is characterized by calcium kidney stones, normocalcemia, and unexplained hypercalcuria. It is believed to be hereditary and is more common in men. Hydrochlorothiazide has been shown to decrease urinary calcium levels and therefore decrease the rate of calcium stone formation.

Allopurinol (choice A) is a xanthine oxidase inhibitor that is used in patients with kidney stones composed of uric acid. It decreases serum and urinary uric acid levels.

Cholestyramine (choice B), an oxalate binding resin, combined with a low-fat diet, is used in patients with intestinal hyperoxaluria for the prevention of calcium stones. These patients have a 24-hour urinary oxalate level greater than 50 mg. It occurs in patients with prior bowel surgery and chronic diseases of the gastrointestinal tract that lead to malabsorption and bacterial overgrowth.

It is inappropriate to recommend a high fluid intake, with no medications or supplements (choice D) to a patient with idiopathic hypercalcuria and kidney stones. Thiazides have been shown to reduce the rate of kidney stone formation in a patient with her condition.

Megadoses of vitamin C (choice E) are controversial and have been associated with the formation of kidney stones. Megadoses of vitamin C are not generally recommended for the prevention of calcium stones in patients with idiopathic hypercalcuria.

A 29-year-old man comes to the emergency department because of a severe “excruciating headache” for the past 12 hours. He is an associate professor at a local college and recently had a 3-day “end of semester party” for some of his favorite students at his “weekend house.” He vaguely recalls that a couple of these students complained of a headache on the last day of their stay, but the previous night was “quite a party.” He is generally healthy, takes no medications, and “drinks a few beers” each night. His temperature is 37.8 C (100.0 F), blood pressure is 150/90 mm Hg, pulse is 70/min, and respirations are 16/min. He has mild nuchal rigidity and flank tenderness. A careful cardiac examination reveals a mid-systolic click. Ophthalmologic examination shows a flat optic disc. During the examination, he turns to the side and vomits on your shoes. You decide to perform a lumbar puncture and place him in the lateral decubitus position with his thighs flexed. You introduce the spinal needle and note that the opening pressure is 230 mm Hg. As you collect the four tubes of spinal fluid you realize that the fluid is red. You run the tubes to the laboratory and put them in the centrifuge and one hour later see that the supernatant of the centrifuged cerebrospinal fluid is a yellowish color. The most likely explanation is
Top of Form

A. acute bacterial meningitis

B. acute viral meningitis

C. a brain tumor

D. a subarachnoid hemorrhage

E. a traumatic lumbar puncture

Bottom of Form

Explanation:

The correct answer is D. This patient most likely has a subarachnoid hemorrhage, presenting with a severe headache, and therefore has gross blood in the cerebrospinal fluid and xanthochromia (the yellow color) of the supernatant of the centrifuged fluid. The xanthochromia is the result of lysis of red cells and release of their intracellular contents into the cerebrospinal fluid, which usually occurs about 2-4 hours after a bleed. The elevated opening pressure can be associated with meningitis, a brain tumor, or a subarachnoid hemorrhage. It is possible that this patient has polycystic kidney disease because he has hypertension, mitral valve prolapse, a subarachnoid hemorrhage, and flank pain. An ultrasound or a CT scan is indicated, and may reveal renal cysts. A CT scan of the head will most likely show a subarachnoid hemorrhage.

Acute bacterial meningitis (choice A) and acute viral meningitis (choice B) are not usually associated with blood in the cerebrospinal fluid. It is possible to have xanthochromatic CSF when the CSF protein is elevated to 150 to 200 mg/dL, however, this patient has many other signs that indicate that he may have polycystic kidney disease and a subarachnoid hemorrhage. The headaches that the students complained of were most likely due to heavy alcohol consumption the previous night, not meningitis.

It is unlikely that this patient has a brain tumor (choice C). While a brain tumor may present with a headache and it is possible to have xanthochromatic CSF, the acute onset of the symptoms is more consistent with a subarachnoid hemorrhage. Also, the hypertension, flank pain, and mid-systolic click are suggestive of polycystic kidney disease, which is associated with berry aneurysms and subarachnoid hemorrhages.

A traumatic lumbar puncture (choice E) is when a meningeal vessel is penetrated during a procedure, and usually shows pinkish-red CSF that clears progressively from tubes one to four. A clear, not yellow, supernatant is generally found after the CSF is centrifuged. This is most likely because the red blood cells have not had a chance to lyse and release their intracellular contents into the cerebrospinal fluid

A 42-year-old man comes to the emergency department because of chest pain that is very focal, just adjacent to the sternum at approximately the junction of the left 4th rib. It does not hurt to take a deep breath, but the pain is exacerbated with certain twisting movements. He denies fever, cough, sputum production, nausea, vomiting, diaphoresis, or shortness of breath. He tells you that he thinks that “it is most likely nothing,” but he is very concerned since his father died of a myocardial infarction at age 45 and he has not seen a physician since he was 14 years old. He has no significant past medical history and is on no medications. He is afebrile with normal vital signs. Oxygen saturation is 100% on room air. He has point tenderness to light palpation over the left 4th costochondral junction. Heart is regular with no murmurs, rubs, or gallops. Lungs are clear and his abdomen is benign. Extremities are normal. Laboratory studies show a leukocyte count of 8,100 mm3, hematocrit of 34%, creatinine of 0.7 mg/dL, blood urea nitrogen of 18 mg/dL, and cardiac enzymes and troponin are normal. Electrocardiogram is normal. Chest x-ray is normal. You explain that he most likely has costochondritis and “prescribe as needed” nonsteroidal antiinflammatory medications. He tells you that he would like to follow up with you in the clinic because he is concerned about his cardiovascular risk. You should tell him that when he comes to the clinic you plan to order
Top of Form

A. no laboratory tests since his pain will most likely have resolved

B. a periodic fasting cholesterol profile and yearly blood pressure checks

C. a periodic fasting cholesterol profile, yearly blood pressure checks, and an annual chest x-ray

D. a periodic fasting cholesterol profile, yearly blood pressure checks, and a coronary angiogram

E. a periodic fasting cholesterol profile, yearly blood pressure checks, and sigmoidoscopy

Bottom of Form

Explanation:

The correct answer is B. Despite the fact that his initial clinical presentation was chest pain, he clearly had costochondritis that has now resolved. Although he may have significant coronary artery disease, (just as anyone could), there is no clinical evidence of that at this time. He does however have some cardiac risk factors, namely his sex and family history. It is the physician’s responsibility to try to modify the risk factors that can be treated such as hypercholesterolemia and high blood pressure. It is thus important to perform periodic fasting cholesterol and blood pressure checks. Infact, even if he had no family history or was a woman, it is still part of good health maintenance to monitor these. For these reasons, no laboratory tests are needed on follow up visits, since his pain will most likely have resolved (choice A) is incorrect.

Annual screening chest x-rays (choice C) for lung cancer screening or for any other reason in asymptomatic patients is not currently recommended. Early detection with low dose CT scanning in high-risk patients is currently under heavy investigation and shows much promise.

Although as mentioned earlier, anyone could have coronary artery disease, but nothing is indicating this in him right now. Thus a coronary arteriogram (choice D) would not be indicated at this time.

Sigmoidoscopy (choice E) is also used in screening and health maintenance, but not recommended until the age of 50.

A 42-year-old woman comes into the clinic complaining of intermittent easy bruising around her eyes and chest, especially after surfing and boogie boarding with her son. She recently suffered a syncopal episode and a work up, which included a stress echocardiogram, revealed a hypertrophic heart with a speckled pattern. When she was discharged from the hospital, she was given a diagnosis of congestive heart failure with a restrictive pattern. An endomyocardial biopsy is scheduled for next week. She denies any significant family history and has been healthy with the exception of easy bruising, occasionally with vomiting. Routine urinalysis from her recent admission revealed proteinuria. Upon further questioning, she has suffered carpal tunnel syndrome bilaterally and occasional numbness and tingling of her toes. The most useful study to diagnose this patient’s condition is
Top of Form

A. complete blood count

B. liver function test

C. morning cortisol level

D. protein immunoelectrophoresis

E. thyroid function tests

Bottom of Form

Explanation:

The correct answer is D. Protein immunoelectrophoresis is the correct answer, because this patient has primary systemic amyloidosis which is an uncommon disease characterized by the extracellular deposition of amyloid fibrils in multiple organs. Cutaneous manifestations of purpura and ecchymoses in the periorbital and facial areas are characteristic features in 16% of AL amyloidosus patients. Involvement of the heart, kidney, liver, thyroid, adrenal, or bone marrow can result in organ dysfunction and early death. Part of the disease process is clonal proliferation of plasma cells that produce homogeneous (monoclonal) immunoglobulin proteins (M protein). Diagnosis of M protein is accomplished with several methods including serum protein electrophoresis, immunoelectrophoresis, immunofixation, and capillary electrophoresis.

Although complete blood count (choice A), liver function tests (choice B), morning cortisol levels (choice C), and thyroid function tests (choice E) may all useful in the evaluation of a patient with systemic amyloidosis. The most useful test at this time is immunoelectrophoresis to detect the M protein.

A 1-year-old boy and 2-year old girl are brought to the office by their new adoptive parents for a routine physical examination. They were adopted just 4 days earlier and the only information that the new parents can provide is that the boy is HIV positive. The children have not received any previous medical care but both seem happy and appear well. Complete physical examinations are unremarkable. Laboratory studies show that the boy is not severely immunocompromised. The most appropriate next step in the management of these children is to administer

Top of Form

A. a bacille Calmette-Guerin vaccine to the boy only
B. a hepatitis B vaccine to the girl only
C. a measles mumps rubella vaccine to both children
D. an oral polio vaccine to both children
E. no vaccines at this time
Bottom of Form

Explanation:
The correct answer is C. Both of these children should receive a measles mumps rubella vaccine (MMR) at this time. The MMR is recommended for all children with HIV at 12 months and again as soon as 4 weeks after the first dose, except when they are severely immunocompromised. All other children in the household should also be given the MMR vaccine.

A bacille Calmette-Guerin (BCG) vaccine to the boy only (choice A) is incorrect because this vaccine, which is used to prevent tuberculosis, is not recommended in the United States.

A hepatitis B vaccine to the girl only (choice B) is incorrect because both children should be given this vaccine. It is recommended that children with HIV should be given the hepatitis B vaccine as soon as possible to prevent infection.

An oral polio vaccine to both children (choice D) is incorrect because neither of these children should be given the live virus vaccine. The inactivated polio virus vaccine (IPV) should be given to both of these children.

No vaccines at this time (choice E) is incorrect because these children require vaccines. The vaccines that should be given to both children are DTaP, IPV, H.flu type b, hepatitis B vaccine, MMR, and the influenza vaccine. The HIV-positive child should also be given the pneumococcal vaccine. The HIV-negative child should be given the varicella vaccine and it should be considered for the HIV-positive child as well.

A 47-year-old man with a long history of alcohol abuse comes to the emergency department after “vomiting blood.” He denies any abdominal pain, chest pain, shortness of breath, weakness, or numbness but does feel lightheaded when he stands and noticed some dark stool yesterday. He drank “some whiskey” last night but denies any smoking or injection drug use. He has no prior history of gastrointestinal bleeding or known liver disease. He occasionally takes aspirin for his hangovers. His temperature is 37.0 C (98.6 F), blood pressure is 128/72 mm Hg, and pulse is 98/min lying down. Standing, his blood pressure is 98/61 mm Hg, pulse is 112/min, and respirations are 16/min. Physical examination shows 1+ pitting edema in the lower extremities and black, guaiac-positive stool. A chest and abdominal x-ray are normal. An electrocardiogram shows sinus tachycardia at 98/min. Laboratory studies show:
The most appropriate next step in the management of this patient is to

Top of Form

A. administer intravenous cimetidine

B. begin intravenous fluid resuscitation

C. order a CT scan of the abdomen

D. perform nasogastric lavage

E. perform an upper endoscopy

Bottom of Form

Explanation:

The correct answer is B. In evaluating and managing a patient with a GI bleed the most important first step is to stabilize their vital signs. By history and exam this patient has evidence for a significant upper GI bleed and he is orthostatic. It is therefore imperative to obtain intravenous access and to start fluid resuscitation preferably with normal saline immediately.

In the management of patients with acute upper GI bleeding intravenous H2 blockers (choice A) are usually prescribed even though the evidence for this common practice is weak. Nevertheless, in our patient the most pressing issue is to correct his intravascular volume depletion as soon as possible.

In evaluating patients with upper GI bleeds it is usually not necessary to obtain an abdominal CT scan (choice C) . In this patient with possible cirrhosis secondary to his alcohol abuse it might be useful at some point to image his liver with a CT scan or ultrasound when he stabilizes, but it is certainly not the first step.

Likewise, nasogastric lavage (choice D) is commonly performed in the emergency department on patients presenting with upper GI bleeding, but as mentioned previously the first priority should be to hydrate the patient.

Upper endoscopy (choice E) should be performed on all patients who present with a significant upper GI bleed. Endoscopy is the gold standard for the diagnosis of the particular lesion that caused the bleeding and can also provide important prognostic information on the chance for rebleeding. Therapeutic endoscopy can also help stop the bleeding with various techniques. The timing of the endoscopy is also somewhat controversial but all authorities agree that the first step should be to fluid resuscitate the patient.

A 64-year-old woman with a diagnosis of chronic paranoid schizophrenia has moved recently to town to live with her sister. She comes to the clinic for the first time, since she needs to have a new doctor and needs her medication. She has been healthy most of her life, and except for left hip replacement surgery, has had no other interventions or treatments. She has been maintained well for years on thioridazine. She has not been in the hospital for the past 15 years. She hears voices occasionally, but has developed strategies to distract herself. She shares that she has not really had any other problems. She goes to church, walks her sister’s dog and watches TV. She denies any current complaints. Her physical examination is unremarkable. Given this patient’s history of treatment and current age, the effect of thioridazine that she should be evaluated for is
Top of Form

A. cataracts

B. diarrhea

C. hypersalivation

D. hypertension

E. prolongation of the Q-T interval

Bottom of Form

Explanation:

The correct answer is E. Thioridazine belongs to the phenothiazine group of antipsychotics that can cause prolongation of Q-T interval, inversion of the T wave, and sometimes a bifid T or U wave. Changes are reversible. It has been noted that several sudden deaths occurred secondary to cardiac arrests in patients treated with thioridazine. The use of EKG periodically is thus advised.

Cataracts (choice A) have been described in beagle dogs treated with quetiapine. Thioridazine can cause pigmentary retinopathy that is dose dependent. Because of its anticholinergic effects, it may precipitate a glaucoma attack.

Diarrhea (choice B) is not a usual side effect of thioridazine. On the contrary, it has anticholinergic properties, thus causing dry mouth, constipation, and paralytic ileus.

Hypersalivation (choice C) is a side effect of clozapine. Thioridazine, because of its anticholinergic effects, causes dryness of the mouth and other mucous membranes.

Hypertension (choice D) is not a side effect of thioridazine. All phenothiazines tend to cause hypotension, secondary to the action on alpha-adrenergic receptors.

An 18-year-old man with type I diabetes mellitus is brought to the emergency department by a friend after being found comatose. There is a known history of noncompliance with medications, however, there is no known history of drug use. Vital signs are: temperature 37 C (98.6 F), blood pressure 80/65 mm Hg, pulse 110/min, and respirations 17/min. Oxygen saturation obtained while the patient is receiving supplemental oxygen of 2 L/min via nasal cannula is 98%. The patient is comatose and is taking rapid, shallow breaths. Deep tendon reflexes are hypoactive. An intravenous line has been placed in the field. A fingerstick glucose is 430 mg/dL. An arterial blood gas, basic chemistry panel, and toxicology screen has been sent to the laboratory. The next step in the management of this patient is
Top of Form

A. a chest x-ray

B. an endotracheal intubation

C. an intravenous fluid replacement with insulin

D. methadone

E. a pulmonary artery catheter insertion

Bottom of Form

Explanation:

The correct answer is C. This patient is suffering from diabetic ketoacidosis (DKA) caused by a severe deficiency of insulin. Clinical symptoms include coma, rapid and shallow breathing, high serum glucose levels, and metabolic acidosis. The immediate management of this patient includes intravenous fluid replacement and insulin infusion. When laboratory results return, electrolyte imbalances must also be corrected.

A chest x-ray (choice A) would be complementary to a complete the evaluation of any comatose patient. In this patient with a picture of diabetic ketoacidosis, a chest x-ray would be a secondary concern. The first priority is intravenous fluid replacement and insulin therapy.

Endotracheal intubation (choice B) is not necessary at this point as the patient has a normal oxygen saturation. Adequacy of respiration will need to be reassessed when the arterial blood gas results are available. The first priority is intravenous fluid replacement and insulin therapy.

Methadone (choice D) is used to treat heroin dependency.

A pulmonary artery catheter (choice E) is not yet necessary as the patient is at the present time hemodynamically stable. The first priority is intravenous fluid replacement and insulin therapy.

A 54-year-old man presents to his primary care physician’s office over a concern regarding prostate cancer. The patient has no history of the disease, but his father died of prostate cancer at the age of 61 and the patient was told that he has an increased risk for developing the cancer. The patient reports that he has had digital rectal examinations each year, but that he would like to be “screened” for prostate cancer. He has no other medical history and takes only a low-dose aspirin daily. He denies smoking and illicit substance abuse and admits to drinking alcohol socially.The most appropriate response to this patient is:
Top of Form

A. “Normal digital rectal examinations exclude any likelihood of prostate cancer.”

B. “PSA blood tests are available.”

C. “PSA blood tests are available but only for patients with known cancer.”

D. “PSA urine tests are available for all men over the age of 50.”

E. “There is no effective screening test for prostate cancer.”

Bottom of Form

Explanation:

The correct answer is B. Men have between a 15-20% lifetime risk of developing prostate cancer. The PSA test was introduced in the late 1980s and quantifies a glycoprotein produced by the prostate that spills over into the blood. Although current guidelines differ by society, clinical practice and standard of care is to test high-risk men over the age of 40 for PSA levels. Any level greater than 4.0 ng/mL requires a prostate biopsy. The positive and negative predictive values of the test vary tremendously with patient population, but roughly 30% of patients with elevated PSA levels will have prostate cancer. These tests are also used to follow therapy in patients with known prostate cancer (choice C).

Although digital rectal examinations (choice A) are capable of detecting enlarged prostates and nodules, they do not effectively exclude small tumors, which fail to distort the gland or that present in areas of the gland which are not peri-urethral.

PSA tests are for detection of PSA in blood, not urine (choice D). There is no PSA present in ejaculate or in urine although ejaculation does transiently increase serum PSA levels for up to 48 hours.

Although the PSA test has variable sensitivity and specificity, it is a fairly robust test when used to screen patients at high risk or with enlarged prostate glands on physical examination. Therefore, it is an effective screening (choice E) test, but the efficacy depends very much on the population being tested and specific characteristics of the patient such as the presence or absence of benign hypertrophy.

A 4-year-old boy is admitted to the hospital with right eyelid swelling and redness. Approximately 3 days prior to admission he developed nasal discharge, fever, and then 24 hours prior, developed the right eyelid swelling and erythema. He has a history of mild asthma and his vaccinations are up to date. His temperature is 38.9 C (102 F), blood pressure 114/68 mm Hg, and pulse is 78/min. There is mild proptosis on the right. Extraocular motor examination is remarkable for difficulty moving the right medially, pupils are equally round and reactive to light. No lymphadenopathy is present. Laboratory studies show: leukocyte count 18,000mm3, hematocrit 35%, blood urea nitrogen 10 mg/dL, creatinine 0.6 mg/dL, sodium 135 mg/dL, and potassium 4.1 mg/dL. Intravenous cefuroxime therapy is initiated. The most appropriate next step in management is to
Top of Form

A. administer amphotericin B, intravenously

B. discharge him after 24 hours of antibiotics

C. order a CT scan of the orbits with intravenous contrast

D. prepare him for surgical exploration of the right orbit

E. prepare him for surgical exploration of the sinuses

Bottom of Form

Explanation:

The correct answer is C. This patient has a cellulitis of the right eyelid. One of the most important things to find out in order to guide therapy is whether the infection is preseptal or postseptal. The orbital septum can be thought of as running along the anterior aspect of the bones of the orbit separating the superficial tissues from the orbital contents themselves. Some signs of postseptal infection that this patient has are ophthalmoplegia and proptosis. The risk of visual loss and spread to deeper structures such as the cavernous sinus are why this diagnosis is important to make, as opposed to a superficial orbital cellulitis, which is fairly easily treated. It is also important to look for any abscesses that might need to be drained. CT scanning can help to elucidate many of these.

Amphotericin B (choice A) is an important tool in the treatment of fungal infections such as mucormycosis. Infection with this fungus is extremely deadly and should be considered in the differential diagnosis of any patient with a severe sinus infection. There is often evidence of devitalized tissue and patients often have some sort of relative immune suppressive disease such as diabetes or chronic renal insufficiency. There is nothing in this case to suggest that amphotericin should be added at this point.

Discharge of the patient should be only after adequate treatment, which may take more than 24 hours (choice B). Intravenous antibiotics can be discontinued and switched to oral after the patient has been stable and afebrile for 24 hours.

Surgical exploration of the sinuses or orbit (choice D and E) should not be performed prior to a CT scan. If an abscess or other signs of more severe infection are seen on the scan, then surgical debridement may be warranted.

A 43-year-old female former nurse comes to the clinic complaining of a rash around a healing laceration on her left dorsal forearm. She reports that she cut her forearm with a knife while removing it from her dishwasher 5 days ago. The cut was not deep and she did not seek medical attention. She has been cleaning it with hydrogen peroxide and applying neomycin ointment, followed by wrapping it in a bandage twice a day. Yesterday evening during the dressing change she noted some pruritus and erythema in the area of the wound; this morning she was alarmed to find the area extremely itchy and with a “horrible rash.” She denies fevers or chills and states that besides the situation with her wrist, she feels well. Looking at her chart and speaking with her you find her only medication is lisinopril for hypertension. She has no other medical problems of which she is aware. Physical examination of the left forearm reveals a shallow, healing, 2.5 cm long laceration. Extending approximately 2 cm from the wound in each direction is erythema and minute vesicles filled with clear fluid. There is no lymphadenopathy and her temperature is normal. The most appropriate management is to
Top of Form

A. prescribe acyclovir ointment

B. prescribe acyclovir tablets

C. prescribe cephalexin tablets

D. prescribe hydrocortisone ointment

E. recommend bacitracin ointment

Bottom of Form

Explanation:

The correct answer is D. The appearance of the skin, the history of applying neomycin ointment, and the time course all point to an allergic contact dermatitis. Neomycin is a very common sensitizer, with as many as 5% of the population being allergic to the compound in some studies. Treatment for mild cases of allergic contact dermatitis is stopping the offending agent and applying topical steroids. This case also highlights the fact that many people wrongly assume that topical products, especially over-the-counter products, are not medicines.

Acyclovir ointment (choice A) has been shown to be marginally effective for speeding healing of herpes labialis, but has not been studied for extralabial sites. For a large area of herpes vesicles, oral acyclovir would be the preferred treatment.

Acyclovir tablets (choice B) would be reasonable if this was a herpetic outbreak. The classic appearance of herpes is grouped vesicles on an erythematous patch or edematous plaque. The localization of herpetic lesions to the forearm would be unusual, as would the subjective symptom of pruritus since herpetic lesions are usually painful. Lack of lymphadenopathy would be another factor that would militate against a diagnosis of herpes. Definitive diagnosis of herpes would be either positive culture or identifying multinucleated cells on a Tzanck prep.

Cephalexin (choice C) is a treatment for impetigo, particularly if it is widespread. Also, if you suspected the patient had cellulitis, systemic antibiotics would be indicated. Clinically cellulitis would have erythema around the wound, but fever and/or lymphadenopathy would be expected. In addition, cellulitis does not present with vesicles nor with pruritus.

Bacitracin ointment (choice E) may be a reasonable choice if you suspected impetigo, but that entity consists of erythematous plaques with a prominent scale-crust. This crust is classically described as honey-colored. Impetigo generally does not exhibit vesicles. It would also be uncommon to develop in an area being cleaned and having antibacterial ointment applied twice a day. Note, the lack of fever and the lack of lymphadenopathy are entirely consistent with impetigo. Another reason bacitracin would be a poor choice in this situation is a high percentage of patients with contact sensitivity to neomycin will cross-react to bacitracin, thereby aggravating the contact dermatitis.

A 51-year-old woman who your partner has been treating for low back pain calls the office complaining of a change in symptoms. She reports that although she has had intermittent low back pain for a number of years, in the past few weeks there has been a profound increase in her pain. She also reports that her left leg often tingles and is numb. In reviewing her records, it is noted that the patient has been seen in the office over the years for back pain and has recently been informed that she can no longer obtain narcotics. The patient works as a daycare manager and often lifts small children. She is married with a 4-year-old daughter. The most appropriate next step is to
Top of Form

A. ask the patient to come to the office for a neurological evaluation

B. order an MRI of her lumbar spine

C. prescribe additional narcotic analgesia

D. refuse to see the patient as she is malingering

E. schedule the patient for epidural steroid injections

Bottom of Form

Explanation:

The correct answer is A. This patient suffers from low back pain. This entity is one the most challenging disorders in all of medicine. It has been estimated that 70% of the population suffers from some sort of low back pain at an annual cost of between $20 and $50 billion. The etiology is often unknown in 85-90% of cases but most cases resolve with support and mild pain relief. The goals of evaluation are primarily to exclude serious underlying visceral or systemic disorders. This patient, despite being seen for back pain in the past, has new neurological symptoms which suggest an underlying etiology that is not simply musculoskeletal. She requires a full physical examination.

In the absence of a full exam, which would point to a specific diagnosis, it is not appropriate to order an MRI of her lumbar spine (choice B) without seeing the patient.

In the presence of new signs or symptoms such as the ones this patient is describing, masking the pain with narcotics (choice C) is incorrect. In addition, narcotics have been shown to be only minimally effective in managing low back pain and are not considered standard management for this condition.

Even though this patient has had a complex past history with the clinic, her new signs and symptoms point towards a specific diagnosis such as spinal stenosis or disk herniation and she requires evaluation. To consider her a malingerer (choice D) without evaluation is inappropriate.

Epidural steroid injections (choice E) is a specific therapy aimed at reducing inflammation in certain conditions such as epidural adhesions, facet degeneration, and refractory low back pain. Their efficacy is uncertain and the treatment is usually attempted only after more serous disease is excluded and the patient has failed physical therapy and oral antiinflammatory agents.

A 58-year-old alcoholic with hepatitis C cirrhosis is admitted to the hospital for management of his ascites. He has been managed as an outpatient with diuretics and oral lactulose, but over the past few weeks, he reports increasing abdominal girth, weight gain and lower extremity edema. He has been noncompliant with his low-sodium diet. His medications include furosemide, spironolactone, lactulose, ciprofloxacin, and thiamine. On physical examination, he appears grossly edematous and appropriately responsive. His lungs are clear and his heart is without extra sounds or murmurs. His abdomen is tense with a fluid wave and shifting dullness on percussion. He has numerous non-blanching telangiectasias on his torso and abdomen. His testes are small for his age and there is no asterixis. Admission laboratory studies show:
Sodium

121 mEq/L

Potassium

4.3 mEq/L

Bicarbonate

29 mEq/L

BUN

38 mg/dL

Creatinine

1.5 mg/dL

Urinalysis shows some granular casts and a urinary sodium concentration of <10 mmol/L. The most appropriate therapy is to

Top of Form

A. administer hypertonic saline

B. administer sodium chloride tablets

C. increase the dose of furosemide

D. increase the dose of spironolactone

E. salt restrict

Bottom of Form

Explanation:

The correct answer is E. True hyponatremia is always hypotonic. There are then three types of hyponatremia: hypovolemic, euvolemic, or hypervolemic. The therapy for the hyponatremia depends on both the urinary concentration of sodium as well as the volume status for each patient. Once the patient’s total body volume status has been estimated, urinary sodium usually allows focusing of the differential diagnosis. For this patient, he clearly has total body volume overload. He has cirrhosis and portal hypertension. The standard therapy for these patients is salt restriction and management of volume status with diuretics. This patient, at the core of his therapy requires salt restriction.

Although some forms of hyponatremia respond to administering hypertonic saline (choice A), knowing the pathophysiology of this patient’s hyponatremia, it can be clearly seen that this intervention will only aggravate this patient’s condition.

Giving the patient sodium chloride tablets (choice B) will clearly not be beneficial to this patient given the above explanations.

Increasing the dose of furosemide (choice C) or the dose of spironolactone (choice D) will certainly serve to diurese the patient, but in the presence of a salt load, the kidneys will respond by avid retention of salt and water and thus aggravate the patient’s condition.

A 24-year-old third-year medical student is brought to the emergency department from the operating room after she began to have trouble breathing after she put on her gown, latex gloves, and mask. This was going to be her first time “scrubbing in” to a case on the first day of her surgery rotation. Another medical student who was “scrubbed in” to the case, tells you that the she was grasping at her throat and attempting to rip off her gown. She is now on a stretcher, gasping for air. She is brought into a room and all of her clothes are taken off, revealing red hands and an eruption of well-circumscribed, erythematous, raised lesions covering her entire body. Her blood pressure is 70/50 mm Hg, pulse is 110/min, and respirations are 45/min. She has marked laryngeal edema and audible wheezes. The other student says that this is the second day of their third year, and that the only other patient contact that they have had was during the first year when they went as a group to evaluate a patient in respiratory isolation. At this time the most correct statement about her condition is:
Top of Form

A. Her rash is unrelated to her respiratory symptoms

B. Her symptoms are caused by the anxiety and stress from the first surgical experience

C. Her symptoms are due to an infectious pathogen

D. Her symptoms are the result of bioactive mediators released when exposed to an antigen

E. She should be given indomethacin immediately to reduce the respiratory symptoms

Bottom of Form

Explanation:

The correct answer is D. This patient is experiencing an anaphylactic reaction, which is a life-threatening response in a sensitized individual to a specific antigen, most likely the latex in the gloves, and is associated with the release of lipid mediators, secretory granule preformed mediators, and cytokines. The manifestations typically occur within seconds to minutes of exposure and include respiratory distress, vascular collapse, urticaria and angioedema, gastrointestinal symptoms, and shock. The reaction involves IgE-dependent activation of mast cells, basophils and the release of mediators such as histamine, cytokines, and lipid mediators. The immediate treatment involves the ABCs (intubation, oxygen, and intravenous saline), epinephrine, a vasopressor agent such as dopamine, an antihistamine such as diphenhydramine, and glucocorticoids to alleviate later recurrence of symptoms.

It is incorrect to say that the rash is unrelated to her respiratory symptoms (choice A) because they are both associated anaphylactic reactions. Anxiety and stress from the first surgical experience (choice B) may be associated with vasovagal (vasodepressor) syncope, which is the cause of the common “faint”. It frequently occurs during periods of stress or fear and often occurs in medical students during new experiences. The symptoms include hypotension, bradycardia, nausea, pallor, diaphoresis, and a sudden, transient loss of consciousness. Lying down and elevating the legs reverses the symptoms. This is very different from a life-threatening anaphylactic reaction where hypotension, tachycardia, respiratory failure, shock, and skin manifestations occur. Assuming a supine position during an anaphylactic reaction does not reverse the symptoms.

This patient is most likely experiencing an anaphylactic reaction from the latex gloves, not an infectious pathogen (choice C). Pathogens may cause septic shock by releasing toxins that then activate cytokines, platelet activating factors, arachidonic acid metabolites, and humoral defense systems. This manifests as hypotension, tachycardia, changes in mental status, and cold extremities. The laryngeal edema and specific rash that this patient has are more consistent with an anaphylactic reaction than septic shock.

She should definitely not be given indomethacin immediately to reduce the respiratory symptoms (choice E) because it is a nonsteroidal anti-inflammatory drug that may cause similar life-threatening respiratory reactions in patients with asthma. You do not know her medical history. Also, this is not part of the management of anaphylaxis, which includes epinephrine, saline, vasopressors, and intubation.

A 45-year-old homeless man is brought to an emergency department by ambulance after being found in the street obtunded, combative, and agitated. The patient is unable to provide any history in the emergency room due to his altered mental status. His temperature is 38.1 C (100.6 F), blood pressure is 200/110 mm Hg, and pulse is 110/min. While in the emergency department, the patient has a generalized tonic-clonic seizure lasting approximately 30 seconds. The patient’s presentation is most consistent with withdrawal from
Top of Form

A. alprazolam

B. caffeine

C. cocaine

D. heroin

E. tricyclic antidepressants

Bottom of Form

Explanation:

The correct answer is A. Withdrawal from a benzodiazepine, such as alprazolam, is a potentially fatal condition that can involve psychomotor agitation, frank delirium, autonomic instability, and seizures. In this patient, another consideration with a similar clinical presentation would be delirium tremens from alcohol withdrawal. Both conditions would be treated with supportive measures and acute parenteral benzodiazepine treatment.

Caffeine (choice B) has a potential withdrawal syndrome associated with irritability, possible concentration difficulties, and headache. However, even abrupt discontinuation of caffeine use is not considered to be at all dangerous.

Cocaine withdrawal (choice C) is characterized by intense feelings of dysphoria, loss of energy, increase in appetite, and intense cravings for the drug. It is not usually associated with delirium, autonomic instability, or seizures and does not carry an associated risk of fatality.

Heroin withdrawal (choice D) is characterized by piloerection, nausea, and diarrhea and can include modest increases in blood pressure and heart rate. However, it is not characterized by delirium, withdrawal seizures, or autonomic instability and is not considered to carry a high risk of fatality. Clonidine or a long-acting opiate agonists such as methadone are the treatments of choice for withdrawal from heroin.

Tricyclic antidepressants (choice E) do not have an associated withdrawal syndrome and may be discontinued abruptly, especially if treatment-emergent complications arise.

A 36-year-old woman comes to the emergency department with a 36-hour history of fever, chills, and right flank pain. She has had similar, but less severe episodes in the past which all improved with antibiotics and hydration. Her temperature is 39.1 C (102.4 F), blood pressure is 110/68 mm Hg, pulse is 115/min, and respirations are 24/min. Physical examination shows severe right sided costovertebral tenderness and mild right lower quadrant tenderness without rebound. She vomits twice during your exam. Urinalysis shows 18 red blood cells per high power field. A CT scan without contrast shows moderate right-sided hydronephrosis with perinephric stranding and a 7 mm stone in the mid right ureter. The appendix is not visualized. Laboratory studies show:
The most appropriate intervention at this time is

Top of Form

A. extra-corporeal shock wave lithotripsy of ureteral stone

B. intravenous hydration

C. intravenous hydration and antibiotics

D. intravenous hydration, antibiotics, and placement of right percutaneous nephrostomy tube

E. observation alone

F. repeat CT scan with oral contrast to evaluate for appendicitis

Bottom of Form

Explanation:

The correct answer is D. This patient is suffering from obstructive uropathy leading to pyonephrosis (distention of the renal pelvis with pus) and sepsis. Treatment of the infected, obstructed collecting system is priority to preventing further renal failure and sepsis. Relief of obstruction (decompression) can be performed with either a percutaneous nephrostomy tube or insertion of ureteral stent (the preferred modality is controversial with available data supporting either modality). Manipulation of an infected renal collecting system at the time of infection should be kept to a minimum, and treatment should be limited to procedures that allow for adequate drainage of the obstructed kidney.

ESWL (choice A) is contraindicated during infection. This therapy, which provides shock waves to renal/ureteral calcifications, would not provide relief of urinary obstruction.

Antibiotics and hydration (choices B and C) are important therapeutic modalities for this patient. However, they are not the definitive therapy and must be accompanied by decompression of the dilated kidney. Broad spectrum antibiotics should be implemented immediately and tailored according to cultures from the pus that is drained at time of nephrostomy tube placement. Without relief of obstruction, any antibiotic treatment will be incomplete.

Because this patient has had similar less severe episodes in the past, one might be tempted to observe her (choice E). However, while working in the emergency department it is important to realize that this patient exhibits none of the prerequisites that would make her eligible for observation (afebrile, no white count, no azotemia, able to tolerate oral intake and oral analgesics).

This patient is clearly very sick. While acute appendicitis (choice F) must always be included in the differential of a patient with right lower quadrant tenderness, fever, and elevated white blood cell count, this patient’s complete clinical picture is not consistent with appendicitis. Delaying definitive treatment for further radiologic studies would only prolong clinical improvement and would further endanger this patient’s health.

A 32-year-old HIV-positive intravenous drug abuser is admitted to the hospital after being found on the city sidewalk having a generalized seizure. A CT scan of the head is performed and shows several rim-enhancing lesions with minimal mass effect. You notice in his chart that he takes multiple “HIV medications” but he is unsure of the exact names and his grandmother died of a ruptured cerebral aneurysm. The most appropriate next step in management is to
Top of Form

A. get a cerebral angiogram

B. order a ventricular cerebrospinal fluid (CSF) aspiration

C. perform a lumbar puncture and include cerebrospinal fluid for Epstein Barr virus (EBV) PCR in tests ordered

D. stop all antiretroviral therapy

E. treat with intravenous acyclovir

Bottom of Form

Explanation:

The correct answer is C. The most common etiologies of rim-enhancing brain lesions in AIDS patients are primary CNS lymphoma (PCNSL) and Toxoplasma gondii infection. Other etiologies such as bacterial or fungal abscess are also possible. CSF EBV PCR test is highly sensitive and specific for PCNSL.

A cerebral angiogram (choice A) should be done if you suspect an aneurysm or vascular malformation. These are unlikely in this case.

Since there is no mass effect it is safe to do a lumbar puncture. Therefore, a ventricular cerebrospinal fluid (CSF) aspiration (choice B) is not necessary.

There is no reason to stop all antiretroviral therapy (choice D).

Intravenous acyclovir (choice E) is used to treat herpes encephalitis, which is unlikely in this case.

A 22-year-old medical student has a PPD placed at his yearly physical exam. He has no medical problems and is currently taking no medications. He returns to your office three days after his initial appointment with 8 mm of induration around his PPD. He is concerned that he is infected with tuberculosis. At this time you should

Top of Form

A. begin sputum collection for AFB staining and culture
B. explain that he needs an extended course of single drug therapy
C. explain that he needs an extended course of four drug therapy
D. explain that no intervention is indicated at this time
E. recommend a chest x-ray to determine appropriate treatment
Bottom of Form

Explanation:
The correct answer is D. When evaluating a patient for a positive tuberculin skin you must first classify which risk category the patient belongs. Patients are divided into three groups: High risk patients are considered to have a positive skin test if the area of induration is greater than or equal to 5 mm. These are people with HIV or those at high risk for HIV (i.e. IV drug users), people who have close contact with patients with active TB, or those who have had TB or evidence of prior TB on chest x-ray. Patients with induration greater than or equal to 10 mm are considered to have a positive test if they immigrated from countries with a high prevalence of TB, are known HIV negative IV drugs users, are medically underserved populations, prisoners, mental institution residents, nursing home residents, those with some chronic medical problems, or hospital workers. All other people are considered to have a positive test if the area of induration is greater than 15 mm. Our patient has an area of induration of less than 10 mm. No further work up is needed in this patient.

Collecting sputum for acid fast bacilli (AFB) (choice A) is helpful in confirming the diagnosis of suspected tuberculosis. If this patient reported typical symptoms of tuberculosis such as fevers, night sweats, weight loss, and cough a positive AFB smear or culture would help us to make the diagnosis of TB.

Extended course of single drug therapy (choice B) would be necessary if this patient had a positive skin test with no evidence of active infection. Preventative therapy has been shown to reduce the incidence of reactivated tuberculosis.

An extended course of four-drug therapy (choice C) is the treatment of choice for active tuberculosis. A typical four-drug regimen includes isoniazid, rifampin, pyrazinamide, and ethambutol or streptomycin.

A recent converter of their skin test should get a chest x-ray (choice E) to evaluate for any evidence of active disease and to have a baseline chest x-ray that could be followed in the future. Once a patient has a positive skin test, it is no longer valuable to perform further skin tests. Rather, yearly chest x-rays for evidence of reactivated TB should be done.

A 59-year-old man is admitted to the hospital for shortness of breath. The patient has a long-standing cardiac history and has suffered two non-Q wave infarctions in the past 20 months. The patient reports bright red blood in the toilet bowl during his last bowel movement. Laboratory data are remarkable for a hematocrit of 22%. Given the patient’s known coronary disease, his attending cardiologist recommends a blood transfusion. As appropriate, you order 2 units of appropriately matched red bloods cells in order to transfuse the patient to a target hematocrit above 30%. While the first unit is being administered, the patient becomes febrile and develops chest and flank pain. You are immediately summoned to his side and on arrival you note erythema around the intravenous access site and a small volume of dark colored urine in his Foley catheter bag. The remainder of the physical examination is unremarkable. The most likely diagnosis is
Top of Form

A. acute febrile hemolytic reaction

B. anaphylaxis

C. delayed hemolytic transfusion reaction

D. pulmonary embolus

E. transfusion associated lung injury

Bottom of Form

Explanation:

The correct answer is A. This patient is most likely having an acute febrile hemolytic reaction. This occurs when there is a mismatch between the patient’s and donor’s ABO types and classically presents with fever, chest/back pain, nausea, pain, and erythema around the infusion site and hemoglobinuria.

Anaphylaxis (choice B) is unlikely given that the patient has no known drug allergies. Also, there is no mention of the bronchospasm or urticaria that would be expected with this process.

A delayed hemolytic transfusion reaction (choice C) is also unlikely given the acuity of presentation. This reaction typically presents about 3-10 days following a transfusion reaction and has a mild course with fever and malaise.

A pulmonary embolus (choice D) becomes less likely given that there is no mention of dyspnea or impaired oxygenation. There also is no mention of the classic physical stigmata of a pulmonary embolus such as a loud pulmonic component of the second heart sound or pulmonic tap.

Transfusion associated lung injury (choice E) is unlikely given the acuity of symptoms. This process, which typically manifests as an acute lung injury type pathology, typically presents with a more delayed course.

A 28-year-old woman comes to the office because of a 4-day history of “itching, burning, and an awful-smelling vaginal discharge”. She says that she and her boyfriend had similar symptoms a few months ago, which resolved after treatment by his physician. Now, she believes that he is “fooling around,” because this “disease” has returned. On physical examination her vulva is erythematous and there are patches of petechiae in the upper vagina and on the cervix. There is a copious amount of yellowish-green, “frothy”, malodorous vaginal discharge. Examination of the discharge on a saline wet mount will most likely reveal
Top of Form

A. branching hyphae and spores

B. epithelial cells with clumps of bacteria and “ground-glass” cytoplasm

C. giant multinucleated cells with intranuclear inclusions

D. koilocytes

E. motile, flagellated organisms

Bottom of Form

Explanation:

The correct answer is E. This patient has the signs and symptoms most consistent with a Trichomonas vaginalis infection, which is diagnosed by finding motile, flagellated organisms on a saline wet mount smear preparation. Patients with T. vaginalis typically experience vulvar itching and burning, a “frothy” malodorous discharge, dysuria, dyspareunia, and frequency and urgency of urination. Vaginal and cervical petechiae (“strawberry cervix”) may be present. The treatment is metronidazole and it is important to treat the partner because T. vaginalis can be transmitted by sex.

Branching hyphae and spores (choice A) are associated with an infection with Candida albicans, which is characterized by intense pruritus and a thick, white (“cottage-cheese”) discharge. This patient’s discharge is not consistent with a Candida infection.

Epithelial cells with clumps of bacteria and “ground-glass” cytoplasm (choice B) are associated with an infection with Gardnerella vaginalis, which is characterized by gray-white, “fishy” discharge. Irritation of the vaginal epithelium is not usually seen. This patient’s discharge is not consistent with a Gardnerella infection.

Giant multinucleated cells with intranuclear inclusions (choice C) are associated with an infection with herpes simplex virus, which is characterized by vesicular lesions, ulcers, paraesthesia, and dysuria. The diagnosis is confirmed with viral cultures and scrapings. Giant multinucleated cells with eosinophilic intranuclear inclusions are seen when stained with Wright’s stain. A saline wet mount smear preparation is not used to diagnose herpes infections. Also, this patient’s discharge is not consistent with a herpes infection.

Koilocytes (choice D) are associated with an infection with the human papilloma virus (HPV) which is characterized by soft, fleshy lesions on the genital region (condyloma acuminata). The diagnosis is established with a biopsy of the lesions. A Pap smear may show “koilocytes”, which are cytologic changes associated with HPV. A saline wet mount smear preparation is not used to diagnose HPV. This patient’s signs and symptoms are inconsistent with HPV.

A 60-year-old man with diabetes and hypertension comes to the clinic because his wife is worried that his skin is turning yellow. The patient’s wife reports that she first noticed the skin changes about 1 month ago and now she says “even his eyes look bright yellow!” He drinks a case of beer a week and smokes 2-3 packs of cigarettes a week. He says he has been feeling well and denies abdominal pain, nausea, or vomiting. Vital signs are normal. He is a thin male and the abdominal examination is normal. Laboratory studies show:
The most appropriate test at this time is

Top of Form

A. CT of abdomen and pelvis

B. endoscopic retrograde cholangiopancreatography (ERCP)

C. mesenteric angiography

D. serum CA19-9

E. upper gastrointestinal barium study

Bottom of Form

Explanation:

The correct answer is A. The presentation of painless jaundice is highly suspicious for a pancreatic head mass and in particular adenocarcinoma of the pancreas. Adenocarcinoma of the pancreas accounts for more than 90% of pancreatic malignancies and jaundice is present in about 65% of patients. Risk factors for pancreatic adenocarcinoma include smoking and diabetes. The best initial evaluation for pancreatic masses is by CT of the abdomen and pelvis.

Endoscopic retrograde cholangiopancreatography (ERCP) (choice B) is not the best initial test to evaluate for pancreatic masses because it primarily evaluates the biliary duct system in the liver and pancreas. It will not define a mass that does not involve the biliary ducts. An ERCP may show a discrete stricture in the main pancreatic duct with proximal dilatation.

A mesenteric angiography (choice C) is not the best initial test to evaluate for pancreatic masses because it primarily evaluates the vascular structures that supply and drain the pancreas and abdominal organs. Angiography can be useful preoperatively because it may show displacement or encasement of the pancreatic or duodenal arteries by a mass. The venous phase is also useful if the superior mesenteric vein or splenic vein is occluded due to tumor extension.

Serum CA19-9 (choice D) is not the best initial test to evaluate for pancreatic adenocarcinoma. It is a tumor marker that has been associated with adenocarcinoma of the pancreas. It is not useful as a screening test, but it has a high sensitivity and specificity as a marker for recurrent disease or metastases after the primary pancreatic tumor is resected.

An upper gastrointestinal barium study (choice E) is not the best initial test to evaluate for pancreatic masses. It is a useful study to evaluate for mucosal lesions within the esophagus, stomach, and small bowel. In the setting of pancreatic malignancy or other masses, the upper gastrointestinal barium study may show a widened loop or an “inverted 3 sign” caused by abnormal indentation of the pancreas along the medial aspect of the duodenum.

A 31-year-old man undergoes an uneventful standard “open” repair of the hernia using “mesh plug” technique. As planned, he is discharged home on the same day of surgery. At his first postoperative visit, he complains of numbness over the upper aspect of his right thigh and the right side of the scrotum. Physical examination reveals an incision that is clean and dry without any evidence of infection. The right hemi-scrotum is moderately edematous and minimally tender, both testicles are normal to palpation. Motor sensation of his right lower extremity is completely intact. Sensory examination reveals decreased sensation to touch over the medial aspect of the thigh just below the inguinal ligament as well as over the right side of the scrotum. He is given instructions to observe this complaint and follow up in 2 weeks. The injured nerve is most likely the

Top of Form

A. iliohypogastric
B. ilioinguinal
C. lateral femoral cutaneous
D. obturator
E. pudendal
Bottom of Form

Explanation:
The correct answer is B. Although it is now possible to perform inguinal hernia repairs laparoscopically, the traditional approach is through an incision over the inguinal canal. At the time of surgery, the inguinal canal is opened via sharp dissection through the external oblique aponeurosis, (the anterior wall of the inguinal canal). The ilioinguinal nerve is then encountered as it runs on the anterior aspect of the spermatic cord. If not properly identified at time of surgery, it is possible to transect the ilioinguinal nerve or to “trap” it during closure. If the ilioinguinal nerve is transected on “entrapped” in closure, the patient will complain of numbness over the nerve’s distribution, (i.e. the upper medial aspect of the thigh and the anterior portion of the scrotum on the affected side).

The iliohypogastric nerve (choice A) is derived from L1 (as is the ilioinguinal nerve), and runs with the ilioinguinal nerve as they both pierce the transversus abdominis muscle near the anterior superior iliac spine, and then pass through the internal and external oblique muscles to supply the skin of the suprapubic and inguinal regions and the abdominal musculature. The iliohypogastric sends a lateral branch to the skin of the gluteal region and then continues on, to pass through the superficial inguinal ring.

The lateral femoral cutaneous nerve (choice C) originates from L2 and L3 and is a direct branch of the lumbar plexus. It enters the thigh deep to the lateral end of the inguinal ligament, near the anterior superior iliac spine and supplies the skin on the anterior and lateral aspects of the thigh.

The obturator nerve (choice D) is the nerve of the adductor muscles of the thigh. It arises from the lumbar plexus (L2, L3, L4), enters the pelvis minor, and then leaves the pelvis via the obturator foramen. The obturator nerve also sends a small cutaneous branch to the medial aspect of the mid thigh.

The pudendal nerve (choice E) arises from the sacral plexus (S2, S3, S4), and accompanies the internal pudendal artery and leaves the pelvis between the piriformis and coccygeus muscles. The nerve hooks around the sacrospinous ligament to enter the perineum through the lesser sciatic foramen to supply the muscles of the perineum, including the external anal sphincter and then ends as the dorsal nerve of the penis or clitoris. It also supplies some sensation to the external genitalia.

The iliohypogastric, lateral femoral cutaneous, obturator, and pudendal nerves are not encountered during traditional inguinal hernia repair.

A 33-year-old woman comes to the office because of generalized weakness and a “pins and needles” feeling in her lower extremities for the past 3 weeks. She states that she feels “unsteady” on her feet. She exercises daily, rarely drinks alcohol, and is a strict vegetarian. Since this is the first time you have met this woman, she tells you that she has not had any major illnesses, but has been hospitalized multiple times over the past few years for anorexia nervosa. Her temperature is 37 C (98.6 F), blood pressure is 110/70 mm Hg, pulse is 60/min, and respirations are 18/min. Examination shows weakness of the proximal and distal muscles of the lower extremities. There is impaired proprioception and vibratory sensation. Deep tendon reflexes are increased. The gait is ataxic. The most likely diagnosis is
Top of Form

A. Guillain-Barre syndrome

B. Lambert-Eaton syndrome

C. myasthenia gravis

D. polymyositis

E. subacute combined degeneration of the spinal cord

Bottom of Form

Explanation:

The correct answer is E. This patient most likely has subacute combined degeneration of the spinal cord, which is a condition caused by vitamin B12 deficiency. It is most often caused by pernicious anemia, but it may be acquired in patients with strict vegetarian diets or small bowel disease. The clinical manifestations include weakness, paraesthesias, loss of vibratory sensation, increased deep tendon reflexes, and extensor plantar responses. The gait is ataxic. Mental changes may also occur. The diagnosis is usually established by measuring serum vitamin B12 levels. The treatment is vitamin B12 replacement.

Guillain-Barre syndrome (choice A) is an acquired demyelinating neuropathy that usually follows a viral respiratory infection or immunizations. It is characterized by ascending weakness. Sensation is intact. A main diagnostic clue is absent deep tendon reflexes.

Lambert-Eaton syndrome (choice B) is a neuromuscular disorder that causes proximal muscle weakness, ptosis, and diplopia. Deep tendon reflexes are depressed or absent. Repetitive nerve stimulation shows increased responses. Small-cell carcinoma of the lung has been associated with this disorder.

Myasthenia gravis (choice C) is a neuromuscular disorder due to autoantibodies to the acetylcholine receptors. It is characterized by weakness and easy muscle fatigability. The extraocular and eyelid muscles are affected, leading to diplopia and ptosis. Deep tendon reflexes are preserved.

Polymyositis (choice D) is a skeletal muscle disorder that is characterized by progressive proximal muscle weakness. Patients often complain of difficulty climbing stairs and brushing hair. Ocular muscles are generally not affected. Creatine kinase is elevated. Muscle biopsy and electromyography confirm the diagnosis.

A 54-year-old African American man comes to the office complaining of swelling in his left scrotum. He states that the swelling has slowly gotten worse over the past 6 months and he can no longer feel his left testicle. As per the patient, the swelling itself does not cause pain. However, the swollen scrotal skin is rubbing against his thigh causing an irritation. The patient’s urologic history is significant for 2 episodes of epididymitis in the past 5 years. He denies any trauma to the scrotum, dysuria, hematuria, infertility, or prior similar episodes. There are no constitutional symptoms elicited with further questioning. The patient is afebrile and on examination the left hemi-scrotum is obviously enlarged and the scrotal skin is tense. There is no erythema of the scrotum. The left testicle is non-palpable. You are able to transilluminate light through the left scrotal mass. The mass is not reducible through the inguinal ring and it does not change in size or consistency with Valsalva or when the patient lies down. The right testicle is descended and normal to palpation. There is no urethral discharge, inguinal adenopathy, or abnormalities on rectal examination. Urinalysis and laboratory values are normal. The most likely underlying cause of this patient’s scrotal swelling is
Top of Form

A. dilatation of the pampiniform plexus

B. fluid collection within the tunica vaginalis

C. renal-cell carcinoma with invasion into the left renal vein

D. testicular neoplasm

E. torsion of the spermatic cord

Bottom of Form

Explanation:

The correct answer is B. This patient has a hydrocele. A hydrocele is a collection of fluid within the tunica (or processus) vaginalis. The diagnosis is made by finding a rounded cystic intrascrotal mass that is not tender unless underlying inflammatory disease is present. The mass transilluminates, helping to differentiate it from a testicular neoplasm. If the fluid is allowed to continue to build up, then it may eventually become large enough to prevent appropriate evaluation of the testicle. If a hydrocele develops in a young man, without apparent cause, then careful evaluation of the testicle and epididymis should be done in order to rule out cancer or infection. If the exact diagnosis is in question, then a scrotal ultrasound should be performed.

Varicocele is caused by dilatation of the pampiniform plexus (choice A). It is more common on the left side. On exam these patients have a mass of dilated, tortuous veins lying posterior to and above the testis. It may extend up to the external inguinal ring and is often tender. The degree of dilatation can be increased by the Valsalva maneuver. In the recumbent position, venous distention disappears. The sudden development of a varicocele in an older man is sometimes a late sign of a renal tumor that has invaded the left renal vein, thereby obstructing left spermatic vein drainage (choice C).

Testicular neoplasm (choice D) as previously stated does not transilluminate. Also, the history associated with a testicular tumor usually involves a dull, heavy feeling within the testicle. These masses have the potential to grow rapidly. Some testicular tumors are associated with a small hydrocele. With a testicular tumor, the testicle is hard, enlarged, and irregular in shape.

Testicular torsion (choice E) occurs when the spermatic cord twists, causing strangulation of the blood supply to the testis. Examination reveals a swollen, tender testicle that is retracted upward. It is more common in young boys who develop acute testicular pain.

A 41-year-old-man comes to the clinic because of right facial weakness for the past 6 days. He denies any pain or changes in hearing. The patient visited the office last week because of an upper respiratory infection for which you advised symptomatic therapy. He has no significant past medical history and takes no medications. Vital signs are: temperature 37 C (98.6 F), blood pressure 90/70 mm Hg, pulse 90/min, and respirations 15/min. Oxygen saturation is 99% on room air. Physical examination reveals weakness of the right side of the face with a droop. The left side of the face is normal. The remainder of a complete neurologic examination and a mini-mental status examination are normal. The next step in the management of this patient is to
Top of Form

A. administer prednisolone intravenous pulse therapy

B. advise him to take aspirin

C. order a CT scan of the head

D. order an MRI of the brain

E. order a nerve conduction study

F. prescribe ampicillin

G. provide reassurance and close clinical follow up

Bottom of Form

Explanation:

The correct answer is G. This patient has Bell’s palsy, an idiopathic inflammatory neuropathy of the facial nerve usually following a viral upper respiratory infection. Over 90% of patients recover without residual symptoms in 1 month. Oral prednisone with or without acyclovir is sometimes used to shorten the recovery period and possibly improve the outcome. However, this is based on a limited amount of studies.

Prednisolone therapy (choice A) is not indicated. High-potency intravenous steroids are not used in Bell’s palsy as they have no proven benefit and many side effects. Low-potency oral steroids are used anecdotally, although they have no proven benefit.

Aspirin (choice B) has no direct role in the treatment of Bell’s palsy.

A CT of the head (choice C) is not necessary in the evaluation of Bell’s palsy. Focal facial nerve neuropathy following an upper respiratory infection is the classic presentation of Bell’s palsy. The presence of other neurologic symptoms or signs would point to a primary neurologic process and would then warrant some type of neuroimaging.

An MRI of the brain (choice D) is not necessary in the evaluation of Bell’s palsy. Focal facial nerve neuropathy following an upper respiratory infection is the classic presentation of Bell’s palsy. The presence of other neurologic symptoms or signs would point to a primary neurologic process and would then warrant some type of neuroimaging.

Nerve conduction studies (choice E) are not necessary in the evaluation of Bell’s palsy.

Ampicillin (choice F) is not indicated because a facial nerve neuropathy is not due to a bacterial infection. The patient should be managed conservatively for Bell’s palsy.

A 17-year-old girl is brought to the office by her mother with a chief complaint of “severe acne that never responds to oral antibiotics.” The patient’s mother refuses to allow a trial of oral contraceptives for the fear that it will encourage her daughter toward sexual behavior with her boyfriend. The patient has heard that “the pill” makes you gain weight, “which is not acceptable.” They both have heard about isotretinoin and would like to be started on this medication as soon as possible. She has multiple inflammatory, deep-seated, cystic lesions on her face with evident pitted scarring. She has similar lesions on her chest and back. On further questioning, she denies being sexually active with her boyfriend and does not want to be on any form of birth control. Laboratory studies, including a liver function test and complete blood count, are unremarkable. Your nurse tells you the urinary HCG is negative from today’s visit. You should:
Top of Form

A. Explain to her and her mother that the Federal Drug Administration (FDA) requires effective contraception used one month before, during, and one month after isotretinoin therapy

B. Give her more oral antibiotics

C. Inject some of her cysts and reassure her of spontaneous improvement within 1-2 years

D. Prescribe isotretinoin and have her follow up in 1 month with urinary HCG and blood work

E. Tell her that isotretinoin is only for patients who have failed courses of oral contraceptives with oral antibiotics

Bottom of Form

Explanation:

The correct answer is A. Isotretinoin is a vitamin A derivative that has been associated with a high risk of birth defects. Since a number of pregnancies were reported in females on isotretinoin, the Food & Drug Administration now emphasizes the importance of two forms of effective contraception in females of reproductive age unless absolute abstinence is chosen (choice A).

Oral antibiotics (choice B) are not a good choice because this patient has already been on oral antibiotics with resultant nodulocystic acne and evidence of scarring. In this case, isotretinoin is the most effective agent, which will reduce follicular keratinization and sebum production.

Intralesional injection with steroid solution (choice C) is not correct because while this modality will offer her temporary improvement in the areas injected, the number of acne scars she will develop if not treated properly will have a deep impact on her psychosocial development.

Prescribing isotretinoin (choice D) without discussing contraceptive issues further at this point is incorrect, as stated above.

Prior to starting patients on isotretinoin, it is useful to try less teratogenic medications such as oral antibiotics, but there are no specific rules requiring patients to fail both oral contraceptives and antibiotics prior to initiation (choice E).

A 57-year-old woman with diabetes and nephrolithiasis is admitted to the medical services for evaluation of her chest pain that began when playing with her grandchild. She had a previous myocardial infarction and is status-post a three-vessel bypass two years prior. Her medications include atenolol, lisinopril, allopurinol, and atorvastatin daily. You are called to the patient’s room because the patient is currently complaining of chest pain. She reports that while talking on the telephone, she became very angry with her daughter and developed chest pain. On arrival she is lying in bed and appears uncomfortable. She is diaphoretic and appropriately anxious. Her blood pressure is 190/110 mm Hg and pulse is 110/min. She has an S4 gallop and scant bi-basilar rales. An electrocardiogram shows sinus tachycardia with a left axis deviation. Voltage criteria are met for LVH and there are ST segment depressions of 2.5mm in leads V1-V5. The most appropriate next step in management is to
Top of Form

A. administer verapamil, intravenously

B. give furosemide, intravenously

C. give morphine, intravenously

D. give nitroglycerin, intravenously

E. obtain a chest radiograph

Bottom of Form

Explanation:

The correct answer is D. This patient is having cardiac ischemia in the setting of increased myocardial oxygen demand. She has known coronary disease and her vital signs at the time of her angina episode show hypertension and tachycardia. The patient must have her blood pressure lowered acutely to the goal of terminating the angina with nitroglycerin.

Verapamil (choice A) is a calcium channel blocker that is used most often to slow the ventricular rate in patients with atrial fibrillation. These drugs are generally contraindicated in an acute myocardial ischemia as they tend to be associated with worse outcome.

Furosemide (choice B) may work to relieve some pulmonary congestion that this patient has as a result of her ischemia and increased left-sided filling pressures. It however fails to address the primary problem of her increased demand.

Morphine (choice C) is usually given to relieve pain, but nitroglycerin is more important at this time because it can relieve pain and also reduce the underlying ischemia.

A chest radiograph (choice E) would not be without benefit in this emergent situation, it has no role in altering any decision making or therapeutic intervention. This patient must have her blood pressure and heart rate lowered.

You are seeing a 34-year-old woman in the emergency department who is now 10 days post emergent C-section for preeclampsia at 29 weeks, and presents with a 3-day history of a headache. She describes it as “pain starting at the right temple and shooting through the head to the left temple”. There is some associated photophobia and phonophobia. At its worst, the headache is a 10 of 10 on the pain scale, with 10 being the worst pain that she ever felt in her life. This is similar to a headache she was having a month ago during her pregnancy. There is no relief with acetaminophen. Prior to delivery she had numbness of her right cheek that was transient. Her blood pressure is 160/90 mm Hg. Examination shows bilateral papilledema. The most appropriate next step in management is to
Top of Form

A. begin intravenous heparin

B. call a neurosurgical consult

C. discharge the patient with a prescription for naratriptan

D. discharge the patient with a soft cervical collar and prescription for metaxalone

E. order a brain MRI and MRV

Bottom of Form

Explanation:

The correct answer is E. This presentation is concerning for venous sinus thrombosis. MRV is the best noninvasive test to confirm this diagnosis. If the MRI/MRV is negative, then she should have a lumbar puncture to rule out idiopathic intracranial hypertension (pseudotumor cerebri).

Intravenous heparin (choice A) is the usual treatment for a venous sinus thrombosis, however, the diagnosis should be confirmed first if possible. Therefore, a brain MRI and MRV is the most appropriate next step in management.

A neurosurgical consult (choice B) is not indicated in this case.

Naratriptan (choice C) is used for the acute treatment of migraine headaches. This is atypical for a migraine, but she should be further evaluated before she is discharged and treated.

Soft cervical collar and metaxalone (choice D) are used to treat headaches from cervical muscle spasms. There was no neck muscle spasms noted on exam, and she should be further evaluated before she is discharged.

A 35-year-old woman comes to the office asking you to drug test her 12-year-old son. She states that her son is normally a very kind and interactive child, however, for the last 3 months, he has become increasingly withdrawn. He is in his room most of the time except for when he is at school. His grades have dropped from an A- average to C- this past semester. He refuses to see any of his friends and does not even eat much during dinner anymore. She has confronted her son multiple times about his situation, but he continually denies everything, including drug or alcohol use. The mother is visibly upset at this situation and is tearful about it. She tells you that you are her son’s “only help.” The most appropriate response to the mother’s request is:
Top of Form

A. “Just bring your son in and we can then get a urine test for alcohol and substance use.”

B. “I am sorry but I can’t legally test your son for any drugs without him consenting to it first.”

C. “It is possible that your son may be suffering from depression and I think that you should bring him in for me to talk to him.”

D. “These are classic symptoms for schizophrenia. The best thing to do would be to start him on some anti-psychotic medications.”

E. “Your son is most probably abusing drugs and alcohol, so you should send him to a rehabilitation facility immediately.”

Bottom of Form

Explanation:

The correct answer is C. The mother is right to be concerned over her son’s change in behavior. Certainly drugs and alcohol abuse can cause such dramatic behavioral changes. However, more likely would be a mental illness such as depression. The DSM–IV criteria for major depressive episode includes that at least one of the symptoms is either (1) depressed mood or (2) loss of interest in pleasure. The patient would also need to have 5 (or more) of the symptoms such as depressed mood, weight loss, insomnia, fatigue, suicidal tendencies, etc. Additionally, the symptoms should cause clinically significant distress or social impairment. It would also be important to screen the son for suicidality when the mother brings him in.

“Just bring your son in and we can then get a urine test for alcohol and substance use.” (choice A) would be appropriate if the son consented to the drug test. However, covert use of the son’s urine would only cause him to distrust not only his mother but also the physician, making any future interventions more difficult.

“I’m sorry but I can’t legally test your son for any drugs without him consenting to it first.” (choice B) is not true because the child is a minor and the parent is consenting to a drug test. However, it wouldn’t be going to the root of this child’s problem since he most likely suffers from a depressive disorder.

“These are classic symptoms for schizophrenia. The best thing to do would be to start him on some anti-psychotic medications.” (choice D) would be a premature statement since the diagnosis of schizophrenia requires both positive and negative symptoms occurring for 6 months. However, further questioning of the child may reveal that he is having a beginning prodrome for schizophrenia and starting him on anti-psychotics may be appropriate in the future.

“Your son is most probably abusing drugs and alcohol, so you should send him to a rehabilitation facility immediately.” (choice E) is inappropriate given the high likelihood that the son is suffering from depression. It is important to make sure that his depression is not due to the direct effects of a substance (e.g., a drug of abuse, a medication). However, evidence of substance abuse should first be obtained through discussion with the son or consented toxicology screens.

A 21-year-old woman comes to the clinic because of painful urination. She also reports that her urine has progressively turned from pink to red over the past 2 days. She has never had symptoms like this before. She is sexually active with 1 partner and they use condoms for birth control. She has a past medical history of depression treated with psychotherapy. Vital signs are: temperature 38.9 C (101.9 F), blood pressure 100/70 mm Hg, pulse 102/min, and respirations 12/min. Physical examination reveals left-sided costovertebral angle tenderness. The pelvic exam is unremarkable. Laboratory studies show:
The next step in the management of this patient should be

Top of Form

A. a blood culture and spinal fluid analysis

B. a blood culture and urinalysis

C. an MRI of the lumbar spine

D. an ultrasonography of the kidneys

E. an x-ray of the lumbar spine

Bottom of Form

Explanation:

The correct answer is B. Blood cultures and urinalysis are necessary because this patient has symptoms of pyelonephritis. The presentation of urinary symptoms, fever, leukocytosis, and bandemia is clinically diagnostic of pyelonephritis. A urine culture should also be obtained prior to starting antibiotics. Empiric broad spectrum antibiotics should be started and coverage can be narrowed when culture and sensitivity data are available. A renal ultrasound can also be performed at a later time to evaluate the kidneys if the patient does not respond to antibiotic therapy. The renal ultrasound is useful to evaluate for the complications of pyelonephritis such as abscess or obstruction.

This patient has no mental status changes or headaches that would necessitate a lumbar puncture (choice A)

This patient has no radiculopathy or back pain that would necessitate an MRI of the lumbar spine (choice C). Costovertebral or flank tenderness is often a sign of kidney inflammation or infection.

An ultrasonography of the kidneys (choice D) is not necessary at this time. A renal ultrasound can be performed if the patient does not respond to antibiotic therapy. The ultrasound is useful to evaluate for complications of pyelonephritis such as abscess or obstruction.

The physical examination of the back reveals flank, not lumbar spine pain. Hence, an x-ray of the lumbar spine (choice E) is not necessary.

A 63-year-old woman comes to the office because of a 3-week history of a “dull, achy” headache. She says that it started out as an intermittent headache that was exacerbated by bending down, lifting heavy objects, sneezing, defecating, and coughing, but lately it has become constant. She cannot associate the headaches with food or hunger, alcohol, weather or barometric pressure changes, sounds, or irregular sleep patterns. She is generally very healthy, but recalls having some nausea and vomiting a few weeks before the headaches started. She has never had headaches before. She does not take any medications, rarely drinks alcohol, and exercises regularly. Her temperature is 37.0 C (98.6 F), 130/80 mm Hg, pulse is 70/min, and respirations are 15/min. Physical examination is unremarkable. An erythrocyte sedimentation rate, complete blood count, and electrolytes are normal. A trial of oral prednisone, sublingual ergotamine, and oral sumatriptan is ineffective. The most appropriate next step is to

Top of Form

A. administer glucocorticoids, intravenously
B. administer sumatriptan, intramuscularly
C. give her oxygen inhalation therapy
D. obtain a temporal artery biopsy
E. order an MRI of the head
F. perform a lumbar puncture
Bottom of Form

Explanation:
The correct answer is E. In any patient with a new headache that is associated with nausea and vomiting and exacerbated by exertion or positional changes, a brain tumor should be suspected. This is the typical presentation of a posterior fossa brain tumor. Weight loss, systemic symptoms, and an elevated erythrocyte sedimentation rate are not typically associated with a primary brain tumor. Papilledema, caused by increased intracranial pressure, is rarely found in patients over 55 years old with an intracranial mass. An MRI or CT scan of the head will establish the diagnosis.

Administration of glucocorticoids, intravenously (choice A) is the appropriate treatment to prevent blindness in a suspected case of temporal arteritis. Temporal arteritis typically presents with a unilateral headache, myalgias, jaw pain, fever, and weight loss. A tender, reddened temporal artery is often found. Visual changes may occur and blindness is a feared complication. This patient does not have any of the usual systemic symptoms associated with temporal arteritis. The diagnosis of temporal arteritis is established with a temporal artery biopsy (choice D). The erythrocyte sedimentation rate is typically elevated in temporal arteritis

Administration of sumatriptan, intramuscularly (choice B) is the treatment for a migraine headache which typically presents with a throbbing headache, nausea, vomiting, photophobia, and functional impairment. An aura consisting of hallucinations and scotomas sometimes occurs before the headache. An intermittent headache that becomes constant is not the typical pattern of a migraine. The sublingual ergotamine and oral sumatriptan that this patient was taking are typically effective in treating a migraine.

Oxygen inhalation therapy (choice C) is the treatment for a cluster headache. A cluster headache is an episodic headache that typically presents with a few short headaches a day for a few weeks that is associated with periorbital pain, reddening of the eye, and lacrimation. This patient’s history is inconsistent with a cluster headache.

A lumbar puncture (choice F) should be performed in a suspected case of meningitis and possibly in a suspected intracranial hemorrhage, if the CT scan fails to show the bleed. Meningitis often presents with a headache, nuchal rigidity, and photophobia. Fever and a rash may be present. An intracranial hemorrhage may present with a headache and nuchal rigidity, seizures, and confusion. A CT scan of the head usually establishes the diagnosis. The patient’s history is inconsistent with meningitis and is more consistent with a brain tumor than an intracranial bleed. A lumbar puncture should not be performed in this patient or any patient with a suspected brain tumor because it may cause brain herniation.

A 31-year-old woman comes to the office for an effective, reversible form of contraception. She has recently gotten married and she and her husband are tired of using condoms. She took oral contraceptive pills (OCPs) while she was in college and she wants you to prescribe them for her again. She is generally very healthy and has no personal or family history of cancer or thromboembolic disease. She exercises 5 days a week, eats a low-fat diet, drinks a couple of glasses of wine each night with dinner, does not take any medications, and smokes a pack of cigarettes a day. You flip through her chart and note that she had an episode of pelvic inflammatory disease 4 years ago. Her blood pressure is 110/80 mm Hg. Physical examination is unremarkable. Pap smear and cervical cultures are normal. A urinary pregnancy test is negative. At this time you should
Top of Form

A. encourage her to stop smoking, explain risks, benefits, and synergistic effects, and prescribe OCPs

B. have her schedule an appointment on the first day of her next menstrual period

C. inform her that an intrauterine device is a better choice than OCPs because she smokes

D. recommend condoms, as they have the same estimated failure rates as OCPs and are safer for smokers

E. tell her that you will prescribe oral contraceptive pills when she quits smoking

Bottom of Form

Explanation:

The correct answer is A. Patients who take OCPs and smoke are at an increased risk for having a myocardial infarction or stroke. These risks are greatest at 35 years of age and older. As with all smokers, she should be encouraged to stop smoking. You should explain risks, benefits, and synergistic effects, and then you can prescribe OCPs.

There is no reason for you to have her schedule an appointment on the first day of her next menstrual period (choice B). All of her laboratory studies were normal, including a pregnancy test, so after you counsel her, you can give her a prescription. Even if the pill should be started on the first day of the next menstrual period, there is no reason not to give her the prescription now.

It is incorrect to inform her that an intrauterine device (IUD) is a better choice than OCPs because she smokes (choice C) because a prior history of pelvic inflammatory disease is a contraindication for an IUD.

The estimated failure rates (choice D) are not the same for OCPs and condoms. The estimated failure rate is 1-3% for OCPs and 5-15% for condoms.

While she is at an increased risk for a myocardial infarction and stroke, it is inappropriate to tell her that you will prescribe oral contraceptive pills when she quits smoking (choice E). Smoking in a 31-year-old woman is not a contraindication for OCPs. You should encourage her to quit, explain the risk and benefits, and prescribe OCPs.

You are seeing a 79-year-old woman with hypertension, diabetes, coronary artery disease, and rheumatoid arthritis in your office for a routine follow-up visit. Her medications include spironolactone, amiloride, NPH insulin, aspirin, prednisone, and ketorolac. Her temperature is 37 C (98.6 F), blood pressure is 99/56 mm Hg, pulse is 58/min, and respirations are 19/min. Physical examination is unremarkable. An electrocardiogram shows a sinus rhythm and non-specific ST and T wave abnormalities. Her serum sodium is 136 mEq/L and serum potassium is 5.8 mEq/L. A review of her laboratory values over the past 6 months indicates that her serum potassium has been gradually increasing. The most appropriate management at this time would be to

Top of Form

A. add clonazepam to her current therapy
B. add metoprolol to her current therapy
C. discontinue the amiloride
D. discontinue the ketorolac
E. discontinue the prednisone
Bottom of Form

Explanation:
The correct answer is C. Spironolactone, an aldosterone antagonist, causes an increase in serum potassium. Amiloride, a potassium sparing diuretic, has a similar effect. The use of both these agents can lead to potentially dangerous hyperkalemia. If hyperkalemia does manifest, one of them should be discontinued.

Clonazepam (choice A), a benzodiazepine anxiolytic, is not warranted since there is no evidence that the patient is anxious.

Metoprolol (choice B) will not decrease serum potassium. Infact, beta-blockers can increase serum potassium. The patient’s heart rate and blood pressure also do not mandate a beta-blocker at this juncture.

Discontinuation of ketorolac (choice D), a nonsteroidal antiinflammatory agent, is unnecessary since it is not associated with hyperkalemia.

Discontinuation of prednisone (choice E) is not needed, since prednisone does not cause or worsen hyperkalemia.

A 25-year-old woman comes to the office because of a 3-day history of extreme vulvar itching and a thick, white vaginal discharge. She says that she is so uncomfortable that she cannot sit still at work because she “constantly feels the need to itch.” You notice in her chart that she has had 2 similar episodes over the past 6 months, and her boyfriend was simultaneously treated both times due to her intense fear of reinfection. She plays on the company softball team, goes hiking with her boyfriend and dog, and is currently training for a marathon. She says that she showers and changes her clothing immediately after working out, does not wear tight pants, and only uses panty liners during her menstrual period. She has been taking doxycycline daily for acne for the past 2 months, and was taking erythromycin for acne during the 6 months prior to switching to doxycycline. Physical examination shows erythematous vulvar and vaginal walls, and a thick, white vaginal discharge that is adherent to the vaginal walls. The pH of the discharge is 4.8. Laboratory studies show:
She is concerned that she has had recurrent infections. The most important information to share with this patient is that

Top of Form

A. her boyfriend may be having sexual relations with other people and transmitting the organism back and forth

B. she probably has early diabetes and she needs to return for serial urinalyses during the next 6 months

C. she should have an HIV test to rule out an underlying immunodeficiency

D. the condition is most likely due to the alteration in vaginal flora and pH caused by the antibiotics

E. this chronic condition is most likely due to the warm, moist environment caused by the panty liners

Bottom of Form

Explanation:

The correct answer is D. This patient has Candida vaginitis that is most likely due to the long-term use of broad-spectrum antibiotics. The chronic use of antibiotics alters the finely balanced vaginal environment that is normally very acidic from the vaginal flora. Lactobacillus is one of the main components of normal vaginal flora that helps to maintain this acidic environment. Any factors that affect the normal pH and decrease the lactobacilli, increase the risk that a vaginal infection with a pathologic organism may occur. Antibiotic use is a known risk factor for altering the normal microbiology of the vagina. Some physicians prescribe prophylactic therapy with an antifungal agent when they prescribe long-term systemic antibiotics.

It is inappropriate to tell her that her boyfriend may be having sexual relations with other people and transmitting the organism back and forth (choice A) because her recurrent infections are most likely due to the long-term use of broad-spectrum antibiotics for acne. These infections coincide with the use of the antibiotics. Also, since her boyfriend was treated during her previous infections, it is unlikely that he is the source of reinfection. If she had no other risk factors for Candidal infections, this may be a possibility, but this is not the case.

She should not be told that she probably has early diabetes and that she needs to return for serial urinalyses during the next 6 months (choice B) because her glucose levels are normal and she has been taking broad-spectrum antibiotics for months, which is a known risk factor for Candidal infections. Serial urinalyses for the next 6 months is not indicated.

If she had chronic Candidal infections and no known risk factors that alter the vaginal environment, it may be appropriate that she have an HIV test to rule out an underlying immunodeficiency (choice C). However, her recurrent infections are most likely due to the long-term use of broad-spectrum antibiotics for acne.

Since she only uses panty liners during her menstrual period, it is unlikely that this chronic condition is most likely due to the warm, moist environment caused by the panty liners (choice E). Panty liners create a warm, moist vaginal environment that may increase the risk of infections with Candida. However, this usually occurs in individuals who wear panty liners every day. This answer is also unlikely because she has been taking broad-spectrum antibiotics for months, which is a known risk factor for Candidal infections.

A 37-year-old woman comes to the office because of a “burning sensation” in the chest for the past 3 months. The “burning” typically begins in the “upper stomach and travels up to the neck.” The symptoms worsen when she lies down to go to sleep. She is a chef at a local American restaurant, has 3 children, and has been married for 12 years. She “tries” to eat a healthy diet, but it is difficult because she is around food all day and night. She has no chronic medical conditions, takes no medications, and does not drink alcohol or caffeine-containing beverages. She recently quit smoking. Her temperature is 37.0 C (98.6 F), blood pressure is 120/80 mm Hg, pulse is 65/min, and respirations are 14/min. Physical examination is unremarkable. An electrocardiogram is unremarkable. A complete blood count and metabolic profile are normal. Serologic testing for H. pylori is negative. The most appropriate next step is to
Top of Form

A. order ambulatory esophageal pH testing

B. order an upper gastrointestinal barium radiograph

C. recommend elevation of the head of bed and avoidance of food before bedtime

D. schedule an upper endoscopy

E. schedule esophageal manometry

Bottom of Form

Explanation:

The correct answer is C. This patient complains of the classic symptoms of gastroesophageal reflux disease (GERD). Reflux disease is usually worse at night because the recumbent position allows gastric acid contents to go up into the esophagus. Since all of the tests ordered in the case were normal, you should first recommend non-pharmacologic therapy before continuing with further diagnostic studies. Elevation of the head of bed, avoiding eating before bed, and avoiding alcohol, tobacco, chocolate, and caffeine should all be recommended. Alcohol, tobacco, chocolate, and caffeine all lower the lower esophageal sphincter pressure leading to gastric reflux. If these measures are ineffective, pharmacologic therapy with a H2 blocker such as cimetidine, famotidine, or ranitidine is indicated. For more severe symptoms, a proton pump inhibitor, such as omeprazole or lansoprazole, is indicated.

Ambulatory esophageal pH testing (choice A) is usually reserved for patients who fail nonpharmacologic and pharmacologic management.

An upper gastrointestinal barium radiograph (choice B) is useful in detecting esophageal rings or strictures, which typically present with dysphagia. This patient complains of heartburn, not dysphagia.

An upper endoscopy (choice D) is usually indicated only after the failure of nonpharmacologic and pharmacologic management for GERD and when a patient has GERD for >5 years, and upper endocsopy is recommended to screen for Barrett’s metaplasia. However, it is not indicated at this time.

Esophageal manometry (choice E) is typically reserved for cases of GERD when surgical therapy is being considered.

A 71-year-old man is postoperative day number 12 from a heart transplant. The patient has a long-standing history of ischemic cardiomyopathy and successfully underwent a 5-hour transplant from a 22-year-old donor. During the procedure he was started on his immunosuppressive therapy that has continued. In the immediate postoperative period he did well. He was extubated on day number 1, had his pulmonary artery and radial arterial catheters removed on day number 3, and was transferred from the coronary care unit on day number 4 with excellent pain control. Over the past 3 days however, he has had increasing fever, lethargy, and mediastinal tenderness. The sternal wound appears mildly erythematous but nonsuppurative. On palpation, the incision site is tender. The most appropriate therapy is at this time is
Top of Form

A. broad-spectrum antibiotics

B. decreasing dose of immunosuppressive drugs

C. initiate antifungal therapy

D. surgical debridement

E. there is no therapy indicated

Bottom of Form

Explanation:

The correct answer is A. This patient likely has mediastinitis, a common postoperative complication after heart transplant. The combination of the nature of the wound (median sternotomy) with the profound immunosuppression creates a furtive environment for mediastinal infection. Such patients require broad-spectrum antibiotic therapy.

Because of the transplant and the possibility of acute-rejection, decreasing the dose of immunosuppressive drugs (choice B) is not tenable.

Bacteria cause most mediastinal infections in the immediate and intermediate postoperative periods. Therefore, fungal infections in transplant recipients and the need for antifungal therapy (choice C) are not usually an issue until 6-9 months of immunosuppressive therapy.

Surgical debridement (choice D) is usually required when the sternum undergoes dehiscence or there is necrotic tissue present that can become a culture medium for additional infections. There is no indication that this patient’s infection is that severe.

Because mediastinitis is a serious infection and can cause sternal dehiscence and chronic infection, antibiotic therapy is indicated despite concerns for selecting for resistant organisms (choice E).

A 44-year-old woman comes to see you for routine check up. While in your office, she starts crying hysterically, stating she has not been able to sleep for the last few months. Also, her mind has been “racing,” her palms and soles are sweaty at all times, and she has thinning of her hair. She denies any alcohol or tobacco use, but admits to 2 cups of coffee a day. Her mother has bipolar disorder and her aunt has obsessive-compulsive personality disorder. Her blood pressure is 130/80 mm Hg and pulse is 100/min. There is notable exophthalmos bilaterally. He skin appears moist and warm. On the pretibial regions, there is a woody induration with pitting edema. The most appropriate next step in evaluation is to
Top of Form

A. determine testosterone level

B. give her aluminum chloride for her sweaty palms and reassure her this is all psychological

C. order thyroid function tests

D. refer her to an ophthalmologist for evaluation of the exophthalmos

E. refer her to a psychiatrist for evaluation of a psychiatric disorder

Bottom of Form

Explanation:

The correct answer is C. Ordering a thyroid function test is correct, because this patient demonstrates not only physiological hyperthyroid changes (i.e. increased heart rate), but also cutaneous findings classic for Graves disease. Skin changes are distinctive in hyperthyroidism. The cutaneous surface is warm, moist, and smooth textured. Palmar erythema or facial flushing may be seen. The hair is thin and has a downy texture and nonscarring alopecia may be observed. Graves disease has a female to male ratio of 7:1. Thyroid acropachy is characterized by digital clubbing and diaphyseal proliferation of the periosteum in acral and distal long bones (tibia, fibula, ulna, and radius). Pretibial myxedema consists of bilateral localized, cutaneous accumulations of glycosaminoglycans and occurs in 4% of patients who have or have had Graves disease. Improvement in plaques of pretibial myxedema have resulted from intralesional injections of triamcinolone acetonide and with clobetasol solution under Duoderm occlusion, applied once weekly for 4-6 weeks. Systemic steroids are of no benefit.

Checking testosterone level (choice A) is incorrect, because an elevated testosterone level would only account for hair thinning, but not her other physiologic and cutaneous changes.

Giving her aluminum chloride (choice B) is incorrect, because this solution will only alleviate sweaty palms and soles for this patient, but does not help to diagnose and treat the underlying issue.

Referring her to ophthalmology (choice D) is incorrect, because eventually this patient may need to see an ophthalmologist for severe exophthalmos, but diagnosing Graves disease is the more appropriate first step for this patient.

Despite her family history of psychiatric illnesses, referral to psychiatry (choice E) is incorrect, because this patient’s manic appearance is most likely secondary to thyroid hormone imbalance.

A 34-year-old man comes to the office because of “erectile problems.” He says that he and his wife have not had sexual intercourse in months because he has not been able to have an erection. He says that it is so upsetting that they have basically stopped trying because it just makes both of them “depressed”. He has seen so many television commercials lately that he expects a cure in the form of a “little blue pill.” He is married, has 3 kids (age 1,3, and 6), works as a narcotics police officer, and competes in triathalons on the weekends. He takes no medications, rarely drinks alcohol, and has had no serious medical conditions. Physical examination is normal. The most appropriate next step in evaluating this patient’s erectile disorder is to ask him
Top of Form

A. “Are you feeling unusually anxious lately?”

B. “Are you sexually attracted to your wife?”

C. “Do you have nocturnal or early morning erections?”

D. “Do you love your wife?”

E. “How often do you ride your bicycle?”

Bottom of Form

Explanation:

The correct answer is C. The main issue in evaluating impotence is distinguishing between psychological causes and organic causes. The presence of nocturnal or early morning erections basically eliminates the organic causes and leads to a diagnosis of psychological impotence. Up to 90% of erectile disorders are due to psychogenic factors. The most common psychological causes include anxiety and depression. Nocturnal penile tumescence, which occurs during REM sleep, can be assessed in a sleep lab or by a stamp test (wrapping stamps around the penis before bedtime and checking in the morning if the “ring” is broken at any of the perforated areas). If nocturnal erections are not present, the impotence is most likely due to an organic cause. The most likely causes are testicular failure, hyperprolactinemia, medications, alcohol, opioids, nicotine, trauma, priapism, diabetes, vascular disease, and neurologic diseases, such as diseases of the spinal cord, and loss of sensory input. Physical examination should include a detailed genital exam, evaluation for signs of feminization, neurologic, and vascular exams. “The little blue pill” (Viagra) is not indicated in a patient with psychogenic erectile disorders.

“Are you feeling unusually anxious lately?” (choice A) is a good question to follow the question about nocturnal erections (if he is having them).

“Are you sexually attracted to your wife?” (choice B) is a relevant question if the patient is having nocturnal erections and is not depressed or anxious. “Do you love your wife?” (choice D) is a question that may be asked if he is having nocturnal erections and is anxious and depressed. It may be a little blunt and can probably be asked in a more subtle manner.

“How often do you ride your bicycle?” (choice E) may be relevant because of neurologic and vascular compromise caused by the seat. However, it is not the most helpful question to distinguish between organic and psychologic causes.

A 42-year-old woman who you have been treating for anxiety calls your office demanding to talk to you . The nurse gets you immediately because she sounds “out of control.” When you finally get to the phone, she is screaming that, “a plumber is working in her house and he hurt his eye.” You ask for the details of the injury and she says that all she knows is that he “got something in his eye and he is freaking out.” You instruct her to look at the bottle and read the label to you, but all you can understand from her hysterical speech is, ” pH of 12.2.” You hear a man in the background screaming in pain. You try to calmly explain to her that the most appropriate, immediate management is to
Top of Form

A. call an ambulance to bring him to the hospital, and place a patch over the affected eye

B. call the ophthalmologist and schedule an appointment for the next available time

C. flush his eye with a substance that she can find in her cabinet with a pH of 6.9 or lower

D. pry the eye open and flush it continuously with cold running water before going to the hospital

E. pry his eye open and vigorously rub the entire eye, especially under the upper and lower lids

Bottom of Form

Explanation:

The correct answer is D. This patient most likely has an alkaline eye burn, which is extremely destructive, and the process of destruction continues as long as the substance is in contact with the tissues. Immediate removal is essential, and the best method is massive irrigation. In the emergency room, sterile saline would be used, but at home, tap water will do. This irrigation is important even before going to the hospital (because the alkaline fluid will remain in the eye if you leave the irrigation to the emergency room physicians).

It is extremely dangerous to call an ambulance to bring him to the hospital, and place a patch over the affected eye (choice A). The alkaline chemical will begin to destroy the eye if you do not begin immediate irrigation while waiting for help.

Alkaline eye burns are emergencies that require immediate irrigation and medical attention. Calling the ophthalmologist and scheduling an appointment for the next available time (choice B) will leave too much time for the chemical to destroy the eye.

It would be inappropriate to tell her to flush his eye with a substance that she can find in her cabinet with a pH of 6.9 or lower (choice C). Irrigation with cold tap water is the treatment while waiting for additional medical help (in the ambulance or at the hospital).

While it is necessary to try to remove as much of the chemical as possible, telling her to pry his eye open and vigorously rub the entire eye, especially under the upper and lower lids (choice E) is incorrect. Massive irrigation is used to dilute the chemical and wash away any particles remaining in the eye. Gentle swiping may be appropriate later on in the treatment to wipe away any little remaining particulate matter, however, this may be done after irrigation.

A 12-year-old boy with asthma is brought to the emergency department by his mother because of intermittent right hip pain for the last 2 weeks. The pain is non-radiating and worse with activity. It has now become more constant, worse with weight bearing, and over-the-counter analgesics only give minimal relief. There is no history of night pain and he denies any recent trauma, weight change, or any constitutional symptoms such as fever, chills, or night sweats. He tells you that he went to his pediatrician’s office 10 days ago for the same hip pain and he was told that his physical examination and laboratory studies, including a complete blood count and erythrocyte sedimentation rate, were unremarkable. The pediatrician’s diagnosis was a “pulled muscle or tendon” in the right hip region and he was advised to rest. Now in the hospital, his physical examination shows an obese patient, a limping gait, a leg length discrepancy of 0.5 cm, and intact motor-sensory examination of lower extremities bilateral. The right hip region has intact skin and no focal tenderness to palpation. Passive range of motion of the right hip is decreased on internal rotation. When the hip is flexed, the thigh externally rotates. The most appropriate next step in management is to
Top of Form

A. admit him to the hospital for intravenous antibiotic therapy, non-weight bearing right lower extremity, and obtain an orthopaedic consultation

B. recommend immediate crutch walking and non-weight bearing of right lower extremity and obtain pelvic x-rays

C. repeat complete blood count and erythrocyte sedimentation rate; if the results are normal then send him home with 1 week follow-up in the office

D. send him home with a prescription for a 0.5 cm shoe insert and physical therapy

E. send him home with a 10-day course of oral antibiotics with a 2-week follow-up in the office

Bottom of Form

Explanation:

The correct answer is B. A skeletally immature and obese 12-year-old patient with this history of hip pain is most likely to have a slipped capital femoral epiphysis (SCFE). This growth plate condition does not occur after skeletal maturity, and for males, maturity is on average at 16 years of age. The obligatory external hip rotation with hip flexion is a very common finding in a slipped epiphysis. Immediate non-weight bearing protects the femoral epiphysis from further injury and potential osteonecrosis. Common x-ray findings are consistent with a slippage of the femoral epiphysis. The slippage has been compared to a scoop of ice cream that has partially slipped from the cone. A double density sign and a break in Klein’s line are two x-ray findings. Treatment requires internal fixation of the epiphysis with long screws. The risks of delayed or non-treatment are further slippage and osteonecrosis.

An orthopaedic consult (choice A) will result in discontinuance of the antibiotics and ordering pelvic x-rays. Non-weight bearing is correct and protective, but antibiotics are not indicated because this is not an infectious process.

This patient does not have transient synovitis or any other infectious or inflammatory processes.Therefore, repeating a complete blood count and erythrocyte sedimentation rate, and sending him home if the results are normal with a one week follow up is not necessary(choice C) is incorrect. Again, a delay in diagnosis will result in significant morbidity.

Leg length discrepancy is very common, and less than 2 cm does not typically require treatment. Patients are very often misdiagnosed as having muscle strains or tendon injuries. This delay in diagnosis results in significant morbidity. Sending him home with a prescription for a 0.5 cm shoe insert and physical therapy (choice D) does not address the problem.

This patient does not have an infection. Therefore, antibiotics and discharge to home (choice E) will only delay the correct diagnosis.

A 67-year-old man presents to your office complaining of severe shortness of breath and a cough. The man is a long-time patient who you have been treating for chronic obstructive pulmonary disease (COPD). He has a 120 pack-year smoking history. He was recently hospitalized for a flare of his COPD and he has been intubated in the past for respiratory distress. His other medical history is notable for diabetes mellitus and hypercholesterolemia. He reports to you that for the past 3 days he has increasing dyspnea and a fever. He has gradually developed a cough. On examination, he is moderately dyspneic at rest and has marked dyspnea on exertion. His blood pressure is 130/80 mm Hg, heart rate is 78 /min, and respirations are 20 /min. The patient does not appear cyanotic. His lung exam demonstrates a markedly prolonged expiratory time and diffusely diminished breath sounds. The most appropriate intervention at this time is to
Top of Form

A. admit the patient to the hospital

B. obtain a chest radiograph and prescribe antibiotics if an infiltrate is present

C. prescribe oral cefuroxime and oral steroids

D. prescribe oral cefuroxime and see the patient in seven days

E. refer the patient to the local emergency department

Bottom of Form

Explanation:

The correct answer is A. This patient has long-standing pulmonary disease and is suffering from some acute respiratory event, either infection or a COPD flare. His past medical history is enlightening because it can be appreciated that this patient has required ventilatory support for his pulmonary disease in the past. In the office, he is clearly in some distress. With his history and known lung disease, this patient should be admitted to the hospital for further care and observation.

Similar reasoning holds for obtaining a chest radiograph and prescribing antibiotics if an infiltrate is present (choice B). Knowing that his distress is due to pneumonia does not alter the fact that he needs to be treated in the hospital.

Prescribing oral cefuroxime and oral steroids (choice C), although a reasonable regimen for a COPD flare, fails to address how this patient’s acute respiratory compromise will be addressed. He has been intubated in the past for flares such as this and discharging the patient out of your care could result in serious harm or even death to this patient.

Prescribing oral cefuroxime and see the patient in seven days (choice D) is clearly inappropriate since the patient is presently in mild to moderate distress and the cause for this is unknown. Even if he does have an infection, the antibiotics will take days to be of any assistance.

It would be inappropriate to refer the patient to a local emergency department (choice E) where he may sit unobserved in triage or may be discharged home if another physician had a different impression of the patient.

A 57-year-old woman with coronary artery disease associated with hyperlipidemia comes to the clinic for a scheduled follow-up appointment. She saw you for the first time approximately 1 month ago to establish care. In the interim, she was started on hydrochlorothiazide for elevated blood pressure (confirmed on a repeat nurses visit) and on simvastatin for a fasting LDL of 190. She has a remote history of alcoholism, but denies any alcohol intake over the past 10 years. Today, she complains of mild, generalized weakness and states that her shoulders and thighs are “achy”. She denies rhinorrhea, fevers, chills, nausea, vomiting, or diarrhea. While she does not complain of any dysuria, she states that her urine has been very dark for the past few days. She denies abdominal or flank pain. Laboratory studies show a mildly elevated white blood cell count, a normal hematocrit, and normal electrolytes. Her AST (or SGOT) is 415 and her ALT is 25. Bilirubin and alkaline phosphatase are within normal limits. The most appropriate next step in evaluation is to
Top of Form

A. determine creatinine kinase level

B. obtain an erythrocyte sedimentation rate

C. order a GGT level and a serum alcohol level

D. send Hepatitis A, B, and C serologies

E. send her for a right upper quadrant ultrasound

Bottom of Form

Explanation:

The correct answer is A. AST is less specific for liver than ALT. AST is found in multiple organs and will be elevated with any muscle injury. In fact, before the advent of assays for the MB fraction or troponin, AST was used to assess for myocardial infarction. The patient was recently placed on simvastatin (an HMG Co-A reductase inhibitor). While these cholesterol-lowering drugs are generally benign, myositis is a complication that the prescribing physician must be aware of. The patient’s presentation of fatigue and muscle aches fits the diagnosis of myositis. In addition, an elevated AST with an otherwise completely normal liver panel should heighten suspicion that the AST is not coming from the liver. The patient’s dark urine is classic for myoglobinuria. The patient should be treated with intravenous fluids to maintain renal perfusion, therefore avoiding renal tubular injury from the myoglobin. Depending on the level of the creatinine kinase, alkalinizing the urine may also help protect the kidney from injury in this setting. There is no definitive treatment for the myositis itself. Typically, the myositis resolves after the offending agent (simvastatin in this case) is discontinued.

An erythrocyte sedimentation rate (choice B) is incorrect. While an erythrocyte sedimentation rate may be useful as a sensitive marker of inflammation, it is not specific for any disease process. As this case illustrates, recently prescribed medicines should always be considered at the top of your differential diagnosis as the etiology of a new disease process. The erythrocyte sedimentation rate would not help make the diagnosis, nor would the result change management.

An elevated AST to ALT ratio may be suggestive of alcoholic liver injury. This is thought to be due to the fact that ethanol decreases ALT synthesis. In addition, ethanol is thought to cause mitochondrial damage in the liver, where AST lives. However, the ratio of AST to ALT in alcoholic hepatitis is more frequently closer to 2:1. The extremely high ratio in this case (again, with a normal ALT) points to an extrahepatic process. An elevated GGT can reflect alcoholic liver damage, but again, this is unlikely in the face of completely normal bilirubin and alkaline phosphatase. Therefore, ordering a GGT level and a serum alcohol level (choice C) is not correct.

Hepatitis A, B, and C serologies (choice D) are incorrect because the viral hepatitides should not cause an isolated level in AST.

A right upper quadrant ultrasound (choice E) is incorrect mainly for the same reasons as elucidated above. A right upper quadrant ultrasound is useful for evaluating suspected structural disease. Typically, structural disease of the liver is suspected when there are clues of hepatic obstruction. Elevated bilirubin and alkaline phosphatase are typically elevated in hepatic obstruction, and both are normal in this case.

A recently published study examined the efficacy of a new drug, Nomomigrane, for treating migraine headaches. For this randomized, double-blind study, 200 migraine sufferers were give Nomomigrane for 6 months, while another 200 migraine sufferers were given a placebo. At the end of the 6 months, all subjects reported whether or not they had been migraine free for the last 3 months. A key table from this study is presented below.
The report included the statement that Nomomigrane worked better than the placebo and assigned a significance level of p < .001. Based on this and other studies, Nomomigrane was recently approved by the FDA for the treatment of resistant migraine headaches. You are considering prescribing Nomomigrane for a patient suffering from severe migraines. As the treatment option is presented the patient asks, ” I know this worked in the research, but what is the chance that the drug will really work for me?” Based on the presented research the most appropriate response is:

Top of Form

A. “I’m very excited about the potential for this new drug and think you will be very pleased with the results.”

B. “One can never tell about these things before hand. We’ll just have to try it and see.”

C. “The drug was reported effective in 6 out of 10 patients.”

D. “The results are strong and convincing. You can have every confidence that this drug will work for you.”

E. “The statistical significance was high in the study. The chance of the drug not working for you is less than 1 in a 1,000.”

F. “This treatment will cut the number of migraines you have been experiencing in half.”

G. “Why do you ask? Do you have concerns about my treatment recommendation?”

Bottom of Form

Explanation:

The correct answer is C. This question asks about clinical efficacy. In the presented study, 120 of the 200 patients given the drug did not report migraines in the measurement period (120/200 = 60%).

Expressing enthusiasm (choice A and D) about a prescribed treatment does help to engender confidence in the treatment on the part of the patient. However, this response does not answer the patient’s question.

Of course, individual patients may be the exception to the rule (choice B), but this research tells us that the drug is likely to be effective. The probability is that the drug will work for this patient.

Statistical significance is high, less than one per 1,000 chance of Type I error. However, statistical significance tells us little about clinical significance. Knowing the “p-value” tells us nothing about the number of patients who get better or the chance that an actual patient will get better (choice E).

The treatment group did report half the migraines compared with the placebo group, but this is a misuse of this ratio. The study did not measure the reduction in migraines within each person. Rather, it examined the presence/absence of migraines between the treatment and control groups. In short, this study does not tell us what sort of reduction in migraines an individual patient might expect, therefore (choice F) is incorrect.

Inquiring after a patient’s concerns over treatment (choice G) is a reasonable tack, but does not answer the question posed by the patient. If the patient asks a question, you should try to answer it.

A 35-year-old homeless man is admitted to the hospital because of psychosis and agitation. He received intramuscular haloperidol while in the emergency department 16 hours earlier. He is now obtunded, with a temperature of 39.1 C (102.4 F) and has muscular rigidity. He has had wide fluctuations in blood pressure over the past 12 hours. Laboratory studies show a creatine phosphokinase in excess of 20,000 U/L and a leukocyte count of 12,800/mm3. Supportive measures such as intravenous fluids and airway support are given and the patient is noted to be hemodynamically stable. The pharmacologic agent that would be contraindicated at this time is
Top of Form

A. acetaminophen

B. benztropine

C. bromocriptine

D. dantrolene

E. lorazepam

Bottom of Form

Explanation:

The correct answer is B. The patient has a diagnosis of neuroleptic malignant syndrome due to rapid administration of high doses of intramuscular haloperidol. The patient’s relatively young age and male sex are predispositions for the development of neuroleptic malignant syndrome, though the biggest risk for any patient is the rapid administration of high potency neuroleptics. Because of the patient’s fever and autonomic instability, benztropine, an anticholinergic medication with a tendency to promote heat retention, would be contraindicated in a patient with neuroleptic malignant syndrome.

Because of the high fevers associated with neuroleptic malignant syndrome, an antipyretic such as acetaminophen (choice A) that is usually given parenterally may be necessary.

Bromocriptine (choice C) is a dopamine agonist that is frequently used in the treatment of neuroleptic malignant syndrome. One of the etiologic theories of neuroleptic malignant syndrome is a dysregulation of dopamine receptors in the hypothalamus resulting in impairment in body temperature control from overblockade of hypothalamic dopamine receptors.

Dantrolene (choice D) is a muscle relaxant frequently used in the treatment of neuroleptic malignant syndrome and may have efficacy in preventing renal failure secondary to myoglobinuria.

Lorazepam (choice E) can be used in the treatment of neuroleptic malignant syndrome both for its muscle relaxant and sedative qualities.

A 75-year-old man who lives in an elder care facility is brought to the emergency department because of a sudden onset of severe, colicky abdominal pain. He is generally in good health and does not take any regular medications. Alcohol and cigarettes are not permitted in his living facility. His temperature is 37 C (98.6 F), blood pressure is 110/70 mm Hg, pulse is 65/min, and respirations are 16/min. Physical examination shows abdominal distention and high-pitched tinkling sounds. An abdominal x-ray shows a dilated colon, forming an “omega loop,” with the narrowed colonic segment pointing to the right lower quadrant. Intravenous fluids are started and a nasogastric tube is placed. The most appropriate management at this time is to
Top of Form

A. admit him to the hospital and begin sulfasalazine and corticosteroids, intravenously

B. admit him to the hospital and begin vancomycin and metronidazole, intravenously

C. admit him to the hospital and observe

D. arrange for immediate laparotomy

E. perform a sigmoidoscopy with rectal tube placement

Bottom of Form

Explanation:

The correct answer is E. This patient most likely has a sigmoid volvulus, which is caused by the bowel twisting on its mesentery, and leads to obstruction and possibly vascular compromise. It typically presents with the acute onset of abdominal pain and obstipation. Patients will have abdominal distention and bowel sounds that are characterized by high-pitched “rushes and gurgles.” The abdominal x-ray shows a dilated colon forming a loop, with the narrowed segment pointing to the obstruction site. Nonoperative reduction with a sigmoidoscopy with rectal tube placement is generally effective in treating the condition. Individuals in nursing homes and mental institutions have an increased risk for developing a sigmoid obstruction, however, the reason is not yet established.

Sulfasalazine and corticosteroids, intravenously (choice A) are used in acute management of ulcerative colitis (UC), which is a disease that typically affects younger individuals. It is characterized by fever, bloody diarrhea, and abdominal pain. A barium enema typically shows a loss of haustral markings, which is generally referred to as a “lead pipe” pattern. Complications include toxic megacolon, perforation, colon cancer, and hemorrhage. This patient is not the typical UC patient, and the clinical findings are more consistent with a sigmoid volvulus than UC.

Vancomycin and metronidazole (choice B) are used to treat pseudomembranous colitis, which is due to Clostridium difficile. It typically occurs during or after the use of antibiotics and presents with severe watery diarrhea. Treatment of pseudomembranous colitis is vancomycin or metronidazole, given orally, to reach high levels in the stool. Treatment is usually with one or the other medications, not both at the same time. This patient is not taking any medications and the clinical findings are more consistent with a sigmoid volvulus than pseudomembranous colitis.

It is inappropriate to admit him and observe (choice C), because he most likely has a sigmoid volvulus, which is a colonic obstruction that requires immediate intervention to prevent strangulation, vascular compromise, and infarction.

If a patient with a sigmoid volvulus presents with generalized abdominal pain, fever, and hypovolemia, immediate laparotomy (choice D), may be necessary because strangulation has probably already occurred. This patient has stable vital signs, which makes this unlikely.

A 20-year-old comes to the clinic because of problem that has bothered her for “a while”. She has had difficulty throughout her academic career in spite of studying an average of 5 hours per night and taking many extra sessions of tutoring in her classes. She is not able to concentrate due to having to continually check that the door in her apartment or study area of the library is locked. She checks to see that it is locked an average of 15 times an hour while trying to study. She is fully aware each time she checks that the door is locked behind her, but cannot resist the temptation to check to make sure. She sometimes counts to 100 backward and forward after checking that her door is locked in order to distract herself, however, this only provides temporary relief. There is no history of abuse, no history of hallucinations, and the patient states overtly that she does not fear for her safety while checking her door. An appropriate medication to treat this patient’s condition is

Top of Form

A. haloperidol
B. lithium
C. lorazepam
D. paroxetine
E. valproic acid
Bottom of Form

Explanation:
The correct answer is D. This patient suffers from obsessive-compulsive disorder and demonstrates compulsive checking rituals that she acknowledges are not grounded in reality-based concerns. The treatment of choice for obsessive-compulsive disorder is higher dose selective serotonin reuptake inhibitors such as paroxetine, fluvoxamine, sertraline, citalopram, and fluoxetine.

Haloperidol (choice A) is an antipsychotic medication that has no indication in the treatment of obsessive-compulsive disorder.

Lithium (choice B) is used in the treatment of bipolar disorder and may be used as an augmentation strategy for the treatment of unipolar depression. It has no indication, however, for the treatment of obsessive-compulsive disorder.

Lorazepam (choice C) is an anxiolytic and antiepileptic medication that is effective for treatment of some anxiety disorders such as panic disorder and generalized anxiety disorder. However, it is not indicated in the treatment of obsessive-compulsive disorder.

Valproic acid (choice E) is an anticonvulsant used in the treatment of bipolar disorder. It has no indication in the treatment of obsessive-compulsive disorder.

A 62-year-old man with a long history of cigarette smoking comes to the office with a 3-month history of painless gross hematuria. Physical examination is unremarkable. Urologic evaluation, including cystoscopy, reveals a medium-sized bladder tumor. You recommend a surgeon to the patient and a transurethral resection is performed. The pathology shows high-grade transitional cell carcinoma invading the muscularis propria. A metastatic workup is negative and the patient is counseled regarding radical cystectomy and urinary diversion. A radical cystectomy, pelvic lymph node dissection, and ileal conduit are performed successfully. The surgical margins and lymph nodes are all negative. An 18-month follow-up CT scan of the pelvis reveals a 4-cm heterogeneous, contrast enhancing mass. A biopsy shows a high-grade transitional cell carcinoma. The patient is referred to an oncologist who suggests chemotherapy using a platinum-based regimen. He comes back to your office and tells you that he has heard so many “horror stories” about chemotherapy and that he is concerned about the toxic side effects of the recommended platinum-based regimen. He should be told that this regimen significantly increases his risk of developing

Top of Form

A. cardiac toxicity
B. myelosuppression
C. nephrotoxicity
D. neurotoxicity
E. pulmonary toxicity
Bottom of Form

Explanation:
The correct answer is C. The mainstay of treatment for advanced, metastatic, or recurrent urothelial carcinoma involves platinum based chemotherapy regimens, usually in combination regimens. The most commonly used regimen is MVAC using methotrexate, vinblastine, adriamycin, and cisplatinum. Unfortunately, toxicity often limits the usefulness of these regimens. The toxicity most often associated with platinum is nephrotoxicity. Acute tubular necrosis develops in approximately 25% of patients, and is often the dose-limiting factor. This toxicity can be prevented by keeping patients well hydrated and using diuretics during therapy.

Cardiac toxicity (choice A) is associated with doxorubicin (adriamycin). A cumulative dose-related cardiomyopathy results from doxorubicin treatment and can be fatal.

Myelosuppression (choice B) is an adverse affect of many chemotherapeutic agents. Most noteworthy are methotrexate, vinblastine, and doxorubicin.

Neurotoxicity (choice D) is associated with the vinca alkaloids, especially vincristine. Vincristine can produce a dose-related mixed motor-sensory and autonomic neuropathy.

Pulmonary toxicity (choice E) is associated with bleomycin. Pulmonary fibrosis can result from bleomycin and pulmonary function tests may be necessary in these patients.

A 37-year-old man is hospitalized in an intensive care unit after suffering major head trauma in a motor vehicle accident. The patient was struck by a car two days ago and was found on the scene to be non-responsive with a Glasgow Coma Score of 4. He was intubated at the scene and transferred to the hospital. In the prior two days, his mental status is unchanged and he remains intubated and ventilated. You have documented a discussion you had with the patient three months ago during an admission for pneumonia. During that discussion the patient clearly stated that he would want to be maintained on life support only if he were likely to regain a meaningful quality of life. You and the medical team believe that he does not have a significant chance of regaining an acceptable level of function. The brother and sister maintain that the situation is reversible while the wife continues to desire the withdrawal of care in fulfillment of her husbands’ wishes. The most appropriate next step is to
Top of Form

A. arrange a family meeting hoping to resolve the patient’s previously expressed wishes with those of the children and husband

B. begin the withdrawal of care despite the reservation of the siblings after discussion with the hospital lawyer

C. consult a psychiatrist to speak to the siblings

D. notify the department of social services for the question of spousal abuse

E. refer the case to the ethics committee for review

Bottom of Form

Explanation:

The correct answer is A. Although it is clear that you are ethically bound to follow the patient’s wishes, which in this care appears to be the withdrawal of care, it is always better to do so with the resolve of the family. Often times, feelings such as guilt will drive family members to insist on seemingly unreasonable or inappropriate action. A thoughtful discussion, whereby the family members are allowed to express their reservations in a supportive setting, often will produce a resolution among previous disparate views.

While withdrawing care (choice B) appears to be what ultimately is the most appropriate action, it is worth trying first to have all of the interested parties “on the same page” before preceding if this resolution can be achieved in a timely manner without causing suffering on the patient’s part.

Whereas consult services such as psychiatry (choice C) may be helpful on selected occasions, the first attempts to resolve the conflict should fall on the primary medical team and primary care physicians.

While physicians have the positive duty to report suspected abuse, there is no indication from the information present that the wife has abused her husband (choice D).

Ethics committees (choice E) may be helpful in resolving conflict, but, again, the primary medical providers should first attempt to resolve conflict in order to respect the patient’s wishes in a thoughtful and timely manner.

A young mother brings her 18-month-old son into your office for a well-child examination. She is concerned that he “seems to be slow for his age.” His birth was without incident and his history is remarkable only for an episode of otitis media at 7 months that responded well to antibiotics. He has a 5-year-old sister at home and lives with both parents and a pet dog. When asked to describe her concern more fully, the mother simply states, “He just isn’t doing the same things that his sister did when she was this age.” She reports that he is walking without help, gives hugs and kisses to family members, is able to feed himself with a spoon, and has a vocabulary of about 15 words. However, he is unable to climb stairs by himself, cannot turn a doorknob to open a door, and shows no interest in potty training. His height is 65th percentile and weight is 75th percentile. He does not speak to you, but answers simple questions by nodding or shaking his head. You elicit no specific findings on physical exam. The most appropriate response to the mother’s concerns is:
Top of Form

A. “His lack of social involvement may indicate autism. I’d like to send you to a pediatric psychiatrist for further evaluation.”

B. “I think that your son should be referred to a developmental specialist since I’m concerned that he shows some delay in motor skills.”

C. “I’d like to perform some audiology tests since your son should be speaking in short phrases by now.”

D. “There is no need to be concerned; your son is right where he should be as far as his development.”

E. “You should begin toilet training immediately, he’ll find it extremely difficult if you don’t start before age 2.”

Bottom of Form

Explanation:

The correct answer is D. By the age of 18 months, most children will have developed a specific set of skills which can be divided into: social, self-help, gross motor, fine motor, and language. Social skills at this age include greeting people by saying “hi” or something similar, giving hugs and kisses, and playing patty-cake. Self-help involves drinking from a cup and feeding self with a spoon. Gross motor skills include walking without help and beginning to run. Fine motor skills would include scribbling with a crayon and stacking 2 blocks. Language skills should encompass talking in single words, asking for food or drink with words, and following simple instructions. This patient is doing well regarding each of these milestones.

“His lack of social involvement may indicate autism. I’d like to send you to a pediatric psychiatrist for further evaluation” (choice A) is incorrect since the patient does not exhibit any signs of autism.

Referral to a developmental specialist (choice B) is inappropriate since the patient’s motor skills are at a level with his age. Most children cannot walk up and down stairs alone or open a door using a doorknob until 2 years of age.

An audiology referral (choice C) is incorrect. By 20 months of age a child should know at least 10 words. This patient is able to use 15. Most children do not speak in phrases until after age 2.

“You should begin toilet training immediately, he’ll find it extremely difficult if you don’t start before age 2,” (choice E) is incorrect since the majority of children are not toilet trained until 3 years.

You are seeing a 23-year-old woman with diabetes for a routine office visit. Her regular medications are glyburide and an oral contraceptive pill (OCP). She is an active smoker and drinks about 4 shots of vodka each weekend. She eats “lots of meat and potatoes” and exercises 2 times a week. During your conversation, she mentions that her sister was recently admitted to the hospital with a “blood clot in her lung.” On closer questioning, you also find that her mother and maternal aunt have been previously admitted for thromboembolic phenomena. She also mentions that they have been diagnosed with a problem with their “factor V something.” Based on this information, the most important behavioral modification that you can recommend to reduce this patient’s thromboembolic risk is to

Top of Form

A. follow a low-fat diet
B. increase aerobic exercise
C. maintain tight blood sugar control
D. quit smoking
E. reduce alcohol intake
Bottom of Form

Explanation:
The correct answer is D. The risk of thromboembolic complications in women taking oral contraceptive agents who also smoke, especially in the setting of a possible factor V leiden mutation (to which the patient is hinting), has been estimated to be as high as 30 times the baseline risk. Therefore, given the available history, the patient should be strongly counseled to give up smoking.

Following a low-fat diet (choice A) will help decrease her risk for coronary and vascular disease in the setting of diabetes, but will do little to decrease the risk for thromboses.

Increasing aerobic exercise (choice B) will help the patient with her glucose control and decrease her risk of coronary and vascular disease in the long run, but do little to decrease her long-term risk for thromboses.

Tighter blood sugar control (choice C) will reduce the patients risk for complications from diabetes such as large/small vessel disease and renal disease, but not affect the patient’s risk for developing thromboses.

Reducing alcohol use (choice E), while providing a diminished risk for alcohol related complications, will not affect the patient’s risk for developing thromboses.

A 29-year-old man comes to the office because one of his 3 sexual partners recently had a Pap smear that showed dysplasia and koilocytic changes. Her physician recommended that all of her sexual partners be evaluated. He has always been healthy and has never had any sexually transmitted diseases. All of his partners are “on the pill” so they do not use condoms. Physical examination is completely unremarkable. There are no visible lesions on his anogenital region. He is still very concerned that he has an infection that you cannot see. The most appropriate next step is to
Top of Form

A. advise him to return if he develops any lesions

B. apply vinegar to his penis and scrotum

C. recommend that he use condoms during all sexual activity

D. send for a fluorescent treponemal antibody absorption (FTA-ABS) serology

E. take random biopsies of the penis

F. tell him that he is healthy

Bottom of Form

Explanation:

The correct answer is B. This patient’s girlfriend most likely has human papillomavirus (HPV) infection, which is associated with dysplastic changes and cervical cancer. This patient should be evaluated for an HPV infection, and if there are no visible lesions, acetic acid (vinegar) should be applied to the anogenital region to detect the presence of the virus. Invisible lesions typically turn white when acetic acid is applied. This is thought to occur because the acetic acid causes maceration and swelling of virally induced epithelial hyperplasia, which usually has an increased glycogen content and enhanced permeability. Even though this is not specific for HPV and false-positives can occur, it may enhance the detection of an HPV infection.

If no lesions are found when acetic acid is applied, you should advise him to return if he develops any lesions (choice A) and recommend that he use condoms during all sexual activity (choice C). Condoms will probably not completely prevent the spread of infection, but they should theoretically reduce transmission.

Since his sexual partner most likely has an HPV infection and he is sexually active with many partners, syphilis screening may be appropriate, but the VDRL (Venereal Disease Research Laboratory) and RPR (rapid plasma reagin) tests are used for screening, not the FTA-ABS (choice D). The FTA-ABS is more specific, but it is usually not considered a screening test because it is more expensive and remains reactive in patients with a prior, treated syphilis infection.

Taking random biopsies of the penis (choice E) is completely inappropriate, and it will make a patient very unhappy. Acetic acid should be applied to help see invisible lesions and biopsies can be taken from suspicious areas.

Since many patients infected with HPV have no visible signs and symptoms, it is inappropriate to tell him that he is healthy (choice F) before further evaluation (application of acetic acid).

A 16-year-old high school student is brought to the emergency department because of severe right lower quadrant abdominal pain. She was at volleyball practice when she suddenly doubled over in pain. She has no significant past medical history and had a few episodes of right-sided abdominal pain in the last week. She says that the earlier episodes of pain were much less severe. She is sexually active with one partner, her boyfriend, and they use condoms for birth control. Her temperature 38.8 C (101.8 F), pulse is 90/min, blood pressure is 120/85 mm Hg, and respirations are 14/min. On physical examination she appears very uncomfortable and is in obvious pain. Abdominal examination is significant for focal exquisite tenderness and guarding in the right lower quadrant. Leukocyte count is 11,200/mm3 with 69% segmented neutrophils and 2% band forms. The most appropriate test or study at this time is
Top of Form

A. lower gastrointestinal barium enema

B. ultrasonography of the appendix

C. ultrasonography of the pelvis

D. upper gastrointestinal barium study with small bowel follow through

E. urinalysis

Bottom of Form

Explanation:

The correct answer is B. Severe pain localized to the right lower quadrant, fever, and leukocytosis with bandemia are classic signs and symptoms of acute appendicitis. Acute appendicitis is the most common cause of acute abdomen. The initial study of choice to make this diagnosis is ultrasonography of the appendix. Ultrasound signs of acute appendicitis are a dilated (greater than 6 mm), non-compressible appendix, and visualization of an appendicolith. An appendicolith is calcified fecal material that gets trapped within the appendix and is usually the cause of appendicitis. The sonographic finding of an appendicolith is very specific for appendicitis. If ultrasonography does not reveal such findings, the next appropriate step is a CT of the abdomen and pelvis. A surgery consult should also be called.

Lower gastrointestinal barium enema (choice A) is not the best study in the work up of acute appendicitis. Complete filling of the appendix to the bulbous tip with barium is strong evidence against appendicitis. Nonfilling of the appendix due to an obstructing appendicolith is what you would expect to see in acute appendicitis. However, even in normal barium enema studies the appendix does not always fill. Furthermore, this is a fairly invasive and time consuming diagnostic study which is not appropriate in this case of an acute abdomen.

Ultrasonography of the pelvis (choice C) is not indicated in this patient at this time. Ultrasonography of the pelvis is useful for evaluating the uterus and ovaries. Although she is sexually active, her symptoms do not suggest a disease process involving these pelvic organs. The differential diagnosis for women of child bearing age who present with right lower quadrant pain includes acute appendicitis, ruptured ovarian cyst, and pelvic inflammatory disease. The latter two disease processes, however, are not associated with fever and leukocytosis.

Upper gastrointestinal barium study with small bowel follow through (choice D) is not indicated in this patient. This examination is used to evaluate the esophagus, stomach, and small bowel. Common indications for this diagnostic study include dysphagia, symptoms of peptic ulcer disease, and gastroesophageal reflux.

Urinalysis (choice E) will not provide useful information in the evaluation of this patient. She has classic signs and symptoms of acute appendicitis and a urinalysis will not be significant. Urinalysis is indicated when the diagnosis of a urinary tract infection is suspected. Symptoms of cystitis include urinary frequency and urgency. These symptoms associated with flank pain and fever are suspicious for pyelonephritis.

A 71-year-old man undergoes transurethral resection of the prostate (TURP) for benign prostatic hyperplasia (BPH) under spinal anesthesia. Towards the end of the resection the patient begins to complain of visual color changes. He is alert and oriented and his blood pressure and heart rate have been stable throughout the procedure. The procedure is aborted and the patient is taken to the recovery room. In the recovery room the patient’s blood pressure is 160/110 mm Hg and pulse is 52/min. He is lethargic and confused. The most likely explanation for his symptoms is
Top of Form

A. anemia

B. hypercalcemia

C. hyperkalemia

D. hypermagnesemia

E. hypernatremia

F. hypocalcemia

G. hypokalemia

H. hypomagnesemia

I. hyponatremia

Bottom of Form

Explanation:

The correct answer is I. This patient is suffering from TUR syndrome. This syndrome is caused by the systemic absorption of irrigating fluids through the venous sinuses that are opened during TURP. This leads to hypervolemia and hyponatremia. TUR syndrome occurs in approximately 2-10% of all cases performed and its mortality rate is 0.2-0.8%. Changes may be seen in the cardiopulmonary, hematologic, renal, and central nervous systems in varying degrees. The most common signs and symptoms include hypertension, bradycardia, nausea and vomiting, mental confusion, blurry vision, lethargy, and respiratory distress. The most severe cases can lead to shock, acute renal failure, seizure, coma, and death. The severity of symptoms is usually low until approximately 2 liters of fluid are absorbed, and then can become severe if more than 3 liters are absorbed. Treatment of TUR syndrome requires prompt recognition of symptoms and expeditious termination of the procedure. Delayed absorption may prevent symptoms from manifesting for up to 24 hours. Mental status changes do not occur until the serum sodium is less than 125 mEq/L. Most cases can be treated with the administration of intravenous normal saline and loop diuretics. Sodium correction should be done no faster than 1.5 mmol/L/hr to prevent the risk of central pontine myelinolysis. In the most severe cases intravenous 3% NS may be administered along with loop diuretics. Because of the risk for TUR syndrome, it is important to monitor serum electrolytes at all times. If suspicion for the syndrome is high then sodium levels may be checked during resection. If clinical concern is minimal then electrolyte evaluation may be deferred until the patient reaches the recovery room.

While bleeding and anemia (choice A) is an obvious risk associated with TURP, this would present with hypotension and tachycardia. Currently, the transfusion rate with TURP is approximately 3.5%.

Hypercalcemia (choice B), hyperkalemia (choice C), hypermagnesemia (choice D), hypernatremia (choice E), hypocalcemia (choice F), hypokalemia (choice G), and hypomagnesemia (choice H) are not commonly seen in patients undergoing TURP.

A 70-year-old man with hypertension, hyperlipidemia, and chronic atrial fibrillation is brought to the emergency department for confusion. He was recently diagnosed with multiple myeloma. His medications include furosemide, captopril, atorvostatin, digoxin, and warfarin. He is allergic to penicillin to which he gets a rash. His temperature is 37.0 C (98.6 F), blood pressure is 100/60 mmHg, pulse is 98/min, and respirations are 23/min. Physical examination shows an irregular cardiac rhythm and a soft systolic murmur at his cardiac base. An electrocardiogram shows atrial fibrillation. Laboratory studies show:
Sodium

143 mEq/L

Potassium

4.5 mEq/L

Chloride

104 mEq/L

Bicarbonate

26 mEq/L

Calcium

13 mg/dL

Glucose

109 mg/dL

The most appropriate next step is management is to

Top of Form

A. administer albumin, intravenously

B. administer a dextrose bolus followed by insulin, intravenously

C. administer magnesium sulfate, intravenously

D. administer pamidronate, intravenously

E. hydrate him with normal saline and then administer furosemide, intravenously

Bottom of Form

Explanation:

The correct answer is E. The most appropriate acute treatment of hypercalcemia is hydration followed by a forced diuresis. It is important to hydrate patients prior to administering the diuretic since most patients with this condition are hypovolemic from hypercalcemia induced nausea/vomiting and diabetes insipitus.

Administration of albumin (choice A), albeit a binder of serum calcium, has no role in the management of hypercalcemia.

Administration of dextrose followed by insulin (choice B) is one of the treatments of choice for acute hyperkalemia. It has no role in the management of hypercalcemia.

Administration of magnesium sulfate (choice C) has no role in the management of hypercalcemia.

Administration of pamidronate (choice D), a bisphosphonate which can decrease bone resorption, can be used in the chronic management of hypercalcemia, but has no role in its acute management.

You are seeing a 63–year-old man on rounds in the medical intensive care unit who was admitted with sepsis related to an infected diabetic foot ulcer. During his admission, he has had multiple complications including respiratory failure, a large perioperative myocardial infarction during a left below the knee, amputation, and atrial fibrillation, which resulted in an embolic stroke. He has been intubated and ventilator dependent since admission. Over the past 3 days his condition has been slowly improving and he is starting to regain consciousness. He now indicates that he is having pain in his scrotum. His temperature is 37.0 C (98.6 F), blood pressure is 112/76 mm Hg, pulse is 92/min, respirations are 22/min (on ventilator). His jugular veins are distended, and his heart is irregularly irregular with an S3 gallop. His lungs have course breath sounds bilaterally, abdomen is mildly distended, and his scrotum is markedly and symmetrically enlarged to approximately four times normal size. There is 4+ pitting edema in the lower extremities bilaterally. An ultrasound of the scrotum is performed which shows normal testes and diffuse thickening of the scrotal skin and a small to moderate sized hydrocele on the left and a small hydrocele on the right. The most appropriate course of treatment for his scrotal pain is
Top of Form

A. ciprofloxacin 500 mg via nasogastric tube twice daily for 14 days

B. diuresis as tolerated by his volume status

C. no specific treatment would help

D. percutaneous aspiration of the hydroceles

E. percutaneous aspiration of the hydroceles followed by placement of drainage tubes bilaterally to prevent reaccumualtion

Bottom of Form

Explanation:

The correct answer is B. Many patients who are volume overloaded for various reasons, whether it is due to massive volume resuscitation or congestive heart failure, will develop some degree of scrotal edema. Often times, it can be very impressive and can also cause the patient pain. This patient definitely has signs of heart failure and has likely been heavily volume loaded, because of his sepsis. The only real treatment is to optimize the patient’s volume status and let the body reabsorb the fluid with diuresis as tolerated.

There are no signs of infection mentioned in the clinical scenario, thus treatment with an antibiotic is not necessary (choice A). Occasionally these patients will get some cellulitic type symptoms in the scrotum, but the treatment of choice for that would be something other than ciprofloxacin.

No specific treatment would help (choice C) is incorrect as he should be treated with diureses to improve his volume status.

The hydroceles seen in this patient are not likely to be contributing to the markedly increased size of the testicles, and percutaneous aspiration (choice D) is not necessary. Placing a drainage tube in the scrotum (choice E) is completely unnecessary and not done.

A 42-year-old woman comes to the office because of the sudden onset of “blurry vision” in her left eye. She says that she wears glasses regularly to see distant objects, but now she is seeing “blurry” in both near and distant objects. She has moderate pain, but denies any nausea or vomiting. She does not have any significant past medical history and does not take any medications. The most appropriate method for performing the ophthalmologic examination is to
Top of Form

A. ask the patient to look directly at your nose as you approach with the ophthalmoscope

B. hold the ophthalmoscope in your left hand when examining the patient’s left eye

C. hold the ophthalmoscope 6 inches in front of your face as you approach the patient’s eye

D. hold the ophthalmoscope up to your right eye as you examine the patient’s left eye

E. press the ophthalmoscope against the patient’s cheek and keep the hand that is holding the instrument in the air

Bottom of Form

Explanation:

The correct answer is B. When performing an ophthalmologic examination you should hold the ophthalmoscope in your left hand, up to your left eye when examining the patient’s left eye, and in the right hand when examining the patient’s right eye. If you picture this, you will realize that this allows you to stand on the patient’s side and invade their personal space as little as possible. If you hold the instrument in your right hand, up to your right eye and examine the patient’s left eye, you need to stand directly in front of them, and basically on top of them. This is totally inappropriate. It will make you and the patient very uncomfortable.

You should tell the patient to look straight ahead and stare at a distant target, not to look directly at your nose as you approach with the ophthalmoscope (choice A).

You should not hold the ophthalmoscope 6 inches in front of your face as you approach the patient’s eye (choice C). You should function as a single unit with the instrument by holding it against your own forehead as you approach her. This allows you to have the most coordinated actions, and allows you to visualize the red reflex as you approach.

You invade the patient’s personal space by holding the ophthalmoscope up to your right eye as you examine the patient’s left eye (choice D). You should hold the instrument in your left hand to your left eye when examining the patient’s left eye, and then switch to your right hand and eye to examine the patient’s right eye. Also, you cannot really be as stable and visualize the eye as well if you hold the instrument in your left hand, up to your right eye as you examine the patient’s left eye. And even then you invade their personal space more than if you held it in left hand to your left eye to examine the patient’s left eye.

You should not press the ophthalmoscope against the patient’s cheek and keep the hand that is holding the instrument in the air (choice E). The instrument should be held against your forehead and you can gently hold the hand with the scope against the patient’s cheek, but not the instrument, for stabilization. You are not very stable if you keep the hand with the instrument in the air.

You are called to see a 45-year-old nursing home resident, who has been there since a motor vehicle accident that left him paralyzed from the neck down 2 months ago. He denies any active medical problems prior to his car accident 2 months ago. In the past month, he has noticed a rash on his back that is occasionally pruritic. He denies any systemic manifestations associated with the rash. He is confined to his bed and the nursing staff turns him to his side once per day by propping him back with multiple pillows. He has notable atrophy of all the extremities. Cutaneous examination reveals numerous non-folliculocentric inflammatory papules distributed over his posterior trunk. There is no involvement of the anterior trunk, extremities, face, or oral mucosa. The most appropriate management of this patient’s condition is to
Top of Form

A. prescribe oral minocycline

B. prescribe a topical corticosteroid

C. prescribe a topical tretinoin

D. recommend a topical benzoyl peroxide

E. tell the nurses to rotate the patient on his sides more frequently and keep his room at a cooler temperature

Bottom of Form

Explanation:

The correct answer is E. Rotating the patient and keeping the room at a cooler temperature is correct because miliaria, also known as heat rash, is a common phenomenon in individuals during prolonged bedrest. It usually involves the posterior trunk of a bedridden patient. The eccrine sweat duct occlusion is the initial event. The duct ruptures, leaks sweat into the surrounding tissues, and induces an inflammatory response. The papular and vesicular lesions resemble folliculitis with one major distinguishing feature. They are not follicular and therefore, do not have a penetrating hair shaft. The best treatment for miliaria is to keep the areas involved cool and avoid occlusive clothing.

Oral minocycline (choice A), topical tretinoin (choice C), and topical benzoyl peroxide (choice D), are incorrect because these are treatments for folliculitis or acneiform eruptions. Since the inflammatory papules on this patient are described as non-folliculocentric, folliculitis is not likely the diagnosis.

A topical corticosteroid (choice B) is incorrect because use of corticosteroids topically may result in miliaria as an adverse reaction.

You are called to the well-baby nursery after a 27-year-old patient of yours, delivers a healthy- appearing term baby boy by normal spontaneous vaginal delivery. You recall that this patient had an uncomplicated prenatal course, however, initial laboratory studies during her first antenatal visit were positive for hepatitis B surface antigen. A complete physical examination of the infant is unremarkable. You should
Top of Form

A. advise the mother not to breast-feed her newborn

B. give routine care to the newborn

C. give the hepatitis B vaccine and hepatitis B immune globulin to the newborn

D. reassure the mother that no hepatitis B vaccine or treatment is needed for her newborn

E. send hepatitis serology from the newborn and await results before providing care

Bottom of Form

Explanation:

The correct answer is C. Infants born to mothers who are HBsAg positive should be given the hepatitis B vaccine and hepatitis B immune globulin at different sites within 12 hours of birth. The second and third doses of the vaccine should be given at 1 month and 6 months. They should be tested for anti-HBs and HBsAg 1-3 months after completing the immunization series.

Advising the mother not to breast-feed her child (choice A) is incorrect. HBsAg-positive mothers can breast-feed because according to the American Academy of Pediatrics and the Report of the Committee on Infectious Diseases, the infant is at no additional risk of acquiring the virus.

Routine care (choice B), which is given to infants born to HBsAg-negative mothers, would be to give a series of 3 hepatitis B vaccines. The first dose at birth to 2 months, the second dose 1 month later, and the third dose at least 4 months after the first dose and 2 months after the second dose, but not before 6 months of age. This infant is born to a HBsAg-positive mother and so he also requires the hepatitis B immune globulin to prevent infection.

Reassuring the mother that no hepatitis B vaccine or treatment is needed for her newborn (choice D) is inappropriate because as stated above, this infant should be given the hepatitis B vaccine and hepatitis B immune globulin.

It is incorrect to send hepatitis serology from the newborn and await results before providing care (choice E). He should be given the hepatitis B vaccine and hepatitis B immune globulin as soon after birth as possible. You should not wait until any laboratory studies return.

A 53-year-old widowed woman comes to the office for a health maintenance examination. She is a new patient who recently moved to your city after her husband died in an office fire 6 months ago. She says that she has no complaints, except for a cough that she began to notice 4 months ago. She denies nasal discharge, “a tickle in the throat,” frequent throat clearing, heartburn and the sensation of regurgitation, fever, sputum production, cigarette smoking, illegal drug use, sexual activity, occupational exposures, and any other symptoms associated with a respiratory infection. She says that the cough is not seasonal or associated with wheezing. Her temperature is 37.0 C (98.6 F), blood pressure is 135/90 mm Hg, pulse is 70/min, and respirations are 14/min. Physical examination is unremarkable. The most appropriate next step is to
Top of Form

A. order an electrocardiogram

B. order an x-ray of the chest

C. question her about medications

D. refer her for fiberoptic bronchoscopy

E. schedule her for pulmonary function tests

Bottom of Form

Explanation:

The correct answer is C. This patient has a chronic cough, which is usually considered chronic because it is lasting more than 3 weeks. It may be due to a variety of things. However, the important lesson in this question is that before you turn to diagnostic studies you need, to make sure that you have obtained a detailed history. The case history will provide the answer to almost every question that you will need to ask her, except what medications she takes. Since she is a new patient, you will need to find out if she is taking an ACE inhibitor, such as captopril or enalapril, which is a frequent cause of a chronic cough in hypertensive patients. They cause a cough in up to 20% of people taking them. The exact mechanism is unknown, but it is thought to somehow be related to bradykinin and substance P. The treatment for the cough is the discontinuation of the ACE inhibitor.

An electrocardiogram (choice A) is unnecessary at this time in this patient, complaining of a chronic cough. She is not complaining of chest pain and there is nothing in her history that suggests an arrhythmia. The most important next step, is to take a detailed history before you order diagnostic tests.

An x-ray of the chest (choice B) may be appropriate in the near future, but it is not the next step at this time. Before you order diagnostic studies, you need to make sure that you ask her any questions that might help you figure out the etiology of her cough. Asking her about medications is very important because ACE inhibitors cause a chronic cough in up to 20% of patients taking this medication.

A fiberoptic bronchoscopy (choice D) is used to obtain histologic and cytologic specimens and to visualize an endobronchial tumor. Before you turn to such a specialized study, you need to first obtain a detailed history. If the patient is not taking an ACE inhibitor, a chest x-ray should usually be performed, and if this is abnormal, sputum cytology, a high resolution CT scan, and fiberoptic bronchoscopy should be considered.

Pulmonary function tests (choice E) are used to assess airway hyperresponsiveness for patients in which you suspect asthma, and lung volumes and diffusion capacity in patients in which you suspect a diffuse interstitial lung disease. A detailed history is necessary before using any of these studies.

A 73-year-old man with emphysema comes to the clinic with complaints of food getting stuck when he swallows, which has been getting progressively worse over the last 8 months. He denies problems swallowing liquids and thinks he has lost about 5 pounds. He used alcohol heavily for many years but quit drinking 10 years ago. He still smokes 1 pack of cigarettes per day and has done so since age 20. He uses albuterol, steroid inhalers and theophylline. His blood pressure is 123/73 mm Hg, pulse is 87/min, and respirations are 20/min. Physical examination reveals bilateral scattered wheezes in the lungs. A chest x-ray shows hyperexpansion and no nodules. The most appropriate next step in management is to
Top of Form

A. order a barium esophagram

B. order an esophageal manometry

C. order an esophageal pH probe

D. treat with omeprazole and follow up in 3 months

E. treat with ranitidine and follow up in 3 months

Bottom of Form

Explanation:

The correct answer is A. This patient most likely has an esophageal squamous cell carcinoma (the most common type of esophageal malignancy). In any patient with dysphagia that is progressive for only solids, it suggests a growing and obstructive lesion. The history of tobacco and alcohol use, puts this person at a much higher risk of carcinoma. The two ways to diagnose this are a barium swallow study, which will show the mucosal mass, or an upper endoscopy study to directly visualize and biopsy the lesion.

Esophageal manometry (choice B) is used to evaluate dysphagia caused by motility disorders. These typically present with dysphagia for solids and liquids and may or may not be progressive.

A pH probe (choice C) is used to evaluate esophageal reflux disease, which does not in itself typically cause dysphagia, but over long periods of time will increase the risk of esophageal adenocarcinoma.

Both omeprazole (choice D) and ranitidine (choice E) are used to treat symptoms of gastroesophageal reflux disease and would not address his dysphagia. Furthermore, waiting 3 months to see the patient again would be inappropriate.

A 26-year-old woman comes to the office for routine postpartum care after delivering a healthy baby boy 6 days earlier. This is her first child and she is concerned about her breastfeeding, diaper changing, and bathing techniques. You ask her to explain how she performs these activities, and it seems as if she is doing everything correctly. She then tells you that she is very upset about her weight, that she gained 45 pounds during the pregnancy, and now is still 37 pounds “overweight”. Her husband is a good friend of yours from medical school. Her blood pressure is 140/90 mm Hg and her pulse is 95/min. Physical examination shows dilated pupils, but is otherwise unremarkable. You suspect that she is using drugs and so you ask her in a direct, nonjudgmental way. She admits to using cocaine twice since the delivery of her son. The most appropriate next step is to
Top of Form

A. advise her to stay away from her son when she is using cocaine

B. call her husband and ask if he knows that she is using drugs

C. contact the child welfare association and inform them of your findings

D. obtain a urine sample to test for cocaine and other illegal drugs

E. tell her that she should not breast feed her son if she is using cocaine

Bottom of Form

Explanation:

The correct answer is E. Cocaine is contraindicated during breast feeding and so you need to advise the mother to stop breast feeding if she is using cocaine. Possible effects of cocaine in the breast milk include growth retardation and neurologic damage. Cocaine use during pregnancy is associated with fetal demise, spontaneous abortions, premature rupture of membranes, preterm labor and delivery, placental abruption, intrauterine growth retardation, and congenital anomalies. Other medications and drugs that are contraindicated during breastfeeding are bromocriptine, cyclophosphamide, cyclosporine, doxorubicin, lithium, ergotamine, methotrexate, amphetamines, heroin, marijuana, nicotine, and phencyclidine.

The most appropriate next step is to tell her is to discontinue breastfeeding if she is using cocaine, not to advise her to stay away from her son when she is using cocaine (choice A). This answer makes it seem like it is okay that she is using drugs, but she should just not be near her baby. This is actually not true. If she is using the cocaine to lose weight, you should recommend a healthy diet and exercise regimen.

It is incorrect to call her husband and ask if he knows that she is using drugs (choice B) or to contact the child welfare association and inform them of your findings (choice C). You should first try to reason with the patient and encourage her to discontinue using cocaine and tell her that she is not only risking her own life now, but also that of her son’s because she is breast feeding.

She already admitted to you that she is using cocaine and so it is unnecessary to obtain a urine sample to test for cocaine and other illegal drugs (choice D).

An 18-year-old boy with Hodgkin’s lymphoma is admitted to the hospital for his third course of chemotherapy which includes prednisone. On his 6th hospital day, you are called by the nurse to assess a new rash which has slowly progressed to cover the entire anterior and posterior trunk, as well as his arms and part of the face. The nurse is concerned about a possible allergic reaction, secondary to the multiple medications this patient is taking. On examination, you note numerous, small, pinpoint pustules and inflammatory papules of the same stage scattered mostly over his trunk and proximal arms. There are some pustules of the same stage over his forehead as well. The patient denies any significant history of acne prior to the initiation of chemotherapy. At this time the most correct statement about his condition is:
Top of Form

A. This is acne vulgaris and should be treated with oral minocycline and topical benzoyl peroxide

B. This is a drug eruption and you should stop all the medications and reinitiate one every week

C. This is a drug eruption secondary to his chemotherapy and topical corticosteroid will alleviate the problem

D. This is miliaria secondary to the occlusion effect by his hospital gown

E. This is steroid acne and topical tretinoin cream may help

Bottom of Form

Explanation:

The correct answer is E. Acneiform eruptions is correct because it is characterized by papules and pustules resembling acne lesions, not necessarily confined to the usual sites of acne vulgaris. The eruptions are distinguished by their sudden onset. Oral medications such as iodides, bromides, testosterone, cyclosporine, antiepileptic medications, lithium, and systemic corticosteroids are common agents that can lead to acneiform eruption. When medium or high doses of corticosteroids are taken for as short a time as 3-5 days, a distinctive eruption may occur, known as steroid acne. It is a sudden out-cropping of inflamed papules, most numerous on the upper trunk and arms, but also seen on the face. The lesions typically present as papules rather than comedones. Tretinoin cream applied once or twice daily may clear the lesions within 1-3 months, despite the continuation of high doses of corticosteroid.

Acne vulgaris (choice A) is incorrect because of the atypical distribution (proximal arms), and the sudden onset of numerous lesions.

An allergic drug eruption (choice B) is incorrect because typical drug eruptions do not present with pustules. His medication should not be terminated.

Typical drug eruptions do not present with pustules and topical steroids (choice C), especially of the fluorinated types, or when applied under occlusion, may also induce an acneiform eruption and worsen this patient’s skin problem

Miliaria (choice D) is incorrect. This is retention of sweat as a result of the occlusion of eccrine sweat ducts and pores, producing an eruption that is common in patients with prolonged bedrest. These patients typically present with numerous folliculocentric inflammatory papules on the posterior trunk, resulting from extended periods of being on their back. The face, central chest, and arms are typically not occluded in bedrest patients.

A 45-year-old disheveled man is brought to the emergency department by paramedics stating, “I can’t see!” The paramedics tell you that he is a homeless man who was found this morning locked in an automotive garage when the manager of the garage went to open for business. They also state that he was found next to an empty bottle of window washing fluid and an empty 40oz beer can. The patient is alert, but uncooperative during questioning and keeps repeating “Doctor… I just can’t see!” As you are examining him, he begins to retch, holds his abdomen and vomits twice. His temperature is 37 C (98.6 F), blood pressure is 150/92 mm Hg, pulse is 100/min, respirations are 24/min, and oxygen saturation is 97% on room air. His pupils are dilated at 5mm and are only sluggishly reactive. Funduscopic examination reveals hyperemia of both discs without papilledema; the cranial nerves are otherwise intact. The remainder of the physical examination is unremarkable except you notice that he is taking very deep and rapid breaths. The most appropriate first diagnostic step is to
Top of Form

A. administer intravenous ethanol and monitor for reversal of his visual deficit

B. observe him in the emergency department as this man is obviously intoxicated with alcohol and just needs to sleep it off

C. order arterial blood gas and serum electrolytes and osmolality

D. perform a CT scan of his head to rule out intracranial pathology

E. request an immediate ophthalmology consult

Bottom of Form

Explanation:

The correct answer is C. This patient presents with the classic signs and symptoms of acute methanol toxicity. Methanol is commonly found in windshield washing fluid, solvents, and paint thinners. Methanol is converted by alcohol dehydrogenase into formaldehyde, which is then converted into formic acid, which produces an anion gap metabolic acidosis. The formic acid also affects the optic nerve function resulting in optic papillitis and retinal edema leading to blindness. An arterial blood gas would help you determine the pH and amount of acidemia, with an associated low bicarbonate level and likely low pCO2 level from respiratory compensation. The respiratory compensation for a metabolic acidosis would explain his rapid respiratory rate at 24/min. The serum electrolyte panel will help determine the anion gap using the formula (Anion Gap = Na-Cl-HCO3). The anion gap is normally less than 12. Methanol poisoning also produces an osmol gap which is the difference between the measured osmolality and calculated osmolarity.

The normal measured serum osmolality is 280-295mOsm (milliosmols). The calculated osmolarity can be determined using the formula [osmolarity = (2xNa)+(BUN/2.8)+(glucose/18)]. An osmol gap of greater than 10mOsm is abnormal and indicates that osmotically active substances are present in the blood (methanol, ethylene glycol, isopropyl alcohol).

Administration of IV Ethanol (choice A) may be necessary as a treatment for methanol toxicity, as well as possible dialysis. However, diagnostic studies need to be performed first to confirm the diagnosis.

Observation without intervention (choice B) is improper management as this patient warrants further investigation.

A CT scan of the head (choice D) is likely to be normal in this patient, and thus of minimal benefit in ascertaining a diagnosis.

An ophthalmology consultation (choice E) will help rule out other pathological causes of blindness, but should not be your first step in the management of this particular patient.